Site Loader

Π‘ΠΎΠ΄Π΅Ρ€ΠΆΠ°Π½ΠΈΠ΅

ΠŸΠΎΡ‡Π΅ΠΌΡƒ всё Π²ΠΎΠΊΡ€ΡƒΠ³ Ρ‚Π°ΠΊΠΎΠ΅, ΠΊΠ°ΠΊΠΎΠ΅ ΠΎΠ½ΠΎ Π΅ΡΡ‚ΡŒ?

ΠŸΠΎΡ‡Π΅ΠΌΡƒ всё Π²ΠΎΠΊΡ€ΡƒΠ³ Ρ‚Π°ΠΊΠΎΠ΅, ΠΊΠ°ΠΊΠΎΠ΅ ΠΎΠ½ΠΎ Π΅ΡΡ‚ΡŒ?

Π‘Π΅Π»Π»ΡƒΡ€ Биварамия ЧандрасСкар

ΠŸΡ€ΠΎΠ΄ΠΎΠ»ΠΆΠ΅Π½ΠΈΠ΅. Π‘ΠΌ. β„– 2, 4, 6/05

Β 3. ЭлСктричСскиС ΠΈ ΠΌΠ°Π³Π½ΠΈΡ‚Π½Ρ‹Π΅ поля (ΠΏΡ€ΠΎΠ΄ΠΎΠ»ΠΆΠ΅Π½ΠΈΠ΅)

ΠŸΡ€Π΅Π΄ΡΡ‚Π°Π²ΠΈΠΌ Ρ‚Π΅ΠΏΠ΅Ρ€ΡŒ Π½Π΅ΠΊΠΎΡ‚ΠΎΡ€Ρ‹ΠΉ ΠΎΠ±ΡŠΡ‘ΠΌ пространства, Π² ΠΊΠΎΡ‚ΠΎΡ€ΠΎΠΌ Π½Π΅Ρ‚ вСщСства, Π½ΠΎ Π² ΠΊΠ°ΠΆΠ΄ΠΎΠΉ Ρ‚ΠΎΡ‡ΠΊΠ΅ Π·Π°Π΄Π°Π½Ρ‹ элСктричСскоС ΠΈ ΠΌΠ°Π³Π½ΠΈΡ‚Π½ΠΎΠ΅ поля. Для Π½Π°Ρ‡Π°Π»Π° ΠΏΡ€Π΅Π΄ΠΏΠΎΠ»ΠΎΠΆΠΈΠΌ, Ρ‡Ρ‚ΠΎ эти поля Π½Π΅ зависят ΠΎΡ‚ Π²Ρ€Π΅ΠΌΠ΅Π½ΠΈ, хотя ΠΌΠΎΠ³ΡƒΡ‚ ΠΏΡ€ΠΈΠ½ΠΈΠΌΠ°Ρ‚ΡŒ Ρ€Π°Π·Π½Ρ‹Π΅ значСния Π² Ρ€Π°Π·Π½Ρ‹Ρ… Ρ‚ΠΎΡ‡ΠΊΠ°Ρ… пространства Π²Π½ΡƒΡ‚Ρ€ΠΈ Π²Ρ‹Π΄Π΅Π»Π΅Π½Π½ΠΎΠ³ΠΎ ΠΎΠ±ΡŠΡ‘ΠΌΠ°. Π­Ρ‚ΠΈ поля ΠΏΠΎΡ€ΠΎΠΆΠ΄Π΅Π½Ρ‹ Π½Π΅ΠΊΠΎΡ‚ΠΎΡ€Ρ‹ΠΌ распрСдСлСниСм зарядов, Ρ‚ΠΎΠΊΠΎΠ² ΠΈ постоянных ΠΌΠ°Π³Π½ΠΈΡ‚ΠΎΠ², ΠΊΠΎΡ‚ΠΎΡ€Ρ‹Π΅ находятся Π²Π½Π΅ Π²Ρ‹Π΄Π΅Π»Π΅Π½Π½ΠΎΠ³ΠΎ ΠΎΠ±ΡŠΡ‘ΠΌΠ°. Нам Π½Π΅ Π½ΡƒΠΆΠ½ΠΎ Π½ΠΈΡ‡Π΅Π³ΠΎ Π·Π½Π°Ρ‚ΡŒ ΠΎΠ± этих распрСдСлСниях, Π²Π°ΠΆΠ½ΠΎ Ρ‚ΠΎΠ»ΡŒΠΊΠΎ, Ρ‡Ρ‚ΠΎ ΠΎΠ½ΠΈ ΡΡƒΡ‰Π΅ΡΡ‚Π²ΡƒΡŽΡ‚.

Рассмотрим поля Π²Π½ΡƒΡ‚Ρ€ΠΈ ΠΎΠ±ΡŠΡ‘ΠΌΠ°. ΠŸΡ€Π΅Π΄ΠΏΠΎΠ»ΠΎΠΆΠΈΠΌ, Ρ‡Ρ‚ΠΎ с зарядами, ΡΠΎΠ·Π΄Π°ΡŽΡ‰ΠΈΠΌΠΈ элСктричСскоС ΠΏΠΎΠ»Π΅, Ρ‡Ρ‚ΠΎ-Ρ‚ΠΎ ΡΠ»ΡƒΡ‡ΠΈΠ»ΠΎΡΡŒ ΠΈ ΠΎΠ½ΠΈ Π½Π°Ρ‡Π°Π»ΠΈ Π΄Π²ΠΈΠ³Π°Ρ‚ΡŒΡΡ, Ρ‚Π°ΠΊ Ρ‡Ρ‚ΠΎ элСктричСскоС ΠΏΠΎΠ»Π΅ Π½Π°Ρ‡Π°Π»ΠΎ ΠΈΠ·ΠΌΠ΅Π½ΡΡ‚ΡŒΡΡ со Π²Ρ€Π΅ΠΌΠ΅Π½Π΅ΠΌ.

ΠœΡ‹ ΠΎΠ±Π½Π°Ρ€ΡƒΠΆΠΈΠ²Π°Π΅ΠΌ Π½ΠΎΠ²ΠΎΠ΅ явлСниС: ΠΈΠ·ΠΌΠ΅Π½ΡΡŽΡ‰Π΅Π΅ΡΡ элСктричСскоС ΠΏΠΎΠ»Π΅ ΠΏΠΎΡ€ΠΎΠΆΠ΄Π°Π΅Ρ‚ ΠΌΠ°Π³Π½ΠΈΡ‚Π½ΠΎΠ΅ ΠΏΠΎΠ»Π΅, Π½Π΅ ΠΈΠΌΠ΅ΡŽΡ‰Π΅Π΅ Π½ΠΈΠΊΠ°ΠΊΠΎΠ³ΠΎ ΠΎΡ‚Π½ΠΎΡˆΠ΅Π½ΠΈΡ ΠΊ полю, созданному постоянными ΠΌΠ°Π³Π½ΠΈΡ‚Π°ΠΌΠΈ ΠΈΠ»ΠΈ ΠΏΡ€ΠΎΠ²ΠΎΠ΄Π°ΠΌΠΈ с Ρ‚ΠΎΠΊΠΎΠΌ Π²Π½Π΅ ΠΎΠ±ΡŠΡ‘ΠΌΠ°. Π’ΠΎΠ·Π½ΠΈΠΊΡˆΠ΅Π΅ ΠΌΠ°Π³Π½ΠΈΡ‚Π½ΠΎΠ΅ ΠΏΠΎΠ»Π΅ связано Ρ‚ΠΎΠ»ΡŒΠΊΠΎ с ΠΈΠ·ΠΌΠ΅Π½ΡΡŽΡ‰ΠΈΠΌΡΡ элСктричСским ΠΏΠΎΠ»Π΅ΠΌ. ΠΠ½Π°Π»ΠΎΠ³ΠΈΡ‡Π½ΡƒΡŽ ΠΊΠ°Ρ€Ρ‚ΠΈΠ½Ρƒ ΠΌΡ‹ Π²ΠΈΠ΄ΠΈΠΌ ΠΈ Π² Ρ‚ΠΎΠΌ случаС, Ссли поля ΠΌΠ΅Π½ΡΡŽΡ‚ΡΡ ролями: ΠΈΠ·ΠΌΠ΅Π½ΡΡŽΡ‰Π΅Π΅ΡΡ (Π½Π°ΠΏΡ€ΠΈΠΌΠ΅Ρ€, Π·Π° счёт измСнСния со Π²Ρ€Π΅ΠΌΠ΅Π½Π΅ΠΌ Ρ‚ΠΎΠΊΠ° Π² ΠΏΡ€ΠΎΠ²ΠΎΠ΄Π°Ρ…) ΠΌΠ°Π³Π½ΠΈΡ‚Π½ΠΎΠ΅ ΠΏΠΎΠ»Π΅ ΠΏΠΎΡ€ΠΎΠΆΠ΄Π°Π΅Ρ‚ элСктричСскоС ΠΏΠΎΠ»Π΅. ΠœΡ‹ Π²ΠΈΠ΄ΠΈΠΌ ΠΎΠΏΡ€Π΅Π΄Π΅Π»Ρ‘Π½Π½ΡƒΡŽ ΡΠΈΠΌΠΌΠ΅Ρ‚Ρ€ΠΈΡŽ ΠΌΠ΅ΠΆΠ΄Ρƒ элСктричСским ΠΈ ΠΌΠ°Π³Π½ΠΈΡ‚Π½Ρ‹ΠΌ полями. ИзмСнСниС со Π²Ρ€Π΅ΠΌΠ΅Π½Π΅ΠΌ ΠΎΠ΄Π½ΠΎΠ³ΠΎ ΠΈΠ· Π½ΠΈΡ… ΠΏΠΎΡ€ΠΎΠΆΠ΄Π°Π΅Ρ‚ Π΄Ρ€ΡƒΠ³ΠΎΠ΅. Π’ΠΎΡ‡Π½Ρ‹Π΅ Π·Π°ΠΊΠΎΠ½Ρ‹, ΡƒΠΏΡ€Π°Π²Π»ΡΡŽΡ‰ΠΈΠ΅ ΠΏΠΎΠ²Π΅Π΄Π΅Π½ΠΈΠ΅ΠΌ ΠΏΠΎΠ»Π΅ΠΉ, Π±Ρ‹Π»ΠΈ записаны Π² матСматичСской Ρ„ΠΎΡ€ΠΌΠ΅ ДТСймсом ΠšΠ»Π΅Ρ€ΠΊΠΎΠΌ МаксвСллом. ΠΠ»ΡŒΠ±Π΅Ρ€Ρ‚ Π­ΠΉΠ½ΡˆΡ‚Π΅ΠΉΠ½ Π΄ΠΎΠΊΠ°Π·Π°Π», Ρ‡Ρ‚ΠΎ симмСтрия ΠΌΠ΅ΠΆΠ΄Ρƒ двумя полями Π²Ρ‹Ρ‚Π΅ΠΊΠ°Π΅Ρ‚ ΠΈΠ· Ρ‚Π΅ΠΎΡ€ΠΈΠΈ ΠΎΡ‚Π½ΠΎΡΠΈΡ‚Π΅Π»ΡŒΠ½ΠΎΡΡ‚ΠΈ.

ΠŸΡ€ΠΎΡΡ‚ΠΎΠΉ способ ΠΏΠΎΡ€ΠΎΠ΄ΠΈΡ‚ΡŒ ΠΏΠ΅Ρ€Π΅ΠΌΠ΅Π½Π½ΠΎΠ΅ Π²ΠΎ Π²Ρ€Π΅ΠΌΠ΅Π½ΠΈ ΠΌΠ°Π³Π½ΠΈΡ‚Π½ΠΎΠ΅ ΠΏΠΎΠ»Π΅ – это Π·Π°ΡΡ‚Π°Π²ΠΈΡ‚ΡŒ Ρ‚ΠΎΠΊ Π² ΠΏΡ€ΠΎΠ²ΠΎΠ΄Π΅ Π½Π΅ΠΏΡ€Π΅Ρ€Ρ‹Π²Π½ΠΎ ΠΌΠ΅Π½ΡΡ‚ΡŒ своС Π½Π°ΠΏΡ€Π°Π²Π»Π΅Π½ΠΈΠ΅, ΠΌΠ΅Π½ΡΡ‚ΡŒΡΡ с частотой  fΒ  Ρ€Π°Π· Π² сСкунду. Π­Ρ‚ΠΎ ΠΎΠ·Π½Π°Ρ‡Π°Π΅Ρ‚, Ρ‡Ρ‚ΠΎ Ρ‚ΠΎΠΊ достигаСт максимума, ΡƒΠΌΠ΅Π½ΡŒΡˆΠ°Π΅Ρ‚ΡΡ Π΄ΠΎ нуля, мСняСт Π½Π°ΠΏΡ€Π°Π²Π»Π΅Π½ΠΈΠ΅, вновь достигаСт максимума ΠΈ ΡƒΠΌΠ΅Π½ΡŒΡˆΠ°Π΅Ρ‚ΡΡ Π΄ΠΎ нуля, ΠΈ Ρ‚Π°ΠΊ

f Ρ€Π°Π· Π·Π° сСкунду. Π’ΠΎΠΊ ΡΠΎΠ²Π΅Ρ€ΡˆΠ°Π΅Ρ‚ Ρ‚Π°ΠΊ Π½Π°Π·Ρ‹Π²Π°Π΅ΠΌΡ‹Π΅ гармоничСскиС колСбания. Π‘Π»Π΅Π΄ΠΎΠ²Π°Ρ‚Π΅Π»ΡŒΠ½ΠΎ, ΠΌΠ°Π³Π½ΠΈΡ‚Π½ΠΎΠ΅ ΠΏΠΎΠ»Π΅ Π² ΠΊΠ°ΠΆΠ΄ΠΎΠΉ Ρ‚ΠΎΡ‡ΠΊΠ΅ Π±ΡƒΠ΄Π΅Ρ‚ Ρ‚Π°ΠΊΠΆΠ΅ ΡΠΎΠ²Π΅Ρ€ΡˆΠ°Ρ‚ΡŒ гармоничСскиС голСбания. Богласно ΠœΠ°ΠΊΡΠ²Π΅Π»Π»Ρƒ, ΠΎΡΡ†ΠΈΠ»Π»ΠΈΡ€ΡƒΡŽΡ‰Π΅Π΅ ΠΌΠ°Π³Π½ΠΈΡ‚Π½ΠΎΠ΅ ΠΏΠΎΠ»Π΅ Π±ΡƒΠ΄Π΅Ρ‚ ΠΏΠΎΡ€ΠΎΠΆΠ΄Π°Ρ‚ΡŒ ΠΎΡΡ†ΠΈΠ»Π»ΠΈΡ€ΡƒΡŽΡ‰Π΅Π΅ элСктричСскоС ΠΏΠΎΠ»Π΅ ΠΈ Ρ‚.Π΄. КолСбания ΠΊΠ°ΠΆΠ΄ΠΎΠ³ΠΎ поля Π²Ρ‹Π·Ρ‹Π²Π°ΡŽΡ‚ колСбания Π΄Ρ€ΡƒΠ³ΠΎΠ³ΠΎ. Π­Ρ‚ΠΈ колСбания Ρ€Π°ΡΠΏΡ€ΠΎΡΡ‚Ρ€Π°Π½ΡΡŽΡ‚ΡΡ Π½Π°Ρ€ΡƒΠΆΡƒ ΠΎΡ‚ ΠΏΡ€ΠΎΠ²ΠΎΠ΄Π° со ΡΠΊΠΎΡ€ΠΎΡΡ‚ΡŒΡŽ, которая опрСдСляСтся ΡΠΎΠΎΡ‚Π½ΠΎΡˆΠ΅Π½ΠΈΠ΅ΠΌ ΠΌΠ΅ΠΆΠ΄Ρƒ двумя ΠΊΠΎΠ»Π΅Π±Π»ΡŽΡ‰ΠΈΠΌΠΈΡΡ полями Π² пустом пространствС (ΠΈ ΡΠ²Π»ΡΡŽΡ‰Π΅ΠΉΡΡ, Ρ‚Π°ΠΊΠΈΠΌ ΠΎΠ±Ρ€Π°Π·ΠΎΠΌ, свойством пустого пространства). Π­Ρ‚Π° ΡΠΊΠΎΡ€ΠΎΡΡ‚ΡŒ Ρ€Π°Π²Π½Π° скорости свСта. Волько Ρ‡Ρ‚ΠΎ описанный экспСримСнт являСтся, ΠΏΠΎ сущСству, способом Π³Π΅Π½Π΅Ρ€Π°Ρ†ΠΈΠΈ Ρ€Π°Π΄ΠΈΠΎΠ²ΠΎΠ»Π½. ΠœΡ‹ ΠΏΠΎΠ½ΠΈΠΌΠ°Π΅ΠΌ Ρ‚Π΅ΠΏΠ΅Ρ€ΡŒ, ΠΏΠΎΡ‡Π΅ΠΌΡƒ эти Π²ΠΎΠ»Π½Ρ‹ Π½Π°Π·Ρ‹Π²Π°ΡŽΡ‚ΡΡ элСктромагнитными: ΠΊΠΎΠ»Π΅Π±Π»ΡŽΡ‰ΠΈΠΌΠΈΡΡ ΠΎΠ±ΡŠΠ΅ΠΊΡ‚Π°ΠΌΠΈ ΡΠ²Π»ΡΡŽΡ‚ΡΡ элСктричСскоС ΠΈ ΠΌΠ°Π³Π½ΠΈΡ‚Π½ΠΎΠ΅ поля.
ΠŸΠΎΠΊΠ°Π·Π°Π½Π½Ρ‹ΠΉ Π½Π° рис. 9-1 спСктр ΠΎΠ±Ρ€Π°Π·ΠΎΠ²Π°Π½ элСктромагнитными Π²ΠΎΠ»Π½Π°ΠΌΠΈ, Ρ€Π°ΡΠΏΡ€ΠΎΡΡ‚Ρ€Π°Π½ΡΡŽΡ‰ΠΈΠΌΠΈΡΡ Π² пустом пространствС со ΡΠΊΠΎΡ€ΠΎΡΡ‚ΡŒΡŽ 3.108 ΠΌ/с ΠΈ ΠΎΡ‚Π»ΠΈΡ‡Π°ΡŽΡ‰ΠΈΠΌΠΈΡΡ Π΄Ρ€ΡƒΠ³ ΠΎΡ‚ Π΄Ρ€ΡƒΠ³Π° Ρ‚ΠΎΠ»ΡŒΠΊΠΎ частотой ΠΈΠ»ΠΈ ΡΠΎΠΎΡ‚Π²Π΅Ρ‚ΡΡ‚Π²ΡƒΡŽΡ‰Π΅ΠΉ Π΄Π»ΠΈΠ½ΠΎΠΉ Π²ΠΎΠ»Π½Ρ‹.

На рис. 9-5 ΠΏΠΎΠΊΠ°Π·Π°Π½ ΠΌΠ³Π½ΠΎΠ²Π΅Π½Π½Ρ‹ΠΉ снимок участка элСктромагнитной Π²ΠΎΠ»Π½Ρ‹. Π‘ этой Π²ΠΎΠ»Π½ΠΎΠΉ связаны Ρ‚Ρ€ΠΈ Π²Π°ΠΆΠ½Ρ‹Ρ… направлСния: Π½Π°ΠΏΡ€Π°Π²Π»Π΅Π½ΠΈΠ΅ распространСния Π²ΠΎΠ»Π½Ρ‹ (ΠΏΠΎΠΊΠ°Π·Π°Π½Π½ΠΎΠ΅ Π²Π΅ΠΊΡ‚ΠΎΡ€ΠΎΠΌ

k) ΠΈ Π΄Π²Π° направлСния, вдоль ΠΊΠΎΡ‚ΠΎΡ€Ρ‹Ρ… Π½Π°ΠΏΡ€Π°Π²Π»Π΅Π½Ρ‹ Π² пространствС элСктричСскоС ΠΈ ΠΌΠ°Π³Π½ΠΈΡ‚Π½ΠΎΠ΅ поля Π• ΠΈ Π’. Π­Ρ‚ΠΈ Ρ‚Ρ€ΠΈ направлСния Π²Π·Π°ΠΈΠΌΠ½ΠΎ пСрпСндикулярны. Вакая Π²ΠΎΠ»Π½Π° отличаСтся ΠΎΡ‚ Π²ΠΎΠ»Π½, ΠΊΠΎΡ‚ΠΎΡ€Ρ‹Π΅ ΠΌΡ‹ ΠΌΠΎΠΆΠ΅ΠΌ Π½Π°Π±Π»ΡŽΠ΄Π°Ρ‚ΡŒ Π² ΠΎΠΊΡ€ΡƒΠΆΠ°ΡŽΡ‰Π΅ΠΌ нас ΠΌΠΈΡ€Π΅, Π½Π°ΠΏΡ€ΠΈΠΌΠ΅Ρ€, ΠΎΡ‚ Π²ΠΎΠ»Π½ Π½Π° Π²ΠΎΠ΄Π΅. ΠŸΠΎΡΡ‚ΠΎΠΌΡƒ опишСм ΠΏΠΎΠ΄Ρ€ΠΎΠ±Π½Π΅Π΅ смысл рисунка. ΠšΠΎΠ½ΠΊΡ€Π΅Ρ‚Π½ΠΎ, ΠΏΡ€Π΅Π΄ΠΏΠΎΠ»ΠΎΠΆΠΈΠΌ, Ρ‡Ρ‚ΠΎ рисунок описываСт распространСниС Π²ΠΎΠ»Π½Ρ‹ синСго свСта, Π΄Π»ΠΈΠ½Π° Π²ΠΎΠ»Π½Ρ‹ ΠΊΠΎΡ‚ΠΎΡ€ΠΎΠΉ Ρ€Π°Π²Π½Π° 4.
10–5 cΠΌ.

Рис. 9-5. ЭлСктромагнитная Π²ΠΎΠ»Π½Π° синСго свСта, Ρ€Π°ΡΠΏΡ€ΠΎΡΡ‚Ρ€Π°Π½ΡΡŽΡ‰Π°ΡΡΡ Π² Π½Π°ΠΏΡ€Π°Π²Π»Π΅Π½ΠΈΠΈ Π²ΠΎΠ»Π½ΠΎΠ²ΠΎΠ³ΠΎ Π²Π΅ΠΊΡ‚ΠΎΡ€Π° k. Π’ΠΎ Π²Ρ€Π΅Π·ΠΊΠ΅ ΠΏΠΎΠΊΠ°Π·Π°Π½Ρ‹ Π²Π·Π°ΠΈΠΌΠ½ΠΎ пСрпСндикулярныС направлСния Ρ‚Ρ€Ρ‘Ρ… Π²Π΅ΠΊΡ‚ΠΎΡ€ΠΎΠ² – k, E, B

Π‘ Ρ‚Π΅Ρ‡Π΅Π½ΠΈΠ΅ΠΌ Π²Ρ€Π΅ΠΌΠ΅Π½ΠΈ Π²ΠΎΠ»Π½Π° распространяСтся Π² Π½Π°ΠΏΡ€Π°Π²Π»Π΅Π½ΠΈΠΈ Π²Π΅ΠΊΡ‚ΠΎΡ€Π° k со ΡΠΊΠΎΡ€ΠΎΡΡ‚ΡŒΡŽ свСта. Рассмотрим Ρ‚ΠΎΡ‡ΠΊΡƒ Π , ΠΌΠΈΠΌΠΎ ΠΊΠΎΡ‚ΠΎΡ€ΠΎΠΉ ΠΎΠ½Π° ΠΏΡ€ΠΎΡ…ΠΎΠ΄ΠΈΡ‚. Π­Ρ‚Π° Π²ΠΎΠ»Π½Π° состоит ΠΈΠ· ΠΊΠΎΠ»Π΅Π±Π»ΡŽΡ‰ΠΈΡ…ΡΡ элСктричСского ΠΈ ΠΌΠ°Π³Π½ΠΈΡ‚Π½ΠΎΠ³ΠΎ ΠΏΠΎΠ»Π΅ΠΉ, ΠΈ, Ρ‡Ρ‚ΠΎΠ±Ρ‹ ΠΏΠΎΡ‡ΡƒΠ²ΡΡ‚Π²ΠΎΠ²Π°Ρ‚ΡŒ ΠΈΡ…, Π½ΡƒΠΆΠ½ΠΎ ΠΏΠΎΠΌΠ΅ΡΡ‚ΠΈΡ‚ΡŒ Π² Ρ‚ΠΎΡ‡ΠΊΡƒ Π  подходящСС ΠΏΡ€ΠΎΠ±Π½ΠΎΠ΅ Ρ‚Π΅Π»ΠΎ. Π§Ρ‚ΠΎΠ±Ρ‹ ΡƒΠ²ΠΈΠ΄Π΅Ρ‚ΡŒ, Ρ‡Ρ‚ΠΎ происходит с элСктричСским ΠΏΠΎΠ»Π΅ΠΌ, прСдставим, Ρ‡Ρ‚ΠΎ Π² Ρ‚ΠΎΡ‡ΠΊΡƒ Π  ΠΏΠΎΠΌΠ΅Ρ‰Ρ‘Π½ элСктрон. Он чувствуСт ΠΊΠΎΠ»Π΅Π±Π»ΡŽΡ‰Π΅Π΅ΡΡ элСктричСскоС ΠΏΠΎΠ»Π΅ ΠΏΡ€ΠΎΠ»Π΅Ρ‚Π°ΡŽΡ‰Π΅ΠΉ ΠΌΠΈΠΌΠΎ Π²ΠΎΠ»Π½Ρ‹. На Π½Π΅Π³ΠΎ дСйствуСт сила, которая достигаСт максимума Π² Ρ‚ΠΎΡ‚ ΠΌΠΎΠΌΠ΅Π½Ρ‚, ΠΊΠΎΠ³Π΄Π° Ρ‚ΠΎΡ‡ΠΊΡƒ Π  ΠΏΡ€ΠΎΡ…ΠΎΠ΄ΠΈΡ‚ Π³ΠΎΡ€Π± Π²ΠΎΠ»Π½Ρ‹ (Ρ‚ΠΎΡ‡ΠΊΠ° Π‘ Π½Π° рисункС).

Π—Π°Ρ‚Π΅ΠΌ сила ΡƒΠΌΠ΅Π½ΡŒΡˆΠ°Π΅Ρ‚ΡΡ Π΄ΠΎ нуля ΠΈ достигаСт максимума Π² ΠΏΡ€ΠΎΡ‚ΠΈΠ²ΠΎΠΏΠΎΠ»ΠΎΠΆΠ½ΠΎΠΌ Π½Π°ΠΏΡ€Π°Π²Π»Π΅Π½ΠΈΠΈ, ΠΊΠΎΠ³Π΄Π° Ρ‚ΠΎΡ‡ΠΊΡƒ Π  ΠΏΡ€ΠΎΡ…ΠΎΠ΄ΠΈΡ‚ Ρ‚ΠΎΡ‡ΠΊΠ° D Π²ΠΎΠ»Π½Ρ‹, ΠΈ Ρ‚.Π΄. Π’Π°ΠΊΠΈΠΌ ΠΎΠ±Ρ€Π°Π·ΠΎΠΌ, элСктрон испытываСт воздСйствиС ΠΎΡΡ†ΠΈΠ»Π»ΠΈΡ€ΡƒΡŽΡ‰Π΅ΠΉ силы, которая колСблСтся с частотой 
f
Β  Π²ΠΎΠ»Π½Ρ‹. Π£ элСктрона имССтся ΠΌΠ°Π³Π½ΠΈΡ‚Π½Ρ‹ΠΉ ΠΌΠΎΠΌΠ΅Π½Ρ‚, поэтому ΠΎΠ½ взаимодСйствуСт Ρ‚Π°ΠΊΠΆΠ΅ ΠΈ с ΠΌΠ°Π³Π½ΠΈΡ‚Π½Ρ‹ΠΌ ΠΏΠΎΠ»Π΅ΠΌ Π²ΠΎΠ»Π½Ρ‹. Π‘Π»Π΅Π΄ΠΎΠ²Π°Ρ‚Π΅Π»ΡŒΠ½ΠΎ, Π½Π° элСктрон дСйствуСт сила, стрСмящаяся Ρ€Π°Π·Π²Π΅Ρ€Π½ΡƒΡ‚ΡŒ Π΅Π³ΠΎ вдоль ΠΌΠ°Π³Π½ΠΈΡ‚Π½ΠΎΠ³ΠΎ поля ΠΈ ΠΊΠΎΠ»Π΅Π±Π»ΡŽΡ‰Π°ΡΡΡ с Ρ‚ΠΎΠΉ ΠΆΠ΅ частотой  f.

Из рис. 9-5 Π²ΠΈΠ΄Π½ΠΎ, Ρ‡Ρ‚ΠΎ ΠΎΡΡ†ΠΈΠ»Π»ΠΈΡ€ΡƒΡŽΡ‰Π΅Π΅ элСктричСскоС ΠΏΠΎΠ»Π΅ всё врСмя находится Π² фиксированной плоскости. Π­Ρ‚ΠΎ ΠΆΠ΅ Π²Π΅Ρ€Π½ΠΎ ΠΈ для ΠΌΠ°Π³Π½ΠΈΡ‚Π½ΠΎΠ³ΠΎ поля с Ρ‚ΠΎΠΉ Ρ€Π°Π·Π½ΠΈΡ†Π΅ΠΉ, Ρ‡Ρ‚ΠΎ ΠΏΠ»ΠΎΡΠΊΠΎΡΡ‚ΡŒ, Π² ΠΊΠΎΡ‚ΠΎΡ€ΠΎΠΉ колСблСтся ΠΌΠ°Π³Π½ΠΈΡ‚Π½ΠΎΠ΅ ΠΏΠΎΠ»Π΅, пСрпСндикулярна плоскости, Π² ΠΊΠΎΡ‚ΠΎΡ€ΠΎΠΉ колСблСтся элСктричСскоС ΠΏΠΎΠ»Π΅. Π­Ρ‚ΠΎ Π²Π΅Ρ€Π½ΠΎ для всСх элСктромагнитных Π²ΠΎΠ»Π½. Если ΠΏΠΎΡΠΌΠΎΡ‚Ρ€Π΅Ρ‚ΡŒ Π½Π° ΠΏΡƒΡ‡ΠΎΠΊ Π²ΠΎΠ»Π½, ΠΎΠ±Ρ€Π°Π·ΡƒΡŽΡ‰ΠΈΡ… свСтовой Π»ΡƒΡ‡ ΠΎΡ‚ Π‘ΠΎΠ»Π½Ρ†Π° ΠΈΠ»ΠΈ ΠΎΡ‚ Π»Π°ΠΌΠΏΡ‹, Ρ‚ΠΎ обнаруТиваСтся, Ρ‡Ρ‚ΠΎ эти Π΄Π²Π΅ Π²Π·Π°ΠΈΠΌΠ½ΠΎ пСрпСндикулярныС плоскости ΠΎΡ€ΠΈΠ΅Π½Ρ‚ΠΈΡ€ΠΎΠ²Π°Π½Ρ‹ Π² Ρ€Π°Π·Π½Ρ‹Ρ… Π²ΠΎΠ»Π½Π°Ρ… ΠΏΠΎ-Ρ€Π°Π·Π½ΠΎΠΌΡƒ.

Π‘ΠΎΠ²ΠΎΠΊΡƒΠΏΠ½ΠΎΡΡ‚ΡŒ Π½Π°ΠΏΡ€Π°Π²Π»Π΅Π½ΠΈΠΉ, Π½Π°ΠΏΡ€ΠΈΠΌΠ΅Ρ€, элСктричСского поля Π² ΠΏΡƒΡ‡ΠΊΠ΅ Π²ΠΎΠ»Π½ Π½Π°ΠΏΠΎΠΌΠΈΠ½Π°Π΅Ρ‚ Π½Π°ΠΏΡ€Π°Π²Π»Π΅Π½Π½Ρ‹Π΅ Π²ΠΎ всС стороны ΠΈΠ³ΠΎΠ»ΠΊΠΈ Π΅ΠΆΠ°. Π’Π°ΠΊΠΎΠΉ свСт называСтся нСполяризованным. Если ΠΏΡ€ΠΎΠΏΡƒΡΡ‚ΠΈΡ‚ΡŒ свСт Ρ‡Π΅Ρ€Π΅Π· ΠΎΠΏΡ€Π΅Π΄Π΅Π»Ρ‘Π½Π½Ρ‹Π΅ вСщСства, Π½Π°Π·Ρ‹Π²Π°Π΅ΠΌΡ‹Π΅ поляризаторами, Ρ‚ΠΎ ΠΌΠΎΠΆΠ½ΠΎ ΠΎΡ‚Ρ„ΠΈΠ»ΡŒΡ‚Ρ€ΠΎΠ²Π°Ρ‚ΡŒ всС Π²ΠΎΠ»Π½Ρ‹ Π·Π° ΠΈΡΠΊΠ»ΡŽΡ‡Π΅Π½ΠΈΠ΅ΠΌ Ρ‚Π΅Ρ…, Ρƒ ΠΊΠΎΡ‚ΠΎΡ€Ρ‹Ρ… элСктричСскоС ΠΏΠΎΠ»Π΅ (ΠΈ соотвСтствСнно ΠΌΠ°Π³Π½ΠΈΡ‚Π½ΠΎΠ΅ ΠΏΠΎΠ»Π΅) располоТСно Π² ΠΎΠ΄Π½ΠΎΠΉ-СдинствСнной плоскости. ΠŸΠΎΠ»ΡƒΡ‡ΠΈΠ²ΡˆΠΈΠΉΡΡ свСт Π½Π°Π·Ρ‹Π²Π°ΡŽΡ‚ поляризованным. Π’ΠΎ вСщСство, ΠΊΠΎΡ‚ΠΎΡ€ΠΎΠ΅ часто ΠΈΡΠΏΠΎΠ»ΡŒΠ·ΡƒΡŽΡ‚ Π² солнСчных ΠΎΡ‡ΠΊΠ°Ρ…, состоит ΠΈΠ· слоёв ΠΏΠ°Ρ€Π°Π»Π»Π΅Π»ΡŒΠ½ΠΎ располоТСнных Π΄Π»ΠΈΠ½Π½Ρ‹Ρ… ΠΌΠΎΠ»Π΅ΠΊΡƒΠ». ЭлСктронная структура этих ΠΌΠΎΠ»Π΅ΠΊΡƒΠ» Ρ‚Π°ΠΊΠΎΠ²Π°, Ρ‡Ρ‚ΠΎ ΠΎΠ½ΠΈ ΠΏΠΎΠ³Π»ΠΎΡ‰Π°ΡŽΡ‚ Π²ΠΎΠ»Π½Ρ‹, элСктричСскоС ΠΏΠΎΠ»Π΅ ΠΊΠΎΡ‚ΠΎΡ€Ρ‹Ρ… ΠΏΠ°Ρ€Π°Π»Π»Π΅Π»ΡŒΠ½ΠΎ Π½Π°ΠΏΡ€Π°Π²Π»Π΅Π½ΠΈΡŽ вытянутости ΠΌΠΎΠ»Π΅ΠΊΡƒΠ», ΠΈ ΠΏΡ€ΠΎΠΏΡƒΡΠΊΠ°ΡŽΡ‚ Π²ΠΎΠ»Π½Ρ‹, Ρƒ ΠΊΠΎΡ‚ΠΎΡ€Ρ‹Ρ… элСктричСскоС ΠΏΠΎΠ»Π΅ пСрпСндикулярно этому Π½Π°ΠΏΡ€Π°Π²Π»Π΅Π½ΠΈΡŽ. Π‘ΠΎΠ»Π½Π΅Ρ‡Π½Ρ‹ΠΉ свСт, доходящий Π΄ΠΎ нас, подвСргаСтся ΠΏΠΎ ΠΏΡƒΡ‚ΠΈ Ρ€Π°ΡΡΠ΅ΡΠ½ΠΈΡŽ Π½Π° ΠΌΠΎΠ»Π΅ΠΊΡƒΠ»Π°Ρ… атмосфСры, Π² Ρ€Π΅Π·ΡƒΠ»ΡŒΡ‚Π°Ρ‚Π΅ Ρ‡Π΅Π³ΠΎ ΠΎΠ½ частично поляризуСтся.
Π’Ρ‹ ΠΌΠΎΠΆΠ΅Ρ‚Π΅ ΠΏΡ€ΠΎΠ²Π΅Ρ€ΠΈΡ‚ΡŒ это, посмотрСв Π½Π° Π½Π΅Π±ΠΎ сквозь ΠΏΠΎΠ»ΡΡ€ΠΈΠ·ΡƒΡŽΡ‰ΠΈΠ΅ солнСчныС ΠΎΡ‡ΠΊΠΈ ΠΈ Π·Π°ΠΌΠ΅Ρ‚ΠΈΠ² ΠΈΠ·ΠΌΠ΅Π½Π΅Π½ΠΈΠ΅ яркости ΠΏΡ€ΠΈ ΠΏΠΎΠ²ΠΎΡ€ΠΎΡ‚Π΅ ΠΎΡ‡ΠΊΠΎΠ².

ΠŸΡ€Π΅Π΄ΡΡ‚Π°Π²ΠΈΠ² элСктромагнитныС Π²ΠΎΠ»Π½Ρ‹ ΠΊΠ°ΠΊ ΡΠΎΠ²ΠΎΠΊΡƒΠΏΠ½ΠΎΡΡ‚ΡŒ взаимосвязанных ΠΊΠΎΠ»Π΅Π±Π»ΡŽΡ‰ΠΈΡ…ΡΡ элСктричСского ΠΈ ΠΌΠ°Π³Π½ΠΈΡ‚Π½ΠΎΠ³ΠΎ ΠΏΠΎΠ»Π΅ΠΉ, ΠΌΡ‹ достигли Π·Π½Π°Ρ‡ΠΈΡ‚Π΅Π»ΡŒΠ½ΠΎΠ³ΠΎ упрощСния. ΠšΠΎΠ½Π΅Ρ‡Π½ΠΎ, Ρ‡Ρ‚ΠΎΠ±Ρ‹ Π²ΠΎΠ»Π½Π° Π²ΠΎΠ·Π½ΠΈΠΊΠ»Π°, Π½ΡƒΠΆΠ΅Π½ ΠΎΠΏΡ€Π΅Π΄Π΅Π»Ρ‘Π½Π½Ρ‹ΠΉ источник – Π»Π°ΠΌΠΏΠΎΡ‡ΠΊΠ° для Π²ΠΈΠ΄ΠΈΠΌΠΎΠ³ΠΎ свСта, Ρ€Π°Π΄ΠΈΠΎΠ°ΠΊΡ‚ΠΈΠ²Π½ΠΎΠ΅ ядро для Π³Π°ΠΌΠΌΠ°-излучСния ΠΈ Ρ‚.ΠΏ. Нам Ρ‚Π΅ΠΏΠ΅Ρ€ΡŒ Π½Π΅ Π½ΡƒΠΆΠ½ΠΎ Π΄ΡƒΠΌΠ°Ρ‚ΡŒ ΠΎ слоТных процСссах, происходящих Π² источникС ΠΈ приводящих ΠΊ ΠΎΠ±Ρ€Π°Π·ΠΎΠ²Π°Π½ΠΈΡŽ Π²ΠΎΠ»Π½, Ρ‡Ρ‚ΠΎΠ±Ρ‹ ΠΏΠΎΠ½ΡΡ‚ΡŒ, Ρ‡Ρ‚ΠΎ происходит ΠΏΡ€ΠΈ ΠΏΠ°Π΄Π΅Π½ΠΈΠΈ Π²ΠΎΠ»Π½Ρ‹ Π½Π° Ρ‚Π²Ρ‘Ρ€Π΄ΠΎΠ΅ Ρ‚Π΅Π»ΠΎ. Π’Π΅ΠΌ Π½Π΅ ΠΌΠ΅Π½Π΅Π΅ ΠΌΡ‹ ΠΊΠΎΡ€ΠΎΡ‚ΠΊΠΎ коснёмся Π² ΡΠ»Π΅Π΄ΡƒΡŽΡ‰Π΅ΠΌ Ρ€Π°Π·Π΄Π΅Π»Π΅ Π΄Π²ΡƒΡ… способов Π³Π΅Π½Π΅Ρ€Π°Ρ†ΠΈΠΈ элСктромагнитных Π²ΠΎΠ»Π½ вСщСством, Ρ‚.Π΅. обсудим, ΠΊΠ°ΠΊ Π²ΠΎΠ»Π½Π° ΠΏΠΎΠΊΠΈΠ΄Π°Π΅Ρ‚ источник.

4. ГСнСрация свСта

ВсС Π½Π΅ΠΎΠ΄Π½ΠΎΠΊΡ€Π°Ρ‚Π½ΠΎ Π²ΠΈΠ΄Π΅Π»ΠΈ Π½Π΅ΠΎΠ½ΠΎΠ²Ρ‹Π΅ Ρ‚Ρ€ΡƒΠ±ΠΊΠΈ, ΠΈΡΠΏΡƒΡΠΊΠ°ΡŽΡ‰ΠΈΠ΅ красный свСт ΠΈ ΠΈΡΠΏΠΎΠ»ΡŒΠ·ΡƒΡŽΡ‰ΠΈΠ΅ΡΡ Π² Ρ€Π΅ΠΊΠ»Π°ΠΌΠ½Ρ‹Ρ… Ρ‰ΠΈΡ‚Π°Ρ…. Π’ Ρ‚Π°ΠΊΠΎΠΉ Ρ‚Ρ€ΡƒΠ±ΠΊΠ΅ содСрТится Π³Π°Π· Π½Π΅ΠΎΠ½, ΠΈ ΠΎΠ½Π° устроСна Ρ‚Π°ΠΊ, Ρ‡Ρ‚ΠΎ ΠΏΡ€ΠΈ ΠΏΡ€ΠΈΠ»ΠΎΠΆΠ΅Π½ΠΈΠΈ напряТСния ΠΌΠ΅ΠΆΠ΄Ρƒ Π΅Ρ‘ ΠΊΠΎΠ½Ρ†Π°ΠΌΠΈ Ρ‡Π΅Ρ€Π΅Π· Ρ‚Ρ€ΡƒΠ±ΠΊΡƒ Π½Π°Ρ‡ΠΈΠ½Π°Π΅Ρ‚ ΠΈΠ΄Ρ‚ΠΈ Ρ‚ΠΎΠΊ элСктронов. Π­Ρ‚ΠΈ элСктроны ΡΡ‚Π°Π»ΠΊΠΈΠ²Π°ΡŽΡ‚ΡΡ с Π°Ρ‚ΠΎΠΌΠ°ΠΌΠΈ Π½Π΅ΠΎΠ½Π° ΠΈ ΠΏΠ΅Ρ€Π΅Π΄Π°ΡŽΡ‚ ΠΈΠΌ Ρ‡Π°ΡΡ‚ΡŒ своСй кинСтичСской энСргии. Π’ Ρ€Π΅Π·ΡƒΠ»ΡŒΡ‚Π°Ρ‚Π΅ Π°Ρ‚ΠΎΠΌΡ‹ Π½Π΅ΠΎΠ½Π° пСрСходят Π² Π±ΠΎΠ»Π΅Π΅ высокоС ΠΊΠ²Π°Π½Ρ‚ΠΎΠ²ΠΎΠ΅ энСргСтичСскоС состояниС. Π—Π°Ρ‚Π΅ΠΌ Π°Ρ‚ΠΎΠΌΡ‹ ΡΠ±Ρ€Π°ΡΡ‹Π²Π°ΡŽΡ‚ лишнюю ΡΠ½Π΅Ρ€Π³ΠΈΡŽ, испуская Ρ„ΠΎΡ‚ΠΎΠ½ ΠΈ Π²ΠΎΠ·Π²Ρ€Π°Ρ‰Π°ΡΡΡŒ Π² исходноС Π½Π΅Π²ΠΎΠ·Π±ΡƒΠΆΠ΄Ρ‘Π½Π½ΠΎΠ΅ состояниС, послС Ρ‡Π΅Π³ΠΎ процСсс повторяСтся. Π’Π°ΠΊΠΈΠΌ ΠΎΠ±Ρ€Π°Π·ΠΎΠΌ, Ρ„ΠΎΡ‚ΠΎΠ½Ρ‹ ΠΌΠΎΠ³ΡƒΡ‚ ΠΈΠΌΠ΅Ρ‚ΡŒ Ρ‚ΠΎΠ»ΡŒΠΊΠΎ Ρ‚Π΅ энСргии (ΠΈ ΡΠΎΠΎΡ‚Π²Π΅Ρ‚ΡΡ‚Π²ΡƒΡŽΡ‰ΠΈΠ΅ частоты), ΠΊΠΎΡ‚ΠΎΡ€Ρ‹Π΅ ΡΠ²Π»ΡΡŽΡ‚ΡΡ разностями энСргий Π²ΠΎΠ·Π±ΡƒΠΆΠ΄Ρ‘Π½Π½Ρ‹Ρ… энСргСтичСских состояний Π°Ρ‚ΠΎΠΌΠ° ΠΈ основного состояния. Π’ случаС Π½Π΅ΠΎΠ½Π° Π±ΠΎΠ»ΡŒΡˆΠΈΠ½ΡΡ‚Π²ΠΎ Ρ„ΠΎΡ‚ΠΎΠ½ΠΎΠ² ΠΈΠ·Π»ΡƒΡ‡Π°ΡŽΡ‚ΡΡ Π² Ρ€Π΅Π·ΡƒΠ»ΡŒΡ‚Π°Ρ‚Π΅ ΠΏΠ΅Ρ€Π΅Ρ…ΠΎΠ΄Π° ΠΈΠ· состояния, энСргия ΠΊΠΎΡ‚ΠΎΡ€ΠΎΠ³ΠΎ отличаСтся ΠΎΡ‚ энСргии основного состояния Π½Π΅ΠΎΠ½Π° Π½Π° Π²Π΅Π»ΠΈΡ‡ΠΈΠ½Ρƒ, ΡΠΎΠΎΡ‚Π²Π΅Ρ‚ΡΡ‚Π²ΡƒΡŽΡ‰ΡƒΡŽ энСргии Ρ„ΠΎΡ‚ΠΎΠ½ΠΎΠ² красного свСта. Π’ Π΄Ρ€ΡƒΠ³ΠΈΡ… Π³Π°Π·Π°Ρ… Π΄ΠΎΠΌΠΈΠ½ΠΈΡ€ΡƒΡŽΡ‚ Ρ„ΠΎΡ‚ΠΎΠ½Ρ‹ Π΄Ρ€ΡƒΠ³ΠΈΡ… Ρ†Π²Π΅Ρ‚ΠΎΠ².

Π₯Π°Ρ€Π°ΠΊΡ‚Π΅Ρ€Π½Ρ‹ΠΌ свойством ΠΏΠΎΠ΄ΠΎΠ±Π½ΠΎΠ³ΠΎ свСта являСтся Ρ‚ΠΎ, Ρ‡Ρ‚ΠΎ ΠΎΠ½ состоит ΠΈΠ· Π²ΠΎΠ»Π½ ΠΎΠΏΡ€Π΅Π΄Π΅Π»Ρ‘Π½Π½Ρ‹Ρ… частот, фиксированных ΠΊΠ²Π°Π½Ρ‚ΠΎΠ²Ρ‹ΠΌΠΈ уровнями Π°Ρ‚ΠΎΠΌΠΎΠ².

Но свСт излучаСтся ΠΈ сильно Π½Π°Π³Ρ€Π΅Ρ‚Ρ‹ΠΌ Ρ‚Π΅Π»ΠΎΠΌ. ΠŸΡ€ΠΈΠΌΠ΅Ρ€Π°ΠΌΠΈ ΠΌΠΎΠ³ΡƒΡ‚ ΡΠ»ΡƒΠΆΠΈΡ‚ΡŒ Π»Π°ΠΌΠΏΠ° накаливания, Π² ΠΊΠΎΡ‚ΠΎΡ€ΠΎΠΉ тонкая Π²ΠΎΠ»ΡŒΡ„Ρ€Π°ΠΌΠΎΠ²Π°Ρ Π½ΠΈΡ‚ΡŒ раскаляСтся Π² Ρ€Π΅Π·ΡƒΠ»ΡŒΡ‚Π°Ρ‚Π΅ прохоТдСния ΠΏΠΎ Π½Π΅ΠΉ Ρ‚ΠΎΠΊΠ°, ΠΈΠ»ΠΈ ΠΏΠΎΠ²Π΅Ρ€Ρ…Π½ΠΎΡΡ‚ΡŒ Π‘ΠΎΠ»Π½Ρ†Π°, Ρ‚Π΅ΠΌΠΏΠ΅Ρ€Π°Ρ‚ΡƒΡ€Π° ΠΊΠΎΡ‚ΠΎΡ€ΠΎΠΉ поддСрТиваСтся происходящими Π²Π½ΡƒΡ‚Ρ€ΠΈ тСрмоядСрными процСссами. Если ΠΏΡ€ΠΎΠΏΡƒΡΡ‚ΠΈΡ‚ΡŒ Ρ‚Π°ΠΊΠΎΠΉ свСт Ρ‡Π΅Ρ€Π΅Π· ΡΡ‚Π΅ΠΊΠ»ΡΠ½Π½ΡƒΡŽ ΠΏΡ€ΠΈΠ·ΠΌΡƒ, ΠΏΠΎ-Ρ€Π°Π·Π½ΠΎΠΌΡƒ ΠΎΡ‚ΠΊΠ»ΠΎΠ½ΡΡŽΡ‰ΡƒΡŽ свСт Ρ€Π°Π·Π½Ρ‹Ρ… частот, получится Π½Π΅ΠΏΡ€Π΅Ρ€Ρ‹Π²Π½ΠΎΠ΅ распрСдСлСниС Ρ†Π²Π΅Ρ‚ΠΎΠ² ΠΎΡ‚ Ρ„ΠΈΠΎΠ»Π΅Ρ‚ΠΎΠ²ΠΎΠ³ΠΎ Π΄ΠΎ красного. Π‘ ΠΏΠΎΠΌΠΎΡ‰ΡŒΡŽ ΡΠΎΠΎΡ‚Π²Π΅Ρ‚ΡΡ‚Π²ΡƒΡŽΡ‰ΠΈΡ… ΠΏΡ€ΠΈΠ±ΠΎΡ€ΠΎΠ² ΠΌΠΎΠΆΠ½ΠΎ ΡƒΡΡ‚Π°Π½ΠΎΠ²ΠΈΡ‚ΡŒ, Ρ‡Ρ‚ΠΎ свСт содСрТит Ρ‚Π°ΠΊΠΆΠ΅ ΠΊΠΎΠΌΠΏΠΎΠ½Π΅Π½Ρ‚Ρ‹ с частотами, Π½Π΅Π²ΠΈΠ΄ΠΈΠΌΡ‹ΠΌΠΈ чСловСчСским Π³Π»Π°Π·ΠΎΠΌ, – инфракрасныС (частоты Π½ΠΈΠΆΠ΅, Ρ‡Π΅ΠΌ Ρƒ красного свСта) ΠΈ ΡƒΠ»ΡŒΡ‚Ρ€Π°Ρ„ΠΈΠΎΠ»Π΅Ρ‚ΠΎΠ²Ρ‹Π΅ (частоты Π²Ρ‹ΡˆΠ΅, Ρ‡Π΅ΠΌ Ρƒ Ρ„ΠΈΠΎΠ»Π΅Ρ‚ΠΎΠ²ΠΎΠ³ΠΎ свСта). На самом Π΄Π΅Π»Π΅ Π² свСтС Π»Π°ΠΌΠΏΠΎΡ‡ΠΊΠΈ ΠΈΠ»ΠΈ Π‘ΠΎΠ»Π½Ρ†Π° содСрТится ΠΏΠΎΠ»Π½Ρ‹ΠΉ спСктр элСктромагнитных Π²ΠΎΠ»Π½, хотя Π±ΠΎΠ»ΡŒΡˆΠΈΠ½ΡΡ‚Π²ΠΎ Ρ„ΠΎΡ‚ΠΎΠ½ΠΎΠ² солнСчного свСта ΠΏΡ€ΠΈΠ½Π°Π΄Π»Π΅ΠΆΠ°Ρ‚ ΠΈΠ½Ρ‚Π΅Ρ€Π²Π°Π»Ρƒ Π²ΠΈΠ΄ΠΈΠΌΠΎΠ³ΠΎ свСта, Π° Ρ‡Π΅ΠΌ дальшС ΠΎΡ‚ этого ΠΈΠ½Ρ‚Π΅Ρ€Π²Π°Π»Π°, Ρ‚Π΅ΠΌ число Ρ„ΠΎΡ‚ΠΎΠ½ΠΎΠ² ΡΠΎΠΎΡ‚Π²Π΅Ρ‚ΡΡ‚Π²ΡƒΡŽΡ‰ΠΈΡ… частот становится мСньшС.

ΠžΡ‚ΠΊΡƒΠ΄Π° ΠΆΠ΅ Π²ΠΎΠ·Π½ΠΈΠΊΠ°Π΅Ρ‚ Ρ‚Π°ΠΊΠΎΠΉ Π½Π΅ΠΏΡ€Π΅Ρ€Ρ‹Π²Π½Ρ‹ΠΉ спСктр частот? ΠœΡ‹ Π·Π½Π°Π΅ΠΌ, Ρ‡Ρ‚ΠΎ тСпловая энСргия Ρ‚Π΅Π»Π° – это энСргия двиТСния ΡΠΎΡΡ‚Π°Π²Π»ΡΡŽΡ‰ΠΈΡ… Ρ‚Π΅Π»ΠΎ частиц (Π°Ρ‚ΠΎΠΌΠΎΠ², ядСр ΠΈ Ρ‚.ΠΏ.). Π­Π½Π΅Ρ€Π³ΠΈΠΈ ΠΎΡ‚Π΄Π΅Π»ΡŒΠ½Ρ‹Ρ… частиц ΠΌΠ΅Π½ΡΡŽΡ‚ΡΡ ΠΎΡ‚ нуля Π΄ΠΎ ΠΎΡ‡Π΅Π½ΡŒ Π±ΠΎΠ»ΡŒΡˆΠΈΡ… Π·Π½Π°Ρ‡Π΅Π½ΠΈΠΉ, Π° распрСдСлСниС частиц ΠΏΠΎ Ρ€Π°Π·Π½Ρ‹ΠΌ ΠΈΠ½Ρ‚Π΅Ρ€Π²Π°Π»Π°ΠΌ энСргии зависит ΠΎΡ‚ Ρ‚Π΅ΠΌΠΏΠ΅Ρ€Π°Ρ‚ΡƒΡ€Ρ‹ Ρ‚Π΅Π»Π°. Всё это ΠΎΠ±ΡΡƒΠΆΠ΄Π°Π»ΠΎΡΡŒ Π² Π³Π». 6. ИмСнно это ΠΎΡ‡Π΅Π½ΡŒ слоТноС Ρ‚Π΅ΠΏΠ»ΠΎΠ²ΠΎΠ΅ Π΄Π²ΠΈΠΆΠ΅Π½ΠΈΠ΅ элСктричСских зарядов, ΠΊΠΎΠ½ΠΊΡ€Π΅Ρ‚Π½ΠΎ элСктронов ΠΈ ядСр, ΠΏΠΎΡ€ΠΎΠΆΠ΄Π°Π΅Ρ‚ Π½Π΅ΠΏΡ€Π΅Ρ€Ρ‹Π²Π½Ρ‹ΠΉ спСктр элСктромагнитного излучСния, ΠΊΠΎΡ‚ΠΎΡ€ΠΎΠ΅ называСтся поэтому Ρ‚Π΅ΠΏΠ»ΠΎΠ²Ρ‹ΠΌ ΠΈΠ·Π»ΡƒΡ‡Π΅Π½ΠΈΠ΅ΠΌ. ΠœΡ‹ Π·Π½Π°Π΅ΠΌ, Ρ‡Ρ‚ΠΎ Ссли Ρ‚Π΅ΠΌΠΏΠ΅Ρ€Π°Ρ‚ΡƒΡ€Π° Ρ‚Π΅Π»Π° Ρ€Π°Π²Π½Π° Π’Β  кСльвинов, Ρ‚ΠΎ срСдняя энСргия Ρ‚Π΅ΠΏΠ»ΠΎΠ²ΠΎΠ³ΠΎ двиТСния, приходящаяся Π½Π° ΠΎΠ΄Π½Ρƒ частицу, составляСт ΠΎΠΊΠΎΠ»ΠΎ kT, Π³Π΄Π΅ k – постоянная Π‘ΠΎΠ»ΡŒΡ†ΠΌΠ°Π½Π°. Π‘ΡƒΠ΄Π΅ΠΌ Π½Π°Π·Ρ‹Π²Π°Ρ‚ΡŒ эту Π²Π΅Π»ΠΈΡ‡ΠΈΠ½Ρƒ Ρ‚Π΅ΠΏΠ»ΠΎΠ²ΠΎΠΉ энСргиСй, приходящСйся Π½Π° ΠΎΠ΄Π½Ρƒ частицу. Если Π²Ρ‹Ρ€Π°Π·ΠΈΡ‚ΡŒ ΡΠ½Π΅Ρ€Π³ΠΈΡŽ Ρ„ΠΎΡ‚ΠΎΠ½ΠΎΠ² Π² долях Ρ‚Π΅ΠΏΠ»ΠΎΠ²ΠΎΠΉ энСргии, приходящСйся Π½Π° ΠΎΠ΄Π½Ρƒ частицу, ΠΈ ΠΏΠΎΡΡ‚Ρ€ΠΎΠΈΡ‚ΡŒ распрСдСлСниС Ρ„ΠΎΡ‚ΠΎΠ½ΠΎΠ² Ρ‚Π΅ΠΏΠ»ΠΎΠ²ΠΎΠ³ΠΎ излучСния Π² Ρ€Π°Π·Π½Ρ‹Ρ… Π΄ΠΈΠ°ΠΏΠ°Π·ΠΎΠ½Π°Ρ… энСргии, Ρ‚ΠΎ получится ΠΊΠ°Ρ€Ρ‚ΠΈΠ½Π°, показанная Π½Π° рис. 9-6. Π€ΠΎΡ€ΠΌΠ° этого распрСдСлСния обусловлСна Ρ‚Π΅ΠΌ, Ρ‡Ρ‚ΠΎ Ρ„ΠΎΡ‚ΠΎΠ½Ρ‹ ΡΠ²Π»ΡΡŽΡ‚ΡΡ Π±ΠΎΠ·ΠΎΠ½Π°ΠΌΠΈ, ΠΈ остаётся Π½Π΅ΠΈΠ·ΠΌΠ΅Π½Π½ΠΎΠΉ ΠΏΡ€ΠΈ любой Ρ‚Π΅ΠΌΠΏΠ΅Ρ€Π°Ρ‚ΡƒΡ€Π΅ Ρ‚Π΅Π»Π°, Π±ΡƒΠ΄ΡŒ Ρ‚ΠΎ 3000 К Ρƒ Π»Π°ΠΌΠΏΠΎΡ‡ΠΊΠΈ накаливания, 6000 К Ρƒ повСрхности Π‘ΠΎΠ»Π½Ρ†Π° ΠΈ Ρ‚.Π΄. Как Π²ΠΈΠ΄Π½ΠΎ, наибольшСС число Ρ„ΠΎΡ‚ΠΎΠ½ΠΎΠ² ΠΈΠΌΠ΅ΡŽΡ‚ энСргии, ΠΏΡ€ΠΈΠΌΠ΅Ρ€Π½ΠΎ Π² Ρ‚Ρ€ΠΈ Ρ€Π°Π·Π° большиС Ρ‚Π΅ΠΏΠ»ΠΎΠ²ΠΎΠΉ энСргии. Π’Π°ΠΊ ΠΊΠ°ΠΊ частота Ρ„ΠΎΡ‚ΠΎΠ½ΠΎΠ² ΠΏΡ€ΠΎΠΏΠΎΡ€Ρ†ΠΈΠΎΠ½Π°Π»ΡŒΠ½Π° ΠΈΡ… энСргии, это ΠΎΠ·Π½Π°Ρ‡Π°Π΅Ρ‚, Π½Π°ΠΏΡ€ΠΈΠΌΠ΅Ρ€, Ρ‡Ρ‚ΠΎ наибольшСС число Ρ„ΠΎΡ‚ΠΎΠ½ΠΎΠ² солнСчного свСта приходится Π½Π° частоту, Π²Π΄Π²ΠΎΠ΅ Π±ΠΎΠ»ΡŒΡˆΡƒΡŽ, Ρ‡Π΅ΠΌ частота свСта ΠΎΡ‚ элСктричСской Π»Π°ΠΌΠΏΠΎΡ‡ΠΊΠΈ. Если ΠΏΠΎΠ΄ΡΡ‚Π°Π²ΠΈΡ‚ΡŒ Ρ€Π΅Π°Π»ΡŒΠ½Ρ‹Π΅ Ρ†ΠΈΡ„Ρ€Ρ‹, Ρ‚ΠΎ получится, Ρ‡Ρ‚ΠΎ Π±ΠΎΠ»ΡŒΡˆΠΈΠ½ΡΡ‚Π²ΠΎ Ρ„ΠΎΡ‚ΠΎΠ½ΠΎΠ² ΠΎΡ‚ Π‘ΠΎΠ»Π½Ρ†Π° относятся ΠΊ красной части спСктра (Π΄Π»ΠΈΠ½Π° Π²ΠΎΠ»Π½Ρ‹ ΠΏΡ€ΠΈΠΌΠ΅Ρ€Π½ΠΎ Ρ€Π°Π²Π½Π° 8.10–5 см), Π° Π±ΠΎΠ»ΡŒΡˆΠΈΠ½ΡΡ‚Π²ΠΎ Ρ„ΠΎΡ‚ΠΎΠ½ΠΎΠ² свСта элСктричСской Π»Π°ΠΌΠΏΠΎΡ‡ΠΊΠΈ относятся ΠΊ инфракрасной части спСктра (1,6.10–4 см).

Рис. 9-6. РаспрСдСлСниС энСргии Ρ‚Π΅ΠΏΠ»ΠΎΠ²ΠΎΠ³ΠΎ излучСния, ΠΏΠΎΠΊΠ°Π·Ρ‹Π²Π°ΡŽΡ‰Π΅Π΅ ΠΎΡ‚Π½ΠΎΡΠΈΡ‚Π΅Π»ΡŒΠ½ΠΎΠ΅ число Ρ„ΠΎΡ‚ΠΎΠ½ΠΎΠ² Π² Ρ€Π°Π·Π½Ρ‹Ρ… ΠΈΠ½Ρ‚Π΅Ρ€Π²Π°Π»Π°Ρ… энСргии ΠΏΡ€ΠΈ любой Ρ‚Π΅ΠΌΠΏΠ΅Ρ€Π°Ρ‚ΡƒΡ€Π΅ Π’. На Π³ΠΎΡ€ΠΈΠ·ΠΎΠ½Ρ‚Π°Π»ΡŒΠ½ΠΎΠΉ оси ΠΎΡ‚Π»ΠΎΠΆΠ΅Π½ΠΎ ΠΎΡ‚Π½ΠΎΡˆΠ΅Π½ΠΈΠ΅ энСргии Ρ„ΠΎΡ‚ΠΎΠ½Π° ΠΊ Ρ‚Π΅ΠΏΠ»ΠΎΠ²ΠΎΠΉ энСргии kT. НСсмотря Π½Π° Ρ‚ΠΎ, Ρ‡Ρ‚ΠΎ ΠΏΡ€ΠΈ любой Ρ‚Π΅ΠΌΠΏΠ΅Ρ€Π°Ρ‚ΡƒΡ€Π΅ Π½Π°Π³Ρ€Π΅Ρ‚ΠΎΠ³ΠΎ Ρ‚Π΅Π»Π° Ρ‚Π΅ΠΏΠ»ΠΎΠ²ΠΎΠ΅ ΠΈΠ·Π»ΡƒΡ‡Π΅Π½ΠΈΠ΅ содСрТит Ρ„ΠΎΡ‚ΠΎΠ½Ρ‹ всСх частот, Π±ΠΎΠ»ΡŒΡˆΠΈΠ½ΡΡ‚Π²ΠΎ ΠΈΠ· Π½ΠΈΡ… ΠΏΠΎΠΏΠ°Π΄Π°Π΅Ρ‚ Π² ΠΈΠ½Ρ‚Π΅Ρ€Π²Π°Π» энСргий Π² окрСстности значСния 3kT. Для Π‘ΠΎΠ»Π½Ρ†Π°, Ρ‚Π΅ΠΌΠΏΠ΅Ρ€Π°Ρ‚ΡƒΡ€Π° повСрхности ΠΊΠΎΡ‚ΠΎΡ€ΠΎΠ³ΠΎ Ρ€Π°Π²Π½Π° 6000 К, этот ΠΈΠ½Ρ‚Π΅Ρ€Π²Π°Π» соотвСтствуСт красной ΠΈ инфракрасной частям Π²ΠΈΠ΄ΠΈΠΌΠΎΠ³ΠΎ свСта

Вопрос. Читая этот тСкст, Π²Ρ‹, ΠΊΠ°ΠΊ ΠΈ ΠΎΠΊΡ€ΡƒΠΆΠ°ΡŽΡ‰ΠΈΠ΅ вас Ρ‚Π΅Π»Π°, Ρ‚Π°ΠΊΠΆΠ΅ испускаСтС Π½Π΅ΠΏΡ€Π΅Ρ€Ρ‹Π²Π½Ρ‹ΠΉ спСктр элСктромагнитного излучСния. Π§Π΅ΠΌΡƒ Ρ€Π°Π²Π½Π° частота Π±ΠΎΠ»ΡŒΡˆΠΈΠ½ΡΡ‚Π²Π° Ρ„ΠΎΡ‚ΠΎΠ½ΠΎΠ² этого свСта ΠΏΠΎ ΡΡ€Π°Π²Π½Π΅Π½ΠΈΡŽ с частотой Π±ΠΎΠ»ΡŒΡˆΠΈΠ½ΡΡ‚Π²Π° Ρ„ΠΎΡ‚ΠΎΠ½ΠΎΠ² свСта элСктричСской Π»Π°ΠΌΠΏΠΎΡ‡ΠΊΠΈ?

ΠžΡ‚Π²Π΅Ρ‚. Π’Π΅ΠΌΠΏΠ΅Ρ€Π°Ρ‚ΡƒΡ€Π° ΠΎΠΊΡ€ΡƒΠΆΠ°ΡŽΡ‰Π΅ΠΉ вас срСды Ρ€Π°Π²Π½Π° ΠΏΡ€ΠΈΠΌΠ΅Ρ€Π½ΠΎ 20 Β°Π‘, Ρ‚. Π΅. 297 К, Ρ‡Ρ‚ΠΎ составляСт ΠΏΡ€ΠΈΠΌΠ΅Ρ€Π½ΠΎ ΠΎΠ΄Π½Ρƒ Π΄Π΅ΡΡΡ‚ΡƒΡŽ Ρ‡Π°ΡΡ‚ΡŒ Ρ‚Π΅ΠΌΠΏΠ΅Ρ€Π°Ρ‚ΡƒΡ€Ρ‹ Π½ΠΈΡ‚ΠΈ накаливания Π² Π»Π°ΠΌΠΏΠΎΡ‡ΠΊΠ΅. ΠŸΠΎΡΡ‚ΠΎΠΌΡƒ Π±ΠΎΠ»ΡŒΡˆΠΈΠ½ΡΡ‚Π²ΠΎ Ρ„ΠΎΡ‚ΠΎΠ½ΠΎΠ² Ρ‚Π΅ΠΏΠ»ΠΎΠ²ΠΎΠ³ΠΎ излучСния ΠΎΠΊΡ€ΡƒΠΆΠ°ΡŽΡ‰Π΅ΠΉ срСды ΠΈΠΌΠ΅ΡŽΡ‚ ΡΠ½Π΅Ρ€Π³ΠΈΡŽ, Ρ€Π°Π²Π½ΡƒΡŽ ΠΎΠ΄Π½ΠΎΠΉ дСсятой энСргии Ρ„ΠΎΡ‚ΠΎΠ½ΠΎΠ² ΠΎΡ‚ Π»Π°ΠΌΠΏΠΎΡ‡ΠΊΠΈ. Π˜Π·Π»ΡƒΡ‡Π΅Π½ΠΈΠ΅ с Ρ‚Π°ΠΊΠΎΠΉ энСргиСй ΠΏΡ€ΠΈΠ½Π°Π΄Π»Π΅ΠΆΠΈΡ‚ Π΄Π°Π»Ρ‘ΠΊΠΎΠΉ части спСктра инфракрасного излучСния. ИмСнно это ΠΈΠ·Π»ΡƒΡ‡Π΅Π½ΠΈΠ΅ фиксируСтся ΠΏΡ€ΠΈΠ±ΠΎΡ€Π°ΠΌΠΈ Π½ΠΎΡ‡Π½ΠΎΠ³ΠΎ видСния.

Как Π²ΠΈΠ΄Π½ΠΎ ΠΈΠ· рис. 9-6, Ρ‡Π΅ΠΌ Π½ΠΈΠΆΠ΅ Ρ‚Π΅ΠΌΠΏΠ΅Ρ€Π°Ρ‚ΡƒΡ€Π°, Ρ‚Π΅ΠΌ мСньшС ΠΈΠ½Ρ‚Π΅Ρ€Π²Π°Π» энСргий Ρ„ΠΎΡ‚ΠΎΠ½ΠΎΠ², Π² ΠΊΠΎΡ‚ΠΎΡ€ΠΎΠΌ обнаруТиваСтся большая Ρ‡Π°ΡΡ‚ΡŒ всСх Ρ„ΠΎΡ‚ΠΎΠ½ΠΎΠ². ВсСлСнная Π·Π°ΠΏΠΎΠ»Π½Π΅Π½Π° элСктромагнитным ΠΈΠ·Π»ΡƒΡ‡Π΅Π½ΠΈΠ΅ΠΌ, ΡΠΎΠΎΡ‚Π²Π΅Ρ‚ΡΡ‚Π²ΡƒΡŽΡ‰ΠΈΠΌ Ρ‚Π΅ΠΌΠΏΠ΅Ρ€Π°Ρ‚ΡƒΡ€Π΅ ΠΏΡ€ΠΈΠΌΠ΅Ρ€Π½ΠΎ 3 К. Π­Ρ‚ΠΎ ΠΎΠ·Π½Π°Ρ‡Π°Π΅Ρ‚, Ρ‡Ρ‚ΠΎ большая Ρ‡Π°ΡΡ‚ΡŒ Ρ„ΠΎΡ‚ΠΎΠ½ΠΎΠ² этого излучСния ΠΈΠΌΠ΅Π΅Ρ‚ Π΄Π»ΠΈΠ½Ρƒ Π²ΠΎΠ»Π½Ρ‹ порядка 1 ΠΌΠΌ. Π­Ρ‚ΠΎ ΠΈΠ·Π»ΡƒΡ‡Π΅Π½ΠΈΠ΅ Π΅ΡΡ‚ΡŒ остаток Π‘ΠΎΠ»ΡŒΡˆΠΎΠ³ΠΎ Π²Π·Ρ€Ρ‹Π²Π°, ΠΏΠΎΡ€ΠΎΠ΄ΠΈΠ²ΡˆΠ΅Π³ΠΎ 10Β ΠΌΠ»Ρ€Π΄ Π»Π΅Ρ‚ Ρ‚ΠΎΠΌΡƒ Π½Π°Π·Π°Π΄ Π½Π°ΡˆΡƒ Π’ΡΠ΅Π»Π΅Π½Π½ΡƒΡŽ.

5. ΠŸΠΎΠ³Π»ΠΎΡ‰Π΅Π½ΠΈΠ΅ свСта

ВсС Ρ‚Π²Ρ‘Ρ€Π΄Ρ‹Π΅ Ρ‚Π΅Π»Π° состоят ΠΈΠ· частиц – элСктронов ΠΈ ядСр, – ΠΎΠ±Π»Π°Π΄Π°ΡŽΡ‰ΠΈΡ… двумя свойствами, ΠΊΠΎΡ‚ΠΎΡ€Ρ‹Π΅ ΠΏΠΎΠ·Π²ΠΎΠ»ΡΡŽΡ‚ ΠΈΠΌ ΠΈΡΠΏΡ‹Ρ‚Ρ‹Π²Π°Ρ‚ΡŒ влияниС элСктричСских ΠΈ ΠΌΠ°Π³Π½ΠΈΡ‚Π½Ρ‹Ρ… ΠΏΠΎΠ»Π΅ΠΉ Π² элСктромагнитной Π²ΠΎΠ»Π½Π΅: эти частицы заряТСны ΠΈ ΡΠ²Π»ΡΡŽΡ‚ΡΡ постоянными ΠΌΠ°Π³Π½ΠΈΡ‚Π°ΠΌΠΈ. ΠšΡ€ΠΎΠΌΠ΅ Ρ‚ΠΎΠ³ΠΎ, согласно Π·Π°ΠΊΠΎΠ½Π°ΠΌ ΠΊΠ²Π°Π½Ρ‚ΠΎΠ²ΠΎΠΉ ΠΌΠ΅Ρ…Π°Π½ΠΈΠΊΠΈ, энСргии элСктронов ΠΎΠΏΠΈΡΡ‹Π²Π°ΡŽΡ‚ΡΡ с ΠΏΠΎΠΌΠΎΡ‰ΡŒΡŽ понятия энСргСтичСских Π·ΠΎΠ½, Π° энСргия элСктромагнитных Π²ΠΎΠ»Π½ ΠΊΠ²Π°Π½Ρ‚ΠΎΠ²Π°Π½Π° ΠΈ Ρ€Π°Π²Π½Π° суммС энСргий Ρ„ΠΎΡ‚ΠΎΠ½ΠΎΠ². Π˜ΡΠΏΠΎΠ»ΡŒΠ·ΡƒΡ эту ΠΈΠ½Ρ„ΠΎΡ€ΠΌΠ°Ρ†ΠΈΡŽ, ΠΏΠΎΠΏΡ€ΠΎΠ±ΡƒΠ΅ΠΌ ΠΎΡ‚Π²Π΅Ρ‚ΠΈΡ‚ΡŒ Π½Π° Π½Π΅ΠΊΠΎΡ‚ΠΎΡ€Ρ‹Π΅ вопросы, Π½Π°ΠΏΡ€ΠΈΠΌΠ΅Ρ€, ΠΏΠΎΡ‡Π΅ΠΌΡƒ стСкло ΠΏΡ€ΠΎΠ·Ρ€Π°Ρ‡Π½ΠΎ для свСта, Π° алюминивая Ρ„ΠΎΠ»ΡŒΠ³Π° Π½Π΅ΠΏΡ€ΠΎΠ·Ρ€Π°Ρ‡Π½Π°, ΠΈΠ»ΠΈ ΠΏΠΎΡ‡Π΅ΠΌΡƒ мСдь красная, Π° сапфир синий.

Π’ этом Ρ€Π°Π·Π΄Π΅Π»Π΅ сосрСдоточимся Π½Π° рассмотрСнии Ρ‚Π΅Ρ… эффСктов, ΠΊΠΎΡ‚ΠΎΡ€Ρ‹Π΅ Π²ΠΎΠ·Π½ΠΈΠΊΠ°ΡŽΡ‚ ΠΏΡ€ΠΈ воздСйствии элСктричСского поля свСтовой Π²ΠΎΠ»Π½Ρ‹ Π½Π° ΠΎΠ±Ρ€Π°Π·ΡƒΡŽΡ‰ΠΈΠ΅ кусок вСщСства Π°Ρ‚ΠΎΠΌΡ‹. ЭлСктричСскоС ΠΏΠΎΠ»Π΅ дСйствуСт Π½Π° элСктричСскиС заряды, Π° ΠΈΠΌΠ΅Π½Π½ΠΎ, Π½Π° элСктроны ΠΈ ядра, входящиС Π² состав Π°Ρ‚ΠΎΠΌΠΎΠ². ПолС, Π΄Π΅ΠΉΡΡ‚Π²ΡƒΡŽΡ‰Π΅Π΅ Π½Π° ΠΎΡ‚Π΄Π΅Π»ΡŒΠ½Ρ‹ΠΉ Π°Ρ‚ΠΎΠΌ, сдвигаСт ядро (ΠΏΠΎΠ»ΠΎΠΆΠΈΡ‚Π΅Π»ΡŒΠ½Ρ‹ΠΉ заряд) Π² ΠΎΠ΄Π½ΠΎΠΌ Π½Π°ΠΏΡ€Π°Π²Π»Π΅Π½ΠΈΠΈ, Π° ΠΎΠΊΡ€ΡƒΠΆΠ°ΡŽΡ‰ΠΈΠ΅ ядро элСктроны (ΠΎΡ‚Ρ€ΠΈΡ†Π°Ρ‚Π΅Π»ΡŒΠ½Ρ‹ΠΉ заряд) – Π² ΠΏΡ€ΠΎΡ‚ΠΈΠ²ΠΎΠΏΠΎΠ»ΠΎΠΆΠ½ΠΎΠΌ Π½Π°ΠΏΡ€Π°Π²Π»Π΅Π½ΠΈΠΈ. Π­Ρ‚ΠΎ ΠΎΡ‚Π½ΠΎΡΠΈΡ‚Π΅Π»ΡŒΠ½ΠΎΠ΅ смСщСниС Π΄Π²ΡƒΡ… зарядов прСкращаСтся Π² Ρ‚ΠΎΡ‚ ΠΌΠΎΠΌΠ΅Π½Ρ‚, ΠΊΠΎΠ³Π΄Π° сила, Π΄Π΅ΠΉΡΡ‚Π²ΡƒΡŽΡ‰Π°Ρ со стороны элСктричСского поля, ΡƒΡ€Π°Π²Π½ΠΎΠ²Π΅ΡˆΠΈΠ²Π°Π΅Ρ‚ΡΡ силами притяТСния ΠΌΠ΅ΠΆΠ΄Ρƒ ядрами ΠΈ элСктронными ΠΎΠ±Π»Π°ΠΊΠ°ΠΌΠΈ Π² Π°Ρ‚ΠΎΠΌΠ°Ρ…. ΠœΡ‹ Π΅Ρ‰Ρ‘ вСрнёмся ΠΊ этому эффСкту.

Π‘Π²Π΅Ρ‚, ΠΏΠΎΠΌΠΈΠΌΠΎ ΠΏΡ€ΠΎΡ‡Π΅Π³ΠΎ, являСтся ΠΏΠΎΡ‚ΠΎΠΊΠΎΠΌ Ρ„ΠΎΡ‚ΠΎΠ½ΠΎΠ², ΠΊΠ²Π°Π½Ρ‚ΠΎΠ² энСргии. Π€ΠΎΡ‚ΠΎΠ½ ΠΌΠΎΠΆΠ΅Ρ‚ Π±Ρ‹Ρ‚ΡŒ ΠΏΠΎΠ³Π»ΠΎΡ‰Ρ‘Π½ элСктроном. Π’ Ρ€Π΅Π·ΡƒΠ»ΡŒΡ‚Π°Ρ‚Π΅ энСргия элСктрона увСличиваСтся Π½Π° Π²Π΅Π»ΠΈΡ‡ΠΈΠ½Ρƒ, Ρ€Π°Π²Π½ΡƒΡŽ энСргии ΠΏΠΎΠ³Π»ΠΎΡ‰Ρ‘Π½Π½ΠΎΠ³ΠΎ Ρ„ΠΎΡ‚ΠΎΠ½Π°. Π§Ρ‚ΠΎΠ±Ρ‹ Ρ‚Π°ΠΊΠΎΠ΅ ΠΏΠΎΠ³Π»ΠΎΡ‰Π΅Π½ΠΈΠ΅ стало Π²ΠΎΠ·ΠΌΠΎΠΆΠ½Ρ‹ΠΌ, Π΄ΠΎΠ»ΠΆΠ½ΠΎ ΡΡƒΡ‰Π΅ΡΡ‚Π²ΠΎΠ²Π°Ρ‚ΡŒ нСзанятоС состояниС с большСй энСргиСй, ΠΊΡƒΠ΄Π° элСктрон ΠΌΠΎΠ³ Π±Ρ‹ ΠΏΠ΅Ρ€Π΅ΠΉΡ‚ΠΈ. Если Ρ‚Π°ΠΊΠΎΠ³ΠΎ состояния Π½Π΅Ρ‚, Ρ„ΠΎΡ‚ΠΎΠ½ Π½Π΅ ΠΌΠΎΠΆΠ΅Ρ‚ ΠΏΠΎΠ³Π»ΠΎΡ‚ΠΈΡ‚ΡŒΡΡ ΠΈ Π±ΡƒΠ΄Π΅Ρ‚ ΠΏΡ€ΠΎΠ΄ΠΎΠ»ΠΆΠ°Ρ‚ΡŒ своё Π΄Π²ΠΈΠΆΠ΅Π½ΠΈΠ΅. Π’ΠΎΡ‚ ΠΌΡ‹ ΠΈ подошли ΠΊ объяснСнию Ρ‚ΠΎΠ³ΠΎ, ΠΏΠΎΡ‡Π΅ΠΌΡƒ ΠΎΡ‚Π΄Π΅Π»ΡŒΠ½Ρ‹Π΅ Ρ‚Π΅Π»Π° ΠΏΡ€ΠΎΠ·Ρ€Π°Ρ‡Π½Ρ‹ для свСта, Π° Π΄Ρ€ΡƒΠ³ΠΈΠ΅ – Π½Π΅Ρ‚.

Как ΠΏΡ€ΠΈΠΌΠ΅Ρ€ ΠΏΡ€ΠΎΠ·Ρ€Π°Ρ‡Π½ΠΎΠ³ΠΎ вСщСства рассмотрим стСкло. Оно являСтся диэлСктриком, ΠΈ ΠΌΡ‹ качСствСнно прСдставляСм сСбС, ΠΊΠ°ΠΊ выглядит элСктронная структура стСкла, Ссли ΠΈΡΠΏΠΎΠ»ΡŒΠ·ΠΎΠ²Π°Ρ‚ΡŒ понятиС энСргСтичСских Π·ΠΎΠ½. На рис. 9-7 ΠΏΠΎΠΊΠ°Π·Π°Π½Ρ‹ Π΄Π²Π΅ энСргСтичСскиС Π·ΠΎΠ½Ρ‹, достаточныС для объяснСния явлСния. НиТняя Π·ΠΎΠ½Π° Π·Π°ΠΏΠΎΠ»Π½Π΅Π½Π°: ΠΊΠ°ΠΆΠ΄ΠΎΠ΅ ΠΊΠ²Π°Π½Ρ‚ΠΎΠ²ΠΎΠ΅ состояниС занято элСктроном. Над этой Π·ΠΎΠ½ΠΎΠΉ находится отдСлённая энСргСтичСской Ρ‰Π΅Π»ΡŒΡŽ (Π·ΠΎΠ½ΠΎΠΉ Π·Π°ΠΏΡ€Π΅Ρ‰Ρ‘Π½Π½Ρ‹Ρ… энСргий) пустая Π·ΠΎΠ½Π°, Π² ΠΊΠΎΡ‚ΠΎΡ€ΠΎΠΉ Π½Π΅Ρ‚ элСктронов. Расстояния ΠΏΠΎ Π²Π΅Ρ€Ρ‚ΠΈΠΊΠ°Π»ΠΈ Π½Π° рисункС ΡΠΎΠΎΡ‚Π²Π΅Ρ‚ΡΡ‚Π²ΡƒΡŽΡ‚ значСниям энСргий. ΠΠ°Ρ‡Π°Π»ΡŒΠ½Ρ‹ΠΉ Ρ„ΠΎΡ‚ΠΎΠ½ с энСргиСй, ΡΠΎΠΎΡ‚Π²Π΅Ρ‚ΡΡ‚Π²ΡƒΡŽΡ‰Π΅ΠΉ Π΄Π»ΠΈΠ½Π΅ ΠΎΡ‚Ρ€Π΅Π·ΠΊΠ° АВ, которая мСньшС, Ρ‡Π΅ΠΌ Π²Π΅Π»ΠΈΡ‡ΠΈΠ½Π° энСргСтичСской Ρ‰Π΅Π»ΠΈ, Π½Π΅ ΠΌΠΎΠΆΠ΅Ρ‚ Π±Ρ‹Ρ‚ΡŒ ΠΏΠΎΠ³Π»ΠΎΡ‰Ρ‘Π½ Π½ΠΈ ΠΎΠ΄Π½ΠΈΠΌ ΠΈΠ· элСктронов Π² Π·Π°ΠΏΠΎΠ»Π½Π΅Π½Π½ΠΎΠΉ Π·ΠΎΠ½Π΅, Ρ‚.ΠΊ. ΠΊΠΎΠ½Π΅Ρ‡Π½ΠΎΠ΅ состояниС элСктрона Π΄ΠΎΠ»ΠΆΠ½ΠΎ Π½Π°Ρ…ΠΎΠ΄ΠΈΡ‚ΡŒΡΡ Π»ΠΈΠ±ΠΎ Π² Π·Π°ΠΏΠΎΠ»Π½Π΅Π½Π½ΠΎΠΉ Π·ΠΎΠ½Π΅, Π³Π΄Π΅ всС мСста заняты, Π»ΠΈΠ±ΠΎ ΠΏΠΎΠΏΠ°Π΄Π°Π΅Ρ‚ Π² Ρ‰Π΅Π»ΡŒ, Π³Π΄Π΅ Π²ΠΎΠΎΠ±Ρ‰Π΅ Π½Π΅ ΠΌΠΎΠΆΠ΅Ρ‚ Π±Ρ‹Ρ‚ΡŒ Π½ΠΈΠΊΠ°ΠΊΠΈΡ… ΠΊΠ²Π°Π½Ρ‚ΠΎΠ²Ρ‹Ρ… состояний элСктронов. ΠŸΠΎΡΡ‚ΠΎΠΌΡƒ Ρ‚Π°ΠΊΠΎΠΉ Ρ„ΠΎΡ‚ΠΎΠ½ Π±ΡƒΠ΄Π΅Ρ‚ ΠΏΡ€ΠΎΡ…ΠΎΠ΄ΠΈΡ‚ΡŒ сквозь вСщСство. ΠŸΡƒΡΡ‚ΡŒ Ρ‚Π΅ΠΏΠ΅Ρ€ΡŒ Ρ„ΠΎΡ‚ΠΎΠ½Ρ‹ ΠΈΠΌΠ΅ΡŽΡ‚ ΡΠ½Π΅Ρ€Π³ΠΈΡŽ Π±ΠΎΠ»ΡŒΡˆΡƒΡŽ, Ρ‡Π΅ΠΌ ΡˆΠΈΡ€ΠΈΠ½Π° энСргСтичСской Ρ‰Π΅Π»ΠΈ, Π½Π°ΠΏΡ€ΠΈΠΌΠ΅Ρ€, CD. Π’ΠΎΠ³Π΄Π° элСктрон Π² Π½ΠΈΠΆΠ½Π΅ΠΉ Π·ΠΎΠ½Π΅ ΠΌΠΎΠΆΠ΅Ρ‚ ΠΏΠΎΠ³Π»ΠΎΡ‚ΠΈΡ‚ΡŒ Ρ„ΠΎΡ‚ΠΎΠ½ ΠΈ ΠΏΠ΅Ρ€Π΅ΠΉΡ‚ΠΈ Π² нСзанятоС состояниС Π² Π²Π΅Ρ€Ρ…Π½Π΅ΠΉ Π·ΠΎΠ½Π΅.

Рис. 9-7. ЭнСргСтичСскиС Π·ΠΎΠ½Ρ‹ элСктронов Π² ΠΏΡ€ΠΎΠ·Ρ€Π°Ρ‡Π½ΠΎΠΌ вСщСствС Ρ‚ΠΈΠΏΠ° стСкла. Π€ΠΎΡ‚ΠΎΠ½Ρ‹ с энСргиями АВ (Π²Π΅Π»ΠΈΡ‡ΠΈΠ½Π΅ энСргии соотвСтствуСт Π΄Π»ΠΈΠ½Π° ΠΎΡ‚Ρ€Π΅Π·ΠΊΠ° АВ), мСньшими ΡˆΠΈΡ€ΠΈΠ½Ρ‹ Ρ‰Π΅Π»ΠΈ ΠΌΠ΅ΠΆΠ΄Ρƒ занятой ΠΈ Π½Π΅Π·Π°ΠΏΠΎΠ»Π½Π΅Π½Π½ΠΎΠΉ Π·ΠΎΠ½Π°ΠΌΠΈ, Π½Π΅ ΠΌΠΎΠ³ΡƒΡ‚ ΠΏΠΎΠ³Π»ΠΎΡ‚ΠΈΡ‚ΡŒΡΡ элСктронами, Ρ‚.ΠΊ. Π½Π΅Ρ‚ Ρ‚Π°ΠΊΠΈΡ… ΠΊΠΎΠ½Π΅Ρ‡Π½Ρ‹Ρ… состояний, Π² ΠΊΠΎΡ‚ΠΎΡ€ΠΎΠ΅ Ρ‚Π΅ ΠΌΠΎΠ³Π»ΠΈ Π±Ρ‹ ΠΏΠ΅Ρ€Π΅ΠΉΡ‚ΠΈ. Π’Π°ΠΊΠΈΠ΅ Ρ„ΠΎΡ‚ΠΎΠ½Ρ‹ проходят сквозь вСщСство. Если ΠΆΠ΅ энСргии Ρ„ΠΎΡ‚ΠΎΠ½ΠΎΠ² ΠΏΡ€Π΅Π²Ρ‹ΡˆΠ°ΡŽΡ‚ ΡˆΠΈΡ€ΠΈΠ½Ρƒ Ρ‰Π΅Π»ΠΈ (CD Π½Π° рисункС), Ρ‚ΠΎ Ρ‚Π°ΠΊΠΈΠ΅ Ρ„ΠΎΡ‚ΠΎΠ½Ρ‹ ΠΏΠΎΠ³Π»ΠΎΡ‰Π°ΡŽΡ‚ΡΡ вСщСством

Π’Π΅ΠΏΠ΅Ρ€ΡŒ ΠΌΠΎΠΆΠ½ΠΎ ΠΏΠΎΠ½ΡΡ‚ΡŒ, ΠΏΠΎΡ‡Π΅ΠΌΡƒ стСкло ΠΏΡ€ΠΎΠ·Ρ€Π°Ρ‡Π½ΠΎ для Π²ΠΈΠ΄ΠΈΠΌΠΎΠ³ΠΎ свСта. Зонная структура стСкла Ρ‚Π°ΠΊΠΎΠ²Π°, Ρ‡Ρ‚ΠΎ Ρƒ Ρ„ΠΎΡ‚ΠΎΠ½ΠΎΠ² Π²ΠΈΠ΄ΠΈΠΌΠΎΠ³ΠΎ свСта Π½Π΅ Ρ…Π²Π°Ρ‚Π°Π΅Ρ‚ энСргии, Ρ‡Ρ‚ΠΎΠ±Ρ‹ ΠΏΠ΅Ρ€Π΅Π±Ρ€ΠΎΡΠΈΡ‚ΡŒ элСктроны ΠΈΠ· Π½ΠΈΠΆΠ½Π΅ΠΉ Π·ΠΎΠ½Ρ‹ Π² Π²Π΅Ρ€Ρ…Π½ΡŽΡŽ, ΠΏΡ€Π΅ΠΎΠ΄ΠΎΠ»Π΅Π² Ρ‰Π΅Π»ΡŒ. ΠŸΠΎΡΡ‚ΠΎΠΌΡƒ Ρ„ΠΎΡ‚ΠΎΠ½Ρ‹ Π²ΠΈΠ΄ΠΈΠΌΠΎΠ³ΠΎ свСта проходят сквозь стСкло Π±Π΅Π· поглощСния. Но Ρ„ΠΎΡ‚ΠΎΠ½Ρ‹ ΡƒΠ»ΡŒΡ‚Ρ€Π°Ρ„ΠΈΠΎΠ»Π΅Ρ‚ΠΎΠ²ΠΎΠ³ΠΎ свСта ΠΈΠΌΠ΅ΡŽΡ‚ Π±ΠΎΠ»Π΅Π΅ Π²Ρ‹ΡΠΎΠΊΡƒΡŽ частоту, ΡΠ»Π΅Π΄ΠΎΠ²Π°Ρ‚Π΅Π»ΡŒΠ½ΠΎ, Π±ΠΎΠ»ΡŒΡˆΡƒΡŽ ΡΠ½Π΅Ρ€Π³ΠΈΡŽ, ΠΊΠΎΡ‚ΠΎΡ€ΠΎΠΉ оказываСтся достаточно, Ρ‡Ρ‚ΠΎΠ±Ρ‹ Π²Ρ‹Ρ€Π²Π°Ρ‚ΡŒ элСктроны ΠΈΠ· Π·Π°ΠΏΠΎΠ»Π½Π΅Π½Π½ΠΎΠΉ Π·ΠΎΠ½Ρ‹ ΠΈ ΠΏΠ΅Ρ€Π΅Π±Ρ€ΠΎΡΠΈΡ‚ΡŒ ΠΈΡ… Π² Π½Π΅Π·Π°Π½ΡΡ‚ΡƒΡŽ Π·ΠΎΠ½Ρƒ. ΠŸΠΎΡΡ‚ΠΎΠΌΡƒ Ρ‚Π°ΠΊΠΎΠΉ свСт Π±ΡƒΠ΄Π΅Ρ‚ ΠΏΠΎΠ³Π»ΠΎΡ‰Π°Ρ‚ΡŒΡΡ стСклом. На рис. 9-7 ΠΎΡ‚Ρ€Π΅Π·ΠΊΡƒ АВ соотвСтствуСт Ρ„ΠΎΡ‚ΠΎΠ½ Π²ΠΈΠ΄ΠΈΠΌΠΎΠ³ΠΎ свСта, Π° ΠΎΡ‚Ρ€Π΅Π·ΠΊΡƒ CD – ΡƒΠ»ΡŒΡ‚Ρ€Π°Ρ„ΠΈΠΎΠ»Π΅Ρ‚ΠΎΠ²ΠΎΠ³ΠΎ. Π’ ΠΊΠ²Π°Ρ€Ρ†Π΅ энСргСтичСская Ρ‰Π΅Π»ΡŒ ΡˆΠΈΡ€Π΅, Ρ‡Π΅ΠΌ Ρƒ ΠΎΠ±Ρ‹Ρ‡Π½ΠΎΠ³ΠΎ стСкла, поэтому Ρ„ΠΎΡ‚ΠΎΠ½Ρ‹ Π½Π΅ Ρ‚ΠΎΠ»ΡŒΠΊΠΎ Π²ΠΈΠ΄ΠΈΠΌΠΎΠ³ΠΎ, Π½ΠΎ ΠΈ ΡƒΠ»ΡŒΡ‚Ρ€Π°Ρ„ΠΈΠΎΠ»Π΅Ρ‚ΠΎΠ²ΠΎΠ³ΠΎ свСта Π½Π΅ ΠΎΠ±Π»Π°Π΄Π°ΡŽΡ‚ достаточной энСргиСй, Ρ‡Ρ‚ΠΎΠ±Ρ‹ элСктроны Π² Π·Π°ΠΏΠΎΠ»Π½Π΅Π½Π½ΠΎΠΉ Π·ΠΎΠ½Π΅ ΠΌΠΎΠ³Π»ΠΈ ΠΈΡ… ΠΏΠΎΠ³Π»ΠΎΡ‚ΠΈΡ‚ΡŒ ΠΈ ΠΏΠ΅Ρ€Π΅ΠΉΡ‚ΠΈ Π² Π²Π΅Ρ€Ρ…Π½ΡŽΡŽ ΠΏΡƒΡΡ‚ΡƒΡŽ Π·ΠΎΠ½Ρƒ. Π’ этом ΠΏΡ€ΠΈΡ‡ΠΈΠ½Π° Ρ‚ΠΎΠ³ΠΎ, Ρ‡Ρ‚ΠΎ ΡƒΠ»ΡŒΡ‚Ρ€Π°Ρ„ΠΈΠΎΠ»Π΅Ρ‚ΠΎΠ²Ρ‹Π΅ Π»Π°ΠΌΠΏΡ‹ Π΄Π΅Π»Π°ΡŽΡ‚ΡΡ Π½Π΅ ΠΈΠ· ΠΎΠ±Ρ‹Ρ‡Π½ΠΎΠ³ΠΎ стСкла, Π° ΠΈΠ· ΠΊΠ²Π°Ρ€Ρ†Π°.

Буммируя, ΠΌΠΎΠΆΠ½ΠΎ ΡƒΡ‚Π²Π΅Ρ€ΠΆΠ΄Π°Ρ‚ΡŒ, Ρ‡Ρ‚ΠΎ свСт ΠΌΠΎΠΆΠ΅Ρ‚ ΠΏΠΎΠ³Π»ΠΎΡ‚ΠΈΡ‚ΡŒΡΡ элСктронами Π² вСщСствС, Ссли ΠΎΠ½ΠΈ способны ΠΏΠ΅Ρ€Π΅ΠΉΡ‚ΠΈ Π² состояния, энСргия ΠΊΠΎΡ‚ΠΎΡ€Ρ‹Ρ… большС Π½Π°Ρ‡Π°Π»ΡŒΠ½ΠΎΠΉ энСргии Π½Π° Π²Π΅Π»ΠΈΡ‡ΠΈΠ½Ρƒ, Ρ€Π°Π²Π½ΡƒΡŽ энСргии Ρ„ΠΎΡ‚ΠΎΠ½Π°. Если Π² Π·ΠΎΠ½Π½ΠΎΠΉ структурС Π΄Π°Π½Π½ΠΎΠ³ΠΎ вСщСства Ρ‚Π°ΠΊΠΈΡ… состояний Π½Π΅Ρ‚, Ρ„ΠΎΡ‚ΠΎΠ½Ρ‹ бСспрСпятствСнно проходят сквозь вСщСство, ΠΎΠ½ΠΎ ΠΏΡ€ΠΎΠ·Ρ€Π°Ρ‡Π½ΠΎ для свСта Π΄Π°Π½Π½ΠΎΠ³ΠΎ Ρ†Π²Π΅Ρ‚Π°. Подобная ΠΊΠ°Ρ€Ρ‚ΠΈΠ½Π° ΠΎΠ±ΡŠΡΡΠ½ΡΠ΅Ρ‚ Ρ‚Π°ΠΊΠΆΠ΅, ΠΏΠΎΡ‡Π΅ΠΌΡƒ вСщСство ΠΌΠΎΠΆΠ΅Ρ‚ Π±Ρ‹Ρ‚ΡŒ ΠΏΡ€ΠΎΠ·Ρ€Π°Ρ‡Π½Ρ‹ΠΌ для Π²ΠΎΠ»Π½ ΠΎΠ΄Π½ΠΎΠΉ части элСктромагнитного спСктра ΠΈ Π½Π΅ΠΏΡ€ΠΎΠ·Ρ€Π°Ρ‡Π½Ρ‹ΠΌ для Π΄Ρ€ΡƒΠ³ΠΎΠΉ части.

Рассмотрим Ρ‚Π΅ΠΏΠ΅Ρ€ΡŒ случай свСта, ΠΏΠ°Π΄Π°ΡŽΡ‰Π΅Π³ΠΎ Π½Π° ΠΏΠΎΠ²Π΅Ρ€Ρ…Π½ΠΎΡΡ‚ΡŒ ΠΌΠ΅Ρ‚Π°Π»Π»Π°. ΠœΡ‹ Π·Π½Π°Π΅ΠΌ, Ρ‡Ρ‚ΠΎ ΠΌΠ΅Ρ‚Π°Π»Π» Ρ…ΠΎΡ€ΠΎΡˆΠΎ ΠΏΡ€ΠΎΠ²ΠΎΠ΄ΠΈΡ‚ элСктричСский Ρ‚ΠΎΠΊ, Ρ‚.ΠΊ. внСшниС элСктроны ΠΊΠ°ΠΆΠ΄ΠΎΠ³ΠΎ Π°Ρ‚ΠΎΠΌΠ° свободно ΠΏΡƒΡ‚Π΅ΡˆΠ΅ΡΡ‚Π²ΡƒΡŽΡ‚ ΠΏΠΎ всСму ΠΌΠ΅Ρ‚Π°Π»Π»Ρƒ. На рис. 9-8 ΠΏΠΎΠΊΠ°Π·Π°Π½ ΠΏΡƒΡ‡ΠΎΠΊ свСта, ΠΎΡ‚Ρ€Π°ΠΆΠ°ΡŽΡ‰ΠΈΠΉΡΡ ΠΎΡ‚ повСрхности ΠΌΠ΅Ρ‚Π°Π»Π»Π°. Π‘Π»ΠΈΠΆΠ°ΠΉΡˆΠΈΠ΅ ΠΊ повСрхности элСктроны ΠΈΡΠΏΡ‹Ρ‚Ρ‹Π²Π°ΡŽΡ‚ дСйствиС силы со стороны ΠΊΠΎΠ»Π΅Π±Π»ΡŽΡ‰Π΅Π³ΠΎΡΡ элСктричСского поля свСтовой Π²ΠΎΠ»Π½Ρ‹ ΠΈ Π½Π°Ρ‡ΠΈΠ½Π°ΡŽΡ‚ сами ΠΊΠΎΠ»Π΅Π±Π°Ρ‚ΡŒΡΡ с Ρ‚ΠΎΠΉ ΠΆΠ΅ частотой, Ρ‡Ρ‚ΠΎ ΠΈ свСт. Один ΠΈΠ· этих элСктронов, ΠΏΠΎΠΌΠ΅Ρ‡Π΅Π½Π½Ρ‹ΠΉ Ρ†ΠΈΡ„Ρ€ΠΎΠΉ 1, ΠΏΠΎΠΊΠ°Π·Π°Π½ Π½Π° рисункС. Π’Π°ΠΊΠΎΠΉ ΠΊΠΎΠ»Π΅Π±Π»ΡŽΡ‰ΠΈΠΉΡΡ слой элСктронов ΠΎΡ‚Π±ΠΈΡ€Π°Π΅Ρ‚ ΡΠ½Π΅Ρ€Π³ΠΈΡŽ Ρƒ свСтовой Π²ΠΎΠ»Π½Ρ‹, поэтому ΠΎΠ½Π° ослабляСтся ΠΈ заставляСт ΠΊΠΎΠ»Π΅Π±Π°Ρ‚ΡŒΡΡ ΡΠ»Π΅Π΄ΡƒΡŽΡ‰ΠΈΠΉ слой элСктронов (ΠΏΠΎΠΌΠ΅Ρ‡Π΅Π½Π½Ρ‹ΠΉ Ρ†ΠΈΡ„Ρ€ΠΎΠΉ 2) с мСньшСй частотой ΠΈ Ρ‚.Π΄., Π΄ΠΎ Ρ‚Π΅Ρ… ΠΏΠΎΡ€, ΠΏΠΎΠΊΠ° Π½Π° ΠΎΠΏΡ€Π΅Π΄Π΅Π»Ρ‘Π½Π½ΠΎΠΉ Π³Π»ΡƒΠ±ΠΈΠ½Π΅ ΠΎΡ‚ повСрхности колСбания поля Π² свСтовой Π²ΠΎΠ»Π½Π΅ ΠΈ ΡΠΎΠΎΡ‚Π²Π΅Ρ‚ΡΡ‚Π²ΡƒΡŽΡ‰ΠΈΠ΅ колСбания элСктронов Π½Π΅ Π·Π°Ρ‚ΡƒΡ…Π½ΡƒΡ‚. Π’ Π±ΠΎΠ»ΡŒΡˆΠΈΠ½ΡΡ‚Π²Π΅ ΠΌΠ΅Ρ‚Π°Π»Π»ΠΎΠ² эта Π³Π»ΡƒΠ±ΠΈΠ½Π° ΠΎΡ‡Π΅Π½ΡŒ ΠΌΠ°Π»Π° ΠΈ составляСт Π²Π΅Π»ΠΈΡ‡ΠΈΠ½Ρƒ порядка 10–6см. Π’Π΅ΠΏΠ΅Ρ€ΡŒ понятно, ΠΏΠΎΡ‡Π΅ΠΌΡƒ Π΄Π°ΠΆΠ΅ ΠΎΡ‡Π΅Π½ΡŒ Ρ‚ΠΎΠ½ΠΊΠΈΠΉ слой ΠΌΠ΅Ρ‚Π°Π»Π»Π° Π½Π΅ пропускаСт свСт.

Рис. 9-8. ΠŸΠΎΠ³Π»ΠΎΡ‰Π΅Π½ΠΈΠ΅ ΠΈ ΠΎΡ‚Ρ€Π°ΠΆΠ΅Π½ΠΈΠ΅ свСта ΠΎΡ‚ ΠΌΠ΅Ρ‚Π°Π»Π»Π°. ΠžΡΡ†ΠΈΠ»Π»ΠΈΡ€ΡƒΡŽΡ‰Π΅Π΅ элСктричСскоС ΠΏΠΎΠ»Π΅ свСтовой Π²ΠΎΠ»Π½Ρ‹ заставляСт ΠΊΠΎΠ»Π΅Π±Π°Ρ‚ΡŒΡΡ элСктроны проводимости (ΠΏΠΎΠΊΠ°Π·Π°Π½Ρ‹ Ρ‡Π΅Ρ‚Ρ‹Ρ€Π΅ элСктрона, ΠΏΡ€ΠΈΡ‡Ρ‘ΠΌ Π΄Π²Π° ΠΏΠ΅Ρ€Π²Ρ‹Ρ… ΠΏΡ€ΠΎΠ½ΡƒΠΌΠ΅Ρ€ΠΎΠ²Π°Π½Ρ‹). Амплитуды ΠΊΠΎΠ»Π΅Π±Π°Π½ΠΈΠΉ элСктронов ΠΏΠΎΠΊΠ°Π·Π°Π½Ρ‹ стрСлками. Π­Ρ‚ΠΈ стрСлки ΡƒΠΌΠ΅Π½ΡŒΡˆΠ°ΡŽΡ‚ΡΡ ΠΏΡ€ΠΈ ΠΏΡ€ΠΎΠ΄Π²ΠΈΠΆΠ΅Π½ΠΈΠΈ Π² Π³Π»ΡƒΠ±ΡŒ ΠΌΠ΅Ρ‚Π°Π»Π»Π°, Ρ‡Ρ‚ΠΎ соотвСтствуСт Π·Π°Ρ‚ΡƒΡ…Π°Π½ΠΈΡŽ ΠΊΠΎΠ»Π΅Π±Π°Π½ΠΈΠΉ. Π’Π°ΠΊΠΈΠΌ ΠΎΠ±Ρ€Π°Π·ΠΎΠΌ, элСктрон частично ΠΏΠΎΠ³Π»ΠΎΡ‰Π°Π΅Ρ‚ ΡΠ½Π΅Ρ€Π³ΠΈΡŽ ΠΏΠ°Π΄Π°ΡŽΡ‰Π΅Π³ΠΎ Ρ„ΠΎΡ‚ΠΎΠ½Π°, Π° частично ΠΏΠ΅Ρ€Π΅ΠΈΠ·Π»ΡƒΡ‡Π°Π΅Ρ‚ Π΅Ρ‘ Π² ΠΎΠΊΡ€ΡƒΠΆΠ°ΡŽΡ‰Π΅Π΅ пространство

Π’Ρ‹ ΠΌΠΎΠΆΠ΅Ρ‚Π΅ Π·Π°Π΄Π°Ρ‚ΡŒ вопрос: ΠΎΡ‚ΠΊΡƒΠ΄Π° ΠΆΠ΅ бСрётся свСт, ΠΎΡ‚Ρ€Π°ΠΆΡ‘Π½Π½Ρ‹ΠΉ ΠΎΡ‚ повСрхности ΠΌΠ΅Ρ‚Π°Π»Π»Π°, Ρ‚. ΠΊ. создаётся Π²ΠΏΠ΅Ρ‡Π°Ρ‚Π»Π΅Π½ΠΈΠ΅, Ρ‡Ρ‚ΠΎ вся энСргия ΠΏΠ°Π΄Π°ΡŽΡ‰Π΅Π³ΠΎ Ρ„ΠΎΡ‚ΠΎΠ½Π° ΠΏΠ΅Ρ€Π΅Ρ…ΠΎΠ΄ΠΈΡ‚ Π² ΡΠ½Π΅Ρ€Π³ΠΈΡŽ ΠΊΠΎΠ»Π΅Π±Π°Π½ΠΈΠΉ элСктронов? Однако вспомним, Ρ‡Ρ‚ΠΎ элСктромагнитныС Π²ΠΎΠ»Π½Ρ‹ ΠΏΠΎΡ€ΠΎΠΆΠ΄Π°ΡŽΡ‚ΡΡ ΠΊΠΎΠ»Π΅Π±Π»ΡŽΡ‰ΠΈΠΌΠΈΡΡ зарядами. Π­Π»Π΅ΠΊΡ‚Ρ€ΠΎΠ½Ρ‹ Π½Π° повСрхности ΠΌΠ΅Ρ‚Π°Π»Π»Π° ΠΊΠΎΠ»Π΅Π±Π»ΡŽΡ‚ΡΡ с Ρ‚ΠΎΠΉ ΠΆΠ΅ частотой, Ρ‡Ρ‚ΠΎ ΠΈ ΠΏΠ°Π΄Π°ΡŽΡ‰ΠΈΠΉ свСт. ИмСнно эти осцилляции ΠΈ ΠΏΠΎΡ€ΠΎΠΆΠ΄Π°ΡŽΡ‚ элСктромагнитныС Π²ΠΎΠ»Π½Ρ‹ Ρ‚ΠΎΠΉ ΠΆΠ΅ частоты, Ρ‡Ρ‚ΠΎ ΠΈ ΠΏΠ°Π΄Π°ΡŽΡ‰Π°Ρ Π²ΠΎΠ»Π½Π°, Π²ΠΎΡ‚ ΠΎΠ½ΠΈ-Ρ‚ΠΎ ΠΈ ΠΎΠ±Ρ€Π°Π·ΡƒΡŽΡ‚ ΠΎΡ‚Ρ€Π°ΠΆΡ‘Π½Π½Ρ‹ΠΉ свСт.

Π‘ΠΎΠΊΡ€. ΠΏΠ΅Ρ€. с Π°Π½Π³Π». А.Π’.Π‘Π΅Ρ€ΠΊΠΎΠ²Π°

ΠŸΡ€ΠΎΠ΄ΠΎΠ»ΠΆΠ΅Π½ΠΈΠ΅ Π² β„– 10

2.9 Π”Π²ΠΈΠΆΠ΅Π½ΠΈΠ΅ частиц вСщСства Π² свободном пространствС ΠΏΠΎΠ΄ дСйствиСм элСктростатичСских ΠΈ магнитостатичСских ΠΏΠΎΠ»Π΅ΠΉ

2.9.1 На ΠΊΠ°ΠΊΠΈΠ΅ частицы вСщСства дСйствуСт элСктричСскоС ΠΏΠΎΠ»Π΅?

☻ ЭлСктричСскоС ΠΏΠΎΠ»Π΅ ΠΏΡ€ΠΎΠΈΠ·Π²ΠΎΠ΄ΠΈΡ‚ силовоС дСйствиС Π½Π° частицы вСщСства, ΠΎΠ±Π»Π°Π΄Π°ΡŽΡ‰ΠΈΠ΅ элСктричСским зарядом. Π­Ρ‚ΠΎ ΠΌΠΎΠ³ΡƒΡ‚ Π±Ρ‹Ρ‚ΡŒ Π½Π΅ Ρ‚ΠΎΠ»ΡŒΠΊΠΎ микрочастицы — элСктроны, ΠΏΡ€ΠΎΡ‚ΠΎΠ½Ρ‹, ΠΈΠΎΠ½Ρ‹ ΠΈ Ρ‚.ΠΏ., Π½ΠΎ ΠΈ макрочастицы β€” частицы ΠΏΡ‹Π»ΠΈ, Π΄Ρ‹ΠΌΠ°, Ρ‚ΡƒΠΌΠ°Π½Π° ΠΈ Ρ‚. Π΄. ΠšΡ€ΠΎΠΌΠ΅ Ρ‚ΠΎΠ³ΠΎ, элСктричСскоС ΠΏΠΎΠ»Π΅ Ρ‚Π°ΠΊΠΆΠ΅ дСйствуСт ΠΈ Π½Π° ΡΠ»Π΅ΠΊΡ‚Ρ€ΠΎΠ½Π΅ΠΉΡ‚Ρ€Π°Π»ΡŒΠ½Ρ‹Π΅ частицы с ΠΎΠ±Ρ‰ΠΈΠΌ Π½ΡƒΠ»Π΅Π²Ρ‹ΠΌ зарядом, Π½ΠΎ ΠΎΠ±Π»Π°Π΄Π°ΡŽΡ‰ΠΈΠ΅ элСктричСской ΠΏΠΎΠ»ΡΡ€Π½ΠΎΡΡ‚ΡŒΡŽ ΠΈΠ»ΠΈ способныС приобрСсти ΠΏΠΎΠ»ΡΡ€Π½ΠΎΡΡ‚ΡŒ Π² элСктричСском ΠΏΠΎΠ»Π΅. Π­Ρ‚ΠΎ Ρ‚ΠΎΠΆΠ΅ ΠΌΠΎΠ³ΡƒΡ‚ Π±Ρ‹Ρ‚ΡŒ ΠΈ ΠΌΠΈΠΊΡ€ΠΎ — ΠΈ ΠΌΠ°ΠΊΡ€ΠΎ частицы.

2.9.2 Π”Π²ΠΈΠΆΠ΅Π½ΠΈΠ΅ свободной заряТСнной частицы Π² элСктростатичСском ΠΏΠΎΠ»Π΅

☻ Π”Π²ΠΈΠΆΠ΅Π½ΠΈΠ΅ частицы зависит ΠΎΡ‚ Π΅Π΅ массы ΠΈ Π΄Π΅ΠΉΡΡ‚Π²ΡƒΡŽΡ‰Π΅ΠΉ Π½Π° Π½Π΅Π΅ силы ΠΈ происходит ΠΏΠΎ Π·Π°ΠΊΠΎΠ½Ρƒ . ДСйствуя Π½Π° заряд частицы силой, элСктростатичСскоС ΠΏΠΎΠ»Π΅ Π²ΠΎΠ·Π±ΡƒΠΆΠ΄Π°Π΅Ρ‚ Π΅Π΅ Π΄Π²ΠΈΠΆΠ΅Π½ΠΈΠ΅ Π² свободном пространствС ΠΏΠΎ Π·Π°ΠΊΠΎΠ½Ρƒ, Ρ‡Ρ‚ΠΎ соотвСтствуСт с ΡƒΡ€Π°Π²Π½Π΅Π½ΠΈΡŽ.

,

Π³Π΄Π΅ — радиус – Π²Π΅ΠΊΡ‚ΠΎΡ€ частицы.

Π£Ρ€Π°Π²Π½Π΅Π½ΠΈΠ΅ содСрТит ΠΈΠ½Ρ„ΠΎΡ€ΠΌΠ°Ρ†ΠΈΡŽ ΠΎ скорости ΠΈ Ρ‚Ρ€Π°Π΅ΠΊΡ‚ΠΎΡ€ΠΈΠΈ двиТСния частицы ΠΈ, Π² зависимости ΠΎΡ‚ ΠΊΠΎΠ½ΠΊΡ€Π΅Ρ‚Π½Ρ‹Ρ… особСнностСй поля ΠΈ ΠΎΡ‚ Π½Π°Ρ‡Π°Π»ΡŒΠ½Ρ‹Ρ… условийи

ΠΏΡ€ΠΈ t=0. ΠžΡΠΎΠ±Π΅Π½Π½ΠΎΡΡ‚ΠΈ элСктростатичСских ΠΏΠΎΠ»Π΅ΠΉ практичСски Π±Π΅Π·Π³Ρ€Π°Π½ΠΈΡ‡Π½Ρ‹. Π­Ρ‚ΠΎ ΠΌΠΎΠ³ΡƒΡ‚ Π±Ρ‹Ρ‚ΡŒ ΠΎΠ΄Π½ΠΎΡ€ΠΎΠ΄Π½Ρ‹Π΅ ΠΈ Ρ€Π°Π·Π½ΠΎΠΎΠ±Ρ€Π°Π·Π½Ρ‹Π΅ Π½Π΅ΠΎΠ΄Π½ΠΎΡ€ΠΎΠ΄Π½Ρ‹Π΅ поля. Π’ Ρ‚Π°ΠΊΠΈΡ… полях Ρ‚Ρ€Π°Π΅ΠΊΡ‚ΠΎΡ€ΠΈΠΈ частиц ΠΌΠΎΠ³ΡƒΡ‚ Π±Ρ‹Ρ‚ΡŒ ΠΈ прямыми, ΠΈ ΠΏΠ°Ρ€Π°Π±ΠΎΠ»Π°ΠΌΠΈ ΠΈ Π΄Ρ€ΡƒΠ³ΠΈΠΌΠΈ Π±ΠΎΠ»Π΅Π΅ слоТными ΠΊΡ€ΠΈΠ²Ρ‹ΠΌΠΈ. Однако Π½Π° любой ΠΈΠ· Π²ΠΎΠ·ΠΌΠΎΠΆΠ½Ρ‹Ρ… Ρ‚Ρ€Π°Π΅ΠΊΡ‚ΠΎΡ€ΠΈΠΉ ускорСниС частицы

.

Π’Π°ΠΊΠΈΠΌ ΠΎΠ±Ρ€Π°Π·ΠΎΠΌ ускорСниС частицы ΠΏΠΎΠ΄ дСйствиСм поля опрСдСляСтся Π² Π·Π½Π°Ρ‡ΠΈΡ‚Π΅Π»ΡŒΠ½ΠΎΠΉ ΠΌΠ΅Ρ€Π΅ ΠΎΡ‚Π½ΠΎΡˆΠ΅Π½ΠΈΠ΅ΠΌ заряда частицы ΠΊ Π΅Π΅ массС, ΠΈΠ»ΠΈ ΠΈΠ½Π°Ρ‡Π΅ Π΅Π΅ ΡƒΠ΄Π΅Π»ΡŒΠ½Ρ‹ΠΌ зарядом. ΠœΠΈΠΊΡ€ΠΎΡ‡Π°ΡΡ‚ΠΈΡ†Ρ‹ ΠΎΠ±Π»Π°Π΄Π°ΡŽΡ‚ вСсьма большим ΡƒΠ΄Π΅Π»ΡŒΠ½Ρ‹ΠΌ зарядом, Π½Π°ΠΏΡ€ΠΈΠΌΠ΅Ρ€ для элСктронатогда ΠΊΠ°ΠΊ Ρƒ макрочастицγ вСсьма ΠΌΠ°Π»ΠΎ, ΠΎΠ±Ρ‹Ρ‡Π½ΠΎ .

2.9.3 На ΠΊΠ°ΠΊΠΈΠ΅ частицы вСщСства дСйствуСт ΠΌΠ°Π³Π½ΠΈΡ‚Π½ΠΎΠ΅ ΠΏΠΎΠ»Π΅

☻ ΠœΠ°Π³Π½ΠΈΡ‚Π½ΠΎΠ΅ ΠΏΠΎΠ»Π΅ дСйствуСт Π½Π° ΠΌΠ°ΠΊΡ€ΠΎ- ΠΈ микрочастицы, ΠΎΠ±Π»Π°Π΄Π°ΡŽΡ‰ΠΈΠ΅ элСктричСским зарядом, Π½ΠΎ Ρ‚ΠΎΠ»ΡŒΠΊΠΎ Π² состоянии ΠΈΡ… двиТСния. Π‘ΠΈΠ»ΠΎΠ²ΠΎΠΌΡƒ Π²ΠΎΠ·Π΄Π΅ΠΉΡΡ‚Π²ΠΈΡŽ со стороны ΠΌΠ°Π³Π½ΠΈΡ‚Π½ΠΎΠ³ΠΎ поля ΠΏΠΎΠ΄Π²Π΅Ρ€Π³Π°ΡŽΡ‚ΡΡ Ρ‚Π°ΠΊΠΆΠ΅ ΡΠ»Π΅ΠΊΡ‚Ρ€ΠΎΠ½Π΅ΠΉΡ‚Ρ€Π°Π»ΡŒΠ½Ρ‹Π΅ частицы, ΠΎΠ±Π»Π°Π΄Π°ΡŽΡ‰ΠΈΠ΅ ΠΌΠ°Π³Π½ΠΈΡ‚Π½Ρ‹ΠΌ ΠΌΠΎΠΌΠ΅Π½Ρ‚ΠΎΠΌ. Π‘Π²ΠΎΠΈΠΌ дСйствиСм Π½Π° Π°Ρ‚ΠΎΠΌ ΠΌΠ°Π³Π½ΠΈΡ‚Π½ΠΎΠ΅ ΠΏΠΎΠ»Π΅ ΠΌΠΎΠΆΠ΅Ρ‚ Π²Π»ΠΈΡΡ‚ΡŒ Π½Π° Π²Π½ΡƒΡ‚Ρ€Π΅Π½Π½Π΅Π΅ состояниС Π°Ρ‚ΠΎΠΌΠ°.

2.9.4 Π”Π²ΠΈΠΆΠ΅Π½ΠΈΠ΅ свободной заряТСнной частицы Π² ΠΎΠ΄Π½ΠΎΡ€ΠΎΠ΄Π½ΠΎΠΌ магнитостатичСском ΠΏΠΎΠ»Π΅

Рис. 2.9.4

Π”Π²ΠΈΠΆΠ΅Π½ΠΈΠ΅ ΠΏΠΎΠ΄ дСйствиСм силы Π›ΠΎΡ€Π΅Π½Ρ†Π° Π² ΠΎΠ΄Π½ΠΎΡ€ΠΎΠ΄Π½ΠΎΠΌ ΠΌΠ°Π³Π½ΠΈΡ‚Π½ΠΎΠΌ ΠΏΠΎΠ»Π΅

☻ ВсС особСнности силового дСйствия ΠΌΠ°Π³Π½ΠΈΡ‚Π½ΠΎΠ³ΠΎ поля Π½Π° Π·Π°Ρ€ΡΠΆΠ΅Π½Π½ΡƒΡŽ частицу Π²Ρ‹Ρ€Π°ΠΆΠ°ΡŽΡ‚ΡΡ силой Π›ΠΎΡ€Π΅Π½Ρ†Π° ΠΈ ΠΎΠΏΡ€Π΅Π΄Π΅Π»ΡΡŽΡ‚ΡΡ ΡƒΡ€Π°Π²Π½Π΅Π½ΠΈΠ΅ΠΌ:

Π² ΠΊΠΎΡ‚ΠΎΡ€ΠΎΠΌ взаимная ориСнтация Π²Π΅ΠΊΡ‚ΠΎΡ€ΠΎΠ² исоставляСт ΠΏΡ€Π°Π²Ρ‹ΠΉ Π²ΠΈΠ½Ρ‚. ΠŸΠΎΡΠΊΠΎΠ»ΡŒΠΊΡƒ сила Π›ΠΎΡ€Π΅Π½Ρ†Π°, Ρ‚ΠΎ Π²Ρ‹Π·Π²Π°Π½Π½ΠΎΠ΅ Сю ускорСниС частицытоТС пСрпСндикулярно ΠΊ ΡΠΊΠΎΡ€ΠΎΡΡ‚ΠΈΠžΡ‚ΡΡŽΠ΄Π° слСдуСт, Ρ‡Ρ‚ΠΎ Π΄Π²ΠΈΠΆΠ΅Π½ΠΈΠ΅ частицы Π² ΠΌΠ°Π³Π½ΠΈΡ‚Π½ΠΎΠΌ ΠΏΠΎΠ»Π΅ Π΄ΠΎΠ»ΠΆΠ½ΠΎ ΠΏΡ€ΠΎΠΈΡΡ…ΠΎΠ΄ΠΈΡ‚ΡŒ Π»ΠΈΠ±ΠΎ ΠΏΠΎ окруТности, Π»ΠΈΠ±ΠΎ ΠΏΠΎ спирали, ΠΊΠ°ΠΊ это ΠΏΠΎΠΊΠ°Π·Π°Π½ΠΎ Π½Π° рис.2.9.4. для случая ΠΎΠ΄Π½ΠΎΡ€ΠΎΠ΄Π½ΠΎΠ³ΠΎ поля.

ΠŸΠΎΠΏΠ΅Ρ€Π΅Ρ‡Π½ΠΎΠ΅ ΠΊ скорости ΠΌΠ°Π³Π½ΠΈΡ‚Π½ΠΎΠ΅ ΠΏΠΎΠ»Π΅ Π·Π°Ρ…Π²Π°Ρ‚Ρ‹Π²Π°Π΅Ρ‚ частицу ΠΈ Π²ΠΎΠ²Π»Π΅ΠΊΠ°Π΅Ρ‚ Π΅Π΅ Π² Π΄Π²ΠΈΠΆΠ΅Π½ΠΈΠ΅ ΠΏΠΎ ΠΊΡ€ΡƒΠ³ΠΎΠ²ΠΎΠΉ ΠΎΡ€Π±ΠΈΡ‚Π΅ с пСриодичСским ΠΏΠΎΠ²Ρ‚ΠΎΡ€Π΅Π½ΠΈΠ΅ΠΌ двиТСния Π½Π° Π΄Π°Π½Π½ΠΎΠΉ ΠΎΡ€Π±ΠΈΡ‚Π΅, ΠΏΡ€ΠΈ этом

Если ΡΠΊΠΎΡ€ΠΎΡΡ‚ΡŒ частицы Π½Π΅ пСрпСндикулярна ΠΊ Π½Π°ΠΏΡ€Π°Π²Π»Π΅Π½ΠΈΡŽ поля, Ρ‚ΠΎ Π΅Π΅ Π΄Π²ΠΈΠΆΠ΅Π½ΠΈΠ΅ Π±ΡƒΠ΄Π΅Ρ‚ ΠΏΡ€ΠΎΠΈΡΡ…ΠΎΠ΄ΠΈΡ‚ΡŒ ΠΏΠΎ спирали, ΠΏΠ°Ρ€Π°ΠΌΠ΅Ρ‚Ρ€Ρ‹ ΠΊΠΎΡ‚ΠΎΡ€ΠΎΠΉ Π±ΡƒΠ΄ΡƒΡ‚ ΠΎΠΏΡ€Π΅Π΄Π΅Π»ΡΡ‚ΡŒΡΡ ΠΏΠΎΠΏΠ΅Ρ€Π΅Ρ‡Π½ΠΎΠΉ ΠΈ ΠΏΡ€ΠΎΠ΄ΠΎΠ»ΡŒΠ½ΠΎΠΉ ΡΠΎΡΡ‚Π°Π²Π»ΡΡŽΡ‰ΠΈΠΌΠΈ скорости:

Радиус Π²ΠΈΡ‚ΠΊΠΎΠ² спирали ΠΈ шаг ΠΌΠ΅ΠΆΠ΄Ρƒ Π²ΠΈΡ‚ΠΊΠ°ΠΌΠΈ ΠΎΠΏΡ€Π΅Π΄Π΅Π»ΡΡŽΡ‚ΡΡ ΠΏΡ€ΠΈ этом выраТСниями

Π’ случаС Π½Π΅ΠΎΠ΄Π½ΠΎΡ€ΠΎΠ΄Π½ΠΎΠ³ΠΎ ΠΌΠ°Π³Π½ΠΈΡ‚Π½ΠΎΠ³ΠΎ поля Π΄Π²ΠΈΠΆΠ΅Π½ΠΈΠ΅ заряТСнной частицы сущСствСнно услоТняСтся. Π‘ΠΏΠΈΡ€Π°Π»ΡŒΠ½Π°Ρ траСктория частицы устрСмляСтся Π² ΠΎΠ±Π»Π°ΡΡ‚ΡŒ сильного поля с ΡƒΠΌΠ΅Π½ΡŒΡˆΠ΅Π½ΠΈΠ΅ΠΌ радиуса Π²ΠΈΡ‚ΠΊΠ° Π½Π° ΠΊΠ°ΠΆΠ΄ΠΎΠΌ ΠΎΠ±ΠΎΡ€ΠΎΡ‚Π΅. По достиТСнии Π² сильном ΠΏΠΎΠ»Π΅ Ρ‚Π°ΠΊΠΎΠΉ Ρ‚ΠΎΡ‡ΠΊΠΈ, Π³Π΄Π΅ ΠΏΡ€ΠΎΠ΄ΠΎΠ»ΡŒΠ½Π°Ρ ΡΠΎΡΡ‚Π°Π²Π»ΡΡŽΡ‰Π°Ρ скорости обращаСтся Π² Π½ΡƒΠ»ΡŒ ΠΈ мСняСт Π·Π½Π°ΠΊ Π½Π° ΠΏΡ€ΠΎΡ‚ΠΈΠ²ΠΎΠΏΠΎΠ»ΠΎΠΆΠ½Ρ‹ΠΉ, частица Π½Π°Ρ‡ΠΈΠ½Π°Π΅Ρ‚ Π΄Π²ΠΈΠ³Π°Ρ‚ΡŒΡΡ Π² ΠΎΠ±Ρ€Π°Ρ‚Π½ΡƒΡŽ сторону ΡƒΠΆΠ΅ ΠΏΠΎ Ρ€Π°ΡΡˆΠΈΡ€ΡΡŽΡ‰Π΅ΠΉΡΡ спирали. ΠŸΡ€ΠΎΠΈΡΡ…ΠΎΠ΄ΠΈΡ‚ своСобразноС ΠΎΡ‚Ρ€Π°ΠΆΠ΅Π½ΠΈΠ΅ заряТСнной частицы ΡΠΈΠ»ΡŒΠ½Ρ‹ΠΌ Π½Π΅ΠΎΠ΄Π½ΠΎΡ€ΠΎΠ΄Π½Ρ‹ΠΌ ΠΌΠ°Π³Π½ΠΈΡ‚Π½Ρ‹ΠΌ ΠΏΠΎΠ»Π΅ΠΌ. Если Ρ‚Π°ΠΊΠΈΠ΅ ΠΎΡ‚Ρ€Π°ΠΆΠ°Ρ‚Π΅Π»ΡŒΠ½Ρ‹Π΅ поля ΠΎΠ±Ρ€Π°Π·ΠΎΠ²Π°Ρ‚ΡŒ с Π΄Π²ΡƒΡ… сторон, Ρ‚ΠΎ частица окаТСтся Π² ΠΌΠ°Π³Π½ΠΈΡ‚Π½ΠΎΠΉ Π»ΠΎΠ²ΡƒΡˆΠΊΠ΅, ΠΊΠΎΡ‚ΠΎΡ€ΡƒΡŽ ΠΎΠ½Π° Π½Π΅ смоТСт ΠΏΠΎΠΊΠΈΠ½ΡƒΡ‚ΡŒ.

2.9.5 Π”Π²ΠΈΠΆΠ΅Π½ΠΈΠ΅ свободной заряТСнной частицы Π² постоянном элСктромагнитном ΠΏΠΎΠ»Π΅

☻ ПолС считаСтся элСктромагнитным ΠΈ постоянным, Ссли Π² ΠΊΠ°ΠΆΠ΄ΠΎΠΉ Ρ‚ΠΎΡ‡ΠΊΠ΅ Π΅Π³ΠΎ пространства ΠΎΠ΄Π½ΠΎΠ²Ρ€Π΅ΠΌΠ΅Π½Π½ΠΎ Π΅ΡΡ‚ΡŒ Π΄Π²Π° поля – элСктричСскоС ΠΈ ΠΌΠ°Π³Π½ΠΈΡ‚Π½ΠΎΠ΅, ΠΏΡ€ΠΈΡ‡Π΅ΠΌ ΠΊΠ°ΠΆΠ΄ΠΎΠ΅ ΠΈΠ· Π½ΠΈΡ… постоянно. Частица вСщСства, ΠΎΠ±Π»Π°Π΄Π°ΡŽΡ‰Π°Ρ элСктричСским зарядомq, Π±ΡƒΠ΄Π΅Ρ‚ ΠΈΡΠΏΡ‹Ρ‚Ρ‹Π²Π°Ρ‚ΡŒ Π² Ρ‚Π°ΠΊΠΈΡ… условиях силовоС дСйствиС ΠΊΠ°ΠΊ со стороны элСктричСского, Ρ‚Π°ΠΊ ΠΈ со стороны ΠΌΠ°Π³Π½ΠΈΡ‚Π½ΠΎΠ³ΠΎ поля. Π Π΅Π·ΡƒΠ»ΡŒΡ‚ΠΈΡ€ΡƒΡŽΡ‰Π΅Π΅ дСйствиС Π½Π° частицу опрСдСляСтся ΠΏΡ€ΠΈ этом ΠΎΠ±ΠΎΠ±Ρ‰Π΅Π½Π½ΠΎΠΉ силой Π›ΠΎΡ€Π΅Π½Ρ†Π°:

Π³Π΄Π΅ ΠΈ- это элСктричСская ΠΈ магнитная ΡΠΎΡΡ‚Π°Π²Π»ΡΡŽΡ‰ΠΈΠ΅ этой силы.

Π”Π²ΠΈΠΆΠ΅Π½ΠΈΠ΅ частицы Π±ΡƒΠ΄Π΅Ρ‚ ΠΏΡ€ΠΎΠΈΡΡ…ΠΎΠ΄ΠΈΡ‚ΡŒ ΠΏΠΎ Π·Π°ΠΊΠΎΠ½Ρƒ , ΠΈΠ· ΠΊΠΎΡ‚ΠΎΡ€ΠΎΠ³ΠΎ слСдуСт

ΠΈΠ»ΠΈ Ρ‡Ρ‚ΠΎ Ρ‚ΠΎΠΆΠ΅

Из Ρ€Π΅ΡˆΠ΅Π½ΠΈΡ Π΄Π°Π½Π½ΠΎΠ³ΠΎ уравнСния ΠΏΡ€ΠΈ Π·Π°Π΄Π°Π½Π½Ρ‹Ρ… Π½Π°Ρ‡Π°Π»ΡŒΠ½Ρ‹Ρ… условиях находятся ΡΠΊΠΎΡ€ΠΎΡΡ‚ΡŒ, ускорСниС ΠΈ траСктория двиТСния частицы Поляив стационарных условиях ΠΌΠΎΠ³ΡƒΡ‚ Π±Ρ‹Ρ‚ΡŒ нСзависимыми ΠΈ управляСмыми ΠΊΠ°ΠΊ ΠΏΠΎ ΠΌΠΎΠ΄ΡƒΠ»ΡŽ, Ρ‚Π°ΠΊ ΠΈ ΠΏΠΎ Π²Π·Π°ΠΈΠΌΠ½ΠΎΠΉ ΠΎΡ€ΠΈΠ΅Π½Ρ‚Π°Ρ†ΠΈΠΈ. ΠŸΡ€ΠΈ Ρ‚Π°ΠΊΠΈΡ… условиях ΠΌΠΎΠΆΠ½ΠΎ ΡƒΠΏΡ€Π°Π²Π»ΡΡ‚ΡŒ Ρ‚Ρ€Π°Π΅ΠΊΡ‚ΠΎΡ€ΠΈΠ΅ΠΉ частицы

Π”Π²ΠΈΠΆΠ΅Π½ΠΈΠ΅ заряТСнной частицы Π² элСктричСском ΠΈ ΠΌΠ°Π³Π½ΠΈΡ‚Π½ΠΎΠΌ полях.

Β 

Если частица, ΠΎΠ±Π»Π°Π΄Π°ΡŽΡ‰Π°Ρ зарядом Π΅, двиТСтся Π² пространствС, Π³Π΄Π΅ имССтся элСктричСскоС ΠΏΠΎΠ»Π΅ с Π½Π°ΠΏΡ€ΡΠΆΡ‘Π½Π½ΠΎΡΡ‚ΡŒΡŽ E Ρ‚ΠΎ Π½Π° Π½Π΅Ρ‘ дСйствуСт сила eE. Если, ΠΊΡ€ΠΎΠΌΠ΅ элСктричСского, имССтся ΠΌΠ°Π³Π½ΠΈΡ‚Π½ΠΎΠ΅ ΠΏΠΎΠ»Π΅, Ρ‚ΠΎ Π½Π° частицу дСйствуСт Π΅Ρ‰Ρ‘ сила Π›ΠΎΡ€Π΅Π½Ρ†Π°, равная e[uB] , Π³Π΄Π΅ u — ΡΠΊΠΎΡ€ΠΎΡΡ‚ΡŒ двиТСния частицы ΠΎΡ‚Π½ΠΎΡΠΈΡ‚Π΅Π»ΡŒΠ½ΠΎ поля, B — магнитная индукция. ΠŸΠΎΡΡ‚ΠΎΠΌΡƒ согласно Π²Ρ‚ΠΎΡ€ΠΎΠΌΡƒ Π·Π°ΠΊΠΎΠ½Ρƒ ΠΡŒΡŽΡ‚ΠΎΠ½Π° ΡƒΡ€Π°Π²Π½Π΅Π½ΠΈΠ΅ двиТСния частиц ΠΈΠΌΠ΅Π΅Ρ‚ Π²ΠΈΠ΄:

НаписанноС Π²Π΅ΠΊΡ‚ΠΎΡ€Π½ΠΎΠ΅ ΡƒΡ€Π°Π²Π½Π΅Π½ΠΈΠ΅ распадаСтся Π½Π° Ρ‚Ρ€ΠΈ скалярных уравнСния, ΠΊΠ°ΠΆΠ΄ΠΎΠ΅ ΠΈΠ· ΠΊΠΎΡ‚ΠΎΡ€Ρ‹Ρ… описываСт Π΄Π²ΠΈΠΆΠ΅Π½ΠΈΠ΅ вдоль ΡΠΎΠΎΡ‚Π²Π΅Ρ‚ΡΡ‚Π²ΡƒΡŽΡ‰Π΅ΠΉ ΠΊΠΎΠΎΡ€Π΄ΠΈΠ½Π°Ρ‚Π½ΠΎΠΉ оси.

ΠŸΡ€Π΅Π΄ΠΏΠΎΠ»ΠΎΠΆΠΈΠΌ, Ρ‡Ρ‚ΠΎ заряТСнныС частицы, двигавшиСся ΠΏΠ΅Ρ€Π²ΠΎΠ½Π°Ρ‡Π°Π»ΡŒΠ½ΠΎ вдоль оси Π₯ со ΡΠΊΠΎΡ€ΠΎΡΡ‚ΡŒΡŽ ΠΏΠΎΠΏΠ°Π΄Π°ΡŽΡ‚ Π² элСктричСскоС ΠΏΠΎΠ»Π΅ плоского кондСнсатора.

Если Π·Π°Π·ΠΎΡ€ ΠΌΠ΅ΠΆΠ΄Ρƒ пластинами ΠΌΠ°Π» ΠΏΠΎ ΡΡ€Π°Π²Π½Π΅Π½ΠΈΡŽ с ΠΈΡ… Π΄Π»ΠΈΠ½ΠΎΠΉ, Ρ‚ΠΎ ΠΊΡ€Π°Π΅Π²Ρ‹ΠΌΠΈ эффСктами ΠΌΠΎΠΆΠ½ΠΎ ΠΏΡ€Π΅Π½Π΅Π±Ρ€Π΅Ρ‡ΡŒ ΠΈ ΡΡ‡ΠΈΡ‚Π°Ρ‚ΡŒ элСктричСскоС ΠΏΠΎΠ»Π΅ ΠΌΠ΅ΠΆΠ΄Ρƒ пластинами ΠΎΠ΄Π½ΠΎΡ€ΠΎΠ΄Π½Ρ‹ΠΌ. Направляя ось Y ΠΏΠ°Ρ€Π°Π»Π»Π΅Π»ΡŒΠ½ΠΎ полю, ΠΌΡ‹ ΠΈΠΌΠ΅Π΅ΠΌ: . Π’Π°ΠΊ ΠΊΠ°ΠΊ ΠΌΠ°Π³Π½ΠΈΡ‚Π½ΠΎΠ³ΠΎ поля Π½Π΅Ρ‚, Ρ‚ΠΎ . Π’ рассматриваСмом случаС Π½Π° заряТСнныС частицы дСйствуСт Ρ‚ΠΎΠ»ΡŒΠΊΠΎ сила со стороны элСктричСского поля, которая ΠΏΡ€ΠΈ Π²Ρ‹Π±Ρ€Π°Π½Π½ΠΎΠΌ Π½Π°ΠΏΡ€Π°Π²Π»Π΅Π½ΠΈΠΈ ΠΊΠΎΠΎΡ€Π΄ΠΈΠ½Π°Ρ‚Π½Ρ‹Ρ… осСй Ρ†Π΅Π»ΠΈΠΊΠΎΠΌ Π½Π°ΠΏΡ€Π°Π²Π»Π΅Π½Π° ΠΏΠΎ оси Y. ΠŸΠΎΡΡ‚ΠΎΠΌΡƒ траСктория двиТСния частиц Π»Π΅ΠΆΠΈΡ‚ Π² плоскости XY ΠΈ уравнСния двиТСния ΠΏΡ€ΠΈΠ½ΠΈΠΌΠ°ΡŽΡ‚ Π²ΠΈΠ΄:

Π”Π²ΠΈΠΆΠ΅Π½ΠΈΠ΅ частиц Π² этом случаС происходит ΠΏΠΎΠ΄ дСйствиСм постоянной силы ΠΈ ΠΏΠΎΠ΄ΠΎΠ±Π½ΠΎ двиТСнию Π³ΠΎΡ€ΠΈΠ·ΠΎΠ½Ρ‚Π°Π»ΡŒΠ½ΠΎ Π±Ρ€ΠΎΡˆΠ΅Π½Π½ΠΎΠ³ΠΎ Ρ‚Π΅Π»Π° Π² ΠΏΠΎΠ»Π΅ тяТСсти. ΠŸΠΎΡΡ‚ΠΎΠΌΡƒ ясно Π±Π΅Π· Π΄Π°Π»ΡŒΠ½Π΅ΠΉΡˆΠΈΡ… расчСтов, Ρ‡Ρ‚ΠΎ частицы Π±ΡƒΠ΄ΡƒΡ‚ Π΄Π²ΠΈΠ³Π°Ρ‚ΡŒΡΡ ΠΏΠΎ ΠΏΠ°Ρ€Π°Π±ΠΎΠ»Π°ΠΌ.

Вычислим ΡƒΠ³ΠΎΠ» , Π½Π° ΠΊΠΎΡ‚ΠΎΡ€Ρ‹ΠΉ отклонится ΠΏΡƒΡ‡ΠΎΠΊ частиц послС прохоТдСния Ρ‡Π΅Ρ€Π΅Π· кондСнсатор. Π˜Π½Ρ‚Π΅Π³Ρ€ΠΈΡ€ΡƒΡ ΠΏΠ΅Ρ€Π²ΠΎΠ΅ ΠΈΠ· ΡƒΡ€Π°Π²Π½Π΅Π½ΠΈΠΉ, Π½Π°Ρ…ΠΎΠ΄ΠΈΠΌ:

Π˜Π½Ρ‚Π΅Π³Ρ€Π°Ρ†ΠΈΡ Π²Ρ‚ΠΎΡ€ΠΎΠ³ΠΎ уравнСния Π΄Π°Ρ‘Ρ‚:

Π’Π°ΠΊ ΠΊΠ°ΠΊ ΠΏΡ€ΠΈ t=0 (ΠΌΠΎΠΌΠ΅Π½Ρ‚ вступлСния частицы Π² кондСнсатор) u(y)=0, Ρ‚ΠΎ c=0, ΠΈ поэтому

ΠžΡ‚ΡΡŽΠ΄Π° ΠΏΠΎΠ»ΡƒΡ‡Π°Π΅ΠΌ для ΡƒΠ³Π»Π° отклонСния:

Π­Ρ„Ρ„Π΅ΠΊΡ‚ Π₯ΠΎΠ»Π»Π° β€” явлСниС возникновСния ΠΏΠΎΠΏΠ΅Ρ€Π΅Ρ‡Π½ΠΎΠΉ разности ΠΏΠΎΡ‚Π΅Π½Ρ†ΠΈΠ°Π»ΠΎΠ² (Π½Π°Π·Ρ‹Π²Π°Π΅ΠΌΠΎΠΉ Ρ‚Π°ΠΊΠΆΠ΅ Π₯олловским напряТСниСм) ΠΏΡ€ΠΈ ΠΏΠΎΠΌΠ΅Ρ‰Π΅Π½ΠΈΠΈ ΠΏΡ€ΠΎΠ²ΠΎΠ΄Π½ΠΈΠΊΠ° с постоянным Ρ‚ΠΎΠΊΠΎΠΌ Π² ΠΌΠ°Π³Π½ΠΈΡ‚Π½ΠΎΠ΅ ΠΏΠΎΠ»Π΅.

Π’ ΠΏΡ€ΠΎΡΡ‚Π΅ΠΉΡˆΠ΅ΠΌ рассмотрСнии эффСкт Π₯ΠΎΠ»Π»Π° выглядит ΡΠ»Π΅Π΄ΡƒΡŽΡ‰ΠΈΠΌ ΠΎΠ±Ρ€Π°Π·ΠΎΠΌ. ΠŸΡƒΡΡ‚ΡŒ Ρ‡Π΅Ρ€Π΅Π· мСталличСский брус Π² слабом ΠΌΠ°Π³Π½ΠΈΡ‚Π½ΠΎΠΌ ΠΏΠΎΠ»Π΅ B Ρ‚Π΅Ρ‡Ρ‘Ρ‚ элСктричСский Ρ‚ΠΎΠΊ ΠΏΠΎΠ΄ дСйствиСм напряТённости E. ΠœΠ°Π³Π½ΠΈΡ‚Π½ΠΎΠ΅ ΠΏΠΎΠ»Π΅ Π±ΡƒΠ΄Π΅Ρ‚ ΠΎΡ‚ΠΊΠ»ΠΎΠ½ΡΡ‚ΡŒΠ½ΠΎΡΠΈΡ‚Π΅Π»ΠΈ заряда (для опрСдСлённости элСктроны) ΠΎΡ‚ ΠΈΡ… двиТСния вдоль ΠΈΠ»ΠΈ ΠΏΡ€ΠΎΡ‚ΠΈΠ² элСктричСского поля ΠΊ ΠΎΠ΄Π½ΠΎΠΉ ΠΈΠ· Π³Ρ€Π°Π½Π΅ΠΉ бруса. ΠŸΡ€ΠΈ этом ΠΊΡ€ΠΈΡ‚Π΅Ρ€ΠΈΠ΅ΠΌ малости Π±ΡƒΠ΄Π΅Ρ‚ ΡΠ»ΡƒΠΆΠΈΡ‚ΡŒ условиС, Ρ‡Ρ‚ΠΎ ΠΏΡ€ΠΈ этом элСктрон Π½Π΅ Π½Π°Ρ‡Π½Ρ‘Ρ‚ Π΄Π²ΠΈΠ³Π°Ρ‚ΡŒΡΡ ΠΏΠΎ спирали.



Π’Π°ΠΊΠΈΠΌ ΠΎΠ±Ρ€Π°Π·ΠΎΠΌ, сила Π›ΠΎΡ€Π΅Π½Ρ†Π° ΠΏΡ€ΠΈΠ²Π΅Π΄Ρ‘Ρ‚ ΠΊ накоплСнию ΠΎΡ‚Ρ€ΠΈΡ†Π°Ρ‚Π΅Π»ΡŒΠ½ΠΎΠ³ΠΎ заряда Π²ΠΎΠ·Π»Π΅ ΠΎΠ΄Π½ΠΎΠΉ Π³Ρ€Π°Π½ΠΈ бруска ΠΈ ΠΏΠΎΠ»ΠΎΠΆΠΈΡ‚Π΅Π»ΡŒΠ½ΠΎΠ³ΠΎ Π²ΠΎΠ·Π»Π΅ ΠΏΡ€ΠΎΡ‚ΠΈΠ²ΠΎΠΏΠΎΠ»ΠΎΠΆΠ½ΠΎΠΉ. НакоплСниС заряда Π±ΡƒΠ΄Π΅Ρ‚ ΠΏΡ€ΠΎΠ΄ΠΎΠ»ΠΆΠ°Ρ‚ΡŒΡΡ Π΄ΠΎ Ρ‚Π΅Ρ… ΠΏΠΎΡ€, ΠΏΠΎΠΊΠ° возникшСС элСктричСскоС ΠΏΠΎΠ»Π΅ зарядов E1 Π½Π΅ скомпСнсируСт ΠΌΠ°Π³Π½ΠΈΡ‚Π½ΡƒΡŽ ΡΠΎΡΡ‚Π°Π²Π»ΡΡŽΡ‰ΡƒΡŽ силы Π›ΠΎΡ€Π΅Π½Ρ†Π°:

Π‘ΠΊΠΎΡ€ΠΎΡΡ‚ΡŒ элСктронов v ΠΌΠΎΠΆΠ½ΠΎ Π²Ρ‹Ρ€Π°Π·ΠΈΡ‚ΡŒ Ρ‡Π΅Ρ€Π΅Π· ΠΏΠ»ΠΎΡ‚Π½ΠΎΡΡ‚ΡŒ Ρ‚ΠΎΠΊΠ°:

Π³Π΄Π΅ n β€” концСнтрация носитСлСй заряда. Π’ΠΎΠ³Π΄Π°

ΠšΠΎΡΡ„Ρ„ΠΈΡ†ΠΈΠ΅Π½Ρ‚ ΠΏΡ€ΠΎΠΏΠΎΡ€Ρ†ΠΈΠΎΠ½Π°Π»ΡŒΠ½ΠΎΡΡ‚ΠΈ ΠΌΠ΅ΠΆΠ΄Ρƒ E1 ΠΈ jB называСтся коэффициСнтом (константой) Π₯ΠΎΠ»Π»Π°. Π’ Ρ‚Π°ΠΊΠΎΠΌ ΠΏΡ€ΠΈΠ±Π»ΠΈΠΆΠ΅Π½ΠΈΠΈ Π·Π½Π°ΠΊ постоянной Π₯ΠΎΠ»Π»Π° зависит ΠΎΡ‚ Π·Π½Π°ΠΊΠ° носитСлСй заряда, Ρ‡Ρ‚ΠΎ позволяСт ΠΎΠΏΡ€Π΅Π΄Π΅Π»ΡΡ‚ΡŒ ΠΈΡ… Ρ‚ΠΈΠΏ для большого числа ΠΌΠ΅Ρ‚Π°Π»Π»ΠΎΠ². Для Π½Π΅ΠΊΠΎΡ‚ΠΎΡ€Ρ‹Ρ… ΠΌΠ΅Ρ‚Π°Π»Π»ΠΎΠ² (Π² ΡΠΈΠ»ΡŒΠ½Ρ‹Ρ… полях), Ρ‚Π°ΠΊΠΈΡ… ΠΊΠ°ΠΊ алюминий, Ρ†ΠΈΠ½ΠΊ, ΠΆΠ΅Π»Π΅Π·ΠΎ, ΠΊΠΎΠ±Π°Π»ΡŒΡ‚, Π½Π°Π±Π»ΡŽΠ΄Π°Π΅Ρ‚ΡΡ ΠΏΠΎΠ»ΠΎΠΆΠΈΡ‚Π΅Π»ΡŒΠ½Ρ‹ΠΉ Π·Π½Π°ΠΊ RH, Ρ‡Ρ‚ΠΎ ΠΎΠ±ΡŠΡΡΠ½ΡΠ΅Ρ‚ΡΡ Π² полуклассичСской ΠΈ ΠΊΠ²Π°Π½Ρ‚ΠΎΠ²ΠΎΠΉ тСориях Ρ‚Π²Ρ‘Ρ€Π΄ΠΎΠ³ΠΎ Ρ‚Π΅Π»Π°.

Масс-спСктромСтрия (масс-спСктроскопия, масс-спСктрография, масс-ΡΠΏΠ΅ΠΊΡ‚Ρ€Π°Π»ΡŒΠ½Ρ‹ΠΉ Π°Π½Π°Π»ΠΈΠ·, масс-спСктромСтричСский Π°Π½Π°Π»ΠΈΠ·) β€” ΠΌΠ΅Ρ‚ΠΎΠ΄ исслСдования вСщСства ΠΏΡƒΡ‚Ρ‘ΠΌ опрСдСлСния ΠΎΡ‚Π½ΠΎΡˆΠ΅Π½ΠΈΡ массы ΠΊ заряду (качСства) ΠΈ количСствазаряТСнных частиц, ΠΎΠ±Ρ€Π°Π·ΡƒΡŽΡ‰ΠΈΡ…ΡΡ ΠΏΡ€ΠΈ Ρ‚ΠΎΠΌ ΠΈΠ»ΠΈ ΠΈΠ½ΠΎΠΌ процСссС воздСйствия Π½Π° вСщСство (см. ΠΌΠ΅Ρ‚ΠΎΠ΄Ρ‹ ΠΈΠΎΠ½ΠΈΠ·Π°Ρ†ΠΈΠΈ). Масс-спСктромСтр β€” это Π²Π°ΠΊΡƒΡƒΠΌΠ½Ρ‹ΠΉ ΠΏΡ€ΠΈΠ±ΠΎΡ€, ΠΈΡΠΏΠΎΠ»ΡŒΠ·ΡƒΡŽΡ‰ΠΈΠΉ физичСскиС Π·Π°ΠΊΠΎΠ½Ρ‹ двиТСния заряТСнных частиц Π² ΠΌΠ°Π³Π½ΠΈΡ‚Π½Ρ‹Ρ… ΠΈ элСктричСских полях, ΠΈ Π½Π΅ΠΎΠ±Ρ…ΠΎΠ΄ΠΈΠΌΡ‹ΠΉ для получСния масс-спСктра.

Π£ΡΠΊΠΎΡ€ΠΈΡ‚Π΅Π»ΡŒ заряТСнных частиц β€” класс устройств для получСния заряТСнных частиц (элСмСнтарных частиц, ΠΈΠΎΠ½ΠΎΠ²) высоких энСргий.

Π’ основС Ρ€Π°Π±ΠΎΡ‚Ρ‹ ускоритСля Π·Π°Π»ΠΎΠΆΠ΅Π½ΠΎ взаимодСйствиС заряТСнных частиц с элСктричСским ΠΈ ΠΌΠ°Π³Π½ΠΈΡ‚Π½Ρ‹ΠΌ полями. ЭлСктричСскоС ΠΏΠΎΠ»Π΅ способно Π½Π°ΠΏΡ€ΡΠΌΡƒΡŽ ΡΠΎΠ²Π΅Ρ€ΡˆΠ°Ρ‚ΡŒ Ρ€Π°Π±ΠΎΡ‚Ρƒ Π½Π°Π΄ частицСй, Ρ‚ΠΎ Π΅ΡΡ‚ΡŒ ΡƒΠ²Π΅Π»ΠΈΡ‡ΠΈΠ²Π°Ρ‚ΡŒ Π΅Ρ‘ ΡΠ½Π΅Ρ€Π³ΠΈΡŽ. ΠœΠ°Π³Π½ΠΈΡ‚Π½ΠΎΠ΅ ΠΆΠ΅ ΠΏΠΎΠ»Π΅, создавая силу Π›ΠΎΡ€Π΅Π½Ρ†Π°, лишь отклоняСт частицу, Π½Π΅ измСняя Π΅Ρ‘ энСргии, ΠΈ Π·Π°Π΄Π°Ρ‘Ρ‚ ΠΎΡ€Π±ΠΈΡ‚Ρƒ, ΠΏΠΎ ΠΊΠΎΡ‚ΠΎΡ€ΠΎΠΉ двиТутся частицы.

УскоритСли ΠΌΠΎΠΆΠ½ΠΎ ΠΏΡ€ΠΈΠ½Ρ†ΠΈΠΏΠΈΠ°Π»ΡŒΠ½ΠΎ Ρ€Π°Π·Π΄Π΅Π»ΠΈΡ‚ΡŒ Π½Π° Π΄Π²Π΅ большиС Π³Ρ€ΡƒΠΏΠΏΡ‹. Π­Ρ‚ΠΎ Π»ΠΈΠ½Π΅ΠΉΠ½Ρ‹Π΅ ускоритСли, Π³Π΄Π΅ ΠΏΡƒΡ‡ΠΎΠΊ частиц ΠΎΠ΄Π½ΠΎΠΊΡ€Π°Ρ‚Π½ΠΎ ΠΏΡ€ΠΎΡ…ΠΎΠ΄ΠΈΡ‚ ΡƒΡΠΊΠΎΡ€ΡΡŽΡ‰ΠΈΠ΅ ΠΏΡ€ΠΎΠΌΠ΅ΠΆΡƒΡ‚ΠΊΠΈ, ΠΈ цикличСскиС ускоритСли, Π² ΠΊΠΎΡ‚ΠΎΡ€Ρ‹Ρ… ΠΏΡƒΡ‡ΠΊΠΈ двиТутся ΠΏΠΎ Π·Π°ΠΌΠΊΠ½ΡƒΡ‚Ρ‹ΠΌ ΠΊΡ€ΠΈΠ²Ρ‹ΠΌ Ρ‚ΠΈΠΏΠ° окруТностСй, проходя ΡƒΡΠΊΠΎΡ€ΡΡŽΡ‰ΠΈΠ΅ ΠΏΡ€ΠΎΠΌΠ΅ΠΆΡƒΡ‚ΠΊΠΈ ΠΏΠΎ ΠΌΠ½ΠΎΠ³Ρƒ Ρ€Π°Π·. МоТно Ρ‚Π°ΠΊΠΆΠ΅ ΠΊΠ»Π°ΡΡΠΈΡ„ΠΈΡ†ΠΈΡ€ΠΎΠ²Π°Ρ‚ΡŒ ускоритСли ΠΏΠΎ Π½Π°Π·Π½Π°Ρ‡Π΅Π½ΠΈΡŽ: ΠΊΠΎΠ»Π»Π°ΠΉΠ΄Π΅Ρ€Ρ‹, источники Π½Π΅ΠΉΡ‚Ρ€ΠΎΠ½ΠΎΠ², бустСры, источники синхротронного излучСния, установки для Ρ‚Π΅Ρ€Π°ΠΏΠΈΠΈ Ρ€Π°ΠΊΠ°, ΠΏΡ€ΠΎΠΌΡ‹ΡˆΠ»Π΅Π½Π½Ρ‹Π΅ ускоритСли.

Β 

Β 

ВлияСт Π»ΠΈ Π½Π° двиТущиСся заряды ΠΌΠ°Π³Π½ΠΈΡ‚Π½ΠΎΠ΅ ΠΏΠΎΠ»Π΅, ΠΊΠΎΡ‚ΠΎΡ€ΠΎΠ΅ ΠΎΠ½ΠΈ ΡΠΎΠ·Π΄Π°ΡŽΡ‚ ΠΏΡ€ΠΈ Π΄Π²ΠΈΠΆΠ΅Π½ΠΈΠΈ, создавая Ρ‚ΠΎΠΊ? Если Π½Π΅Ρ‚, Ρ‚ΠΎ ΠΊΠ°ΠΊ Π²ΠΎΠ·ΠΌΠΎΠΆΠ½Π° самоиндукция?

Бпросил

ИзмСнСно 6 Π»Π΅Ρ‚, 8 мСсяцСв Π½Π°Π·Π°Π΄

ΠŸΡ€ΠΎΡΠΌΠΎΡ‚Ρ€Π΅Π½ΠΎ 5ΠΊ Ρ€Π°Π·

$\begingroup$

ΠŸΡƒΡΡ‚ΡŒ сгусток зарядов двиТСтся с постоянной ΡΠΊΠΎΡ€ΠΎΡΡ‚ΡŒΡŽ $\mathbf v\;. $ ΠŸΠΎΡΠΊΠΎΠ»ΡŒΠΊΡƒ заряды двиТутся, ΠΎΠ½ΠΈ Π±ΡƒΠ΄ΡƒΡ‚ ΡΠΎΠ·Π΄Π°Π²Π°Ρ‚ΡŒ ΠΌΠ°Π³Π½ΠΈΡ‚Π½ΠΎΠ΅ ΠΏΠΎΠ»Π΅ $\bf B$, Ρ‚Π°ΠΊ ΠΊΠ°ΠΊ ΠΈΠΌΠ΅Π½Π½ΠΎ Ρ‚ΠΎΠΊ создаСт ΠΌΠ°Π³Π½ΠΈΡ‚Π½ΠΎΠ΅ ΠΏΠΎΠ»Π΅. 92} \;;$$, Ρ‚Π°ΠΊ ΠΊΠ°ΠΊ $d\mathbf l$ ΠΈ $\hat{\mathbf r}$ Π½Π°ΠΏΡ€Π°Π²Π»Π΅Π½Ρ‹ Π² ΠΎΠ΄Π½ΠΎΠΌ Π½Π°ΠΏΡ€Π°Π²Π»Π΅Π½ΠΈΠΈ, Ρ‡ΠΈΡΠ»ΠΈΡ‚Π΅Π»ΡŒ Ρ€Π°Π²Π΅Π½ Π½ΡƒΠ»ΡŽ, Π° Π² Π·Π½Π°ΠΌΠ΅Π½Π°Ρ‚Π΅Π»Π΅ $r$ Ρ€Π°Π²Π΅Π½ Π½ΡƒΠ»ΡŽ, Ρ‡Ρ‚ΠΎ снова Π΄Π΅Π»Π°Π΅Ρ‚ силу бСсконСчной . Π­Ρ‚ΠΎ Π½Π΅Π²ΠΎΠ·ΠΌΠΎΠΆΠ½ΠΎ.

Однако, ΠΊΠΎΠ³Π΄Π° Ρ‚ΠΎΠΊ нСпостоянСн, двиТущиСся заряды ΡΠΎΠ·Π΄Π°ΡŽΡ‚ зависящСС ΠΎΡ‚ Π²Ρ€Π΅ΠΌΠ΅Π½ΠΈ ΠΌΠ°Π³Π½ΠΈΡ‚Π½ΠΎΠ΅ ΠΏΠΎΠ»Π΅, Ρ‡Ρ‚ΠΎ ΠΏΡ€ΠΈΠ²ΠΎΠ΄ΠΈΡ‚ ΠΊ самоиндукции.

Π­Π”Π‘ самоиндукции, вызванная ΠΈΠ·ΠΌΠ΅Π½Π΅Π½ΠΈΠ΅ΠΌ ΠΏΠΎΡ‚ΠΎΠΊΠ° ΠΌΠ°Π³Π½ΠΈΡ‚Π½ΠΎΠ³ΠΎ поля, создаваСмого зарядами, дСйствуСт Π½Π° заряды; ΠΊΠ°ΠΆΠ΄Ρ‹ΠΉ заряд испытываСт силу $\mathbf F= q\; \mathbf v\times \mathbf B $, Π³Π΄Π΅ $\mathbf B$ создаСтся ΠΈΠΌΠΈ самими.

Π˜Ρ‚Π°ΠΊ, это ΠΎΠ·Π½Π°Ρ‡Π°Π΅Ρ‚, Ρ‡Ρ‚ΠΎ заряды двиТутся ΠΈ ΡΠΎΠ·Π΄Π°ΡŽΡ‚ ΠΌΠ°Π³Π½ΠΈΡ‚Π½ΠΎΠ΅ ΠΏΠΎΠ»Π΅, ΠΊΠΎΡ‚ΠΎΡ€ΠΎΠ΅ снова воздСйствуСт Π½Π° заряды, создавшиС ΠΏΠΎΠ»Π΅.

Π’ стационарном состоянии заряды Π½Π΅ ΠΏΠΎΠ΄Π²Π΅Ρ€ΠΆΠ΅Π½Ρ‹ влиянию собствСнного ΠΌΠ°Π³Π½ΠΈΡ‚Π½ΠΎΠ³ΠΎ поля, Ρ‚ΠΎΠ³Π΄Π° ΠΊΠ°ΠΊ, с Π΄Ρ€ΡƒΠ³ΠΎΠΉ стороны, Π² нСстационарном состоянии ΠΌΠ°Π³Π½ΠΈΡ‚Π½ΠΎΠ΅ ΠΏΠΎΠ»Π΅, создаваСмоС этими двиТущимися зарядами, ΠΏΡ€ΠΈΠ΄Π°Π΅Ρ‚ силу ΠΊΠ°ΠΆΠ΄ΠΎΠΌΡƒ заряду, Ρ‡Ρ‚ΠΎ ΠΏΡ€ΠΈΠ²ΠΎΠ΄ΠΈΡ‚ ΠΊ возникновСнию само- э. Π΄.с.

Но ΠΏΠΎΡ‡Π΅ΠΌΡƒ Ρ‚Π°ΠΊ?

Поля ΡΠ²Π»ΡΡŽΡ‚ΡΡ нСзависимыми ΠΎΠ±ΡŠΠ΅ΠΊΡ‚Π°ΠΌΠΈ; ΠΎΠ½ΠΈ ΠΌΠΎΠ³ΡƒΡ‚ Π΄Π΅Π»ΠΈΡ‚ΡŒΡΡ ΠΈΠΌΠΏΡƒΠ»ΡŒΡΠΎΠΌ ΠΈ энСргиСй с зарядами, ΠΊΠ°ΠΊ сказал Π’ΠΈΠΌΠ΅ΠΉ.

На заряды Π΄Π΅ΠΉΡΡ‚Π²ΠΈΡ‚Π΅Π»ΡŒΠ½ΠΎ влияСт ΠΈΡ… ΠΏΠΎΠ»Π΅, ΠΊΠΎΠ³Π΄Π° ΠΎΠ½ΠΈ Ρ‡ΠΈΡ‚Π°ΡŽΡ‚ΡΡ. Но, насколько я Ρ‡ΠΈΡ‚Π°Π», я Π½Π΅ ΠΎΠ±Π½Π°Ρ€ΡƒΠΆΠΈΠ», Ρ‡Ρ‚ΠΎ ΠΏΡ€ΠΈ нСстационарном состоянии Ρ‚ΠΎΠΊΠ° ΠΈΠ·Π»ΡƒΡ‡Π°ΡŽΡ‚ΡΡ заряды. Π’Π΅ΠΌ Π½Π΅ ΠΌΠ΅Π½Π΅Π΅ Π½Π° Π½ΠΈΡ… влияСт ΠΈΠ·ΠΌΠ΅Π½ΡΡŽΡ‰ΠΈΠΉΡΡ ΠΏΠΎΡ‚ΠΎΠΊ создаваСмого ΠΈΠΌΠΈ ΠΌΠ°Π³Π½ΠΈΡ‚Π½ΠΎΠ³ΠΎ поля. Π­Ρ‚ΠΎ создаСт ΡΠΎΠ±ΡΡ‚Π²Π΅Π½Π½ΡƒΡŽ Π­Π”Π‘; Ρ‡Ρ‚ΠΎ ΠΎΠ·Π½Π°Ρ‡Π°Π΅Ρ‚, Ρ‡Ρ‚ΠΎ ΠΌΠ°Π³Π½ΠΈΡ‚Π½ΠΎΠ΅ ΠΏΠΎΠ»Π΅ воздСйствуСт Π½Π° Ρ‚Π΅ двиТущиСся заряды, ΠΊΠΎΡ‚ΠΎΡ€Ρ‹Π΅ ΡΠΎΠ·Π΄Π°ΡŽΡ‚ ΠΏΠΎΠ»Π΅.

Однако, ΠΏΠΎΡ‡Π΅ΠΌΡƒ Π½Π° двиТущиСся заряды Π½Π΅ дСйствуСт ΠΈΡ… собствСнноС ΠΌΠ°Π³Π½ΠΈΡ‚Π½ΠΎΠ΅ ΠΏΠΎΠ»Π΅, ΠΊΠΎΠ³Π΄Π° Ρ‚ΠΎΠΊ находится Π² ΡƒΡΡ‚Π°Π½ΠΎΠ²ΠΈΠ²ΡˆΠ΅ΠΌΡΡ состоянии, Π½ΠΎ Π² нСстационарном состоянии Π½Π° заряды дСйствуСт сила ΠΌΠ°Π³Π½ΠΈΡ‚Π½ΠΎΠ³ΠΎ поля, ΠΊΠΎΡ‚ΠΎΡ€ΠΎΠ΅ ΠΎΠ½ΠΈ ΡΠΎΠ·Π΄Π°ΡŽΡ‚?

Если Π½Π° заряды Π΄Π΅ΠΉΡΡ‚Π²ΠΈΡ‚Π΅Π»ΡŒΠ½ΠΎ влияСт ΠΌΠ°Π³Π½ΠΈΡ‚Π½ΠΎΠ΅ ΠΏΠΎΠ»Π΅, ΠΊΠΎΡ‚ΠΎΡ€ΠΎΠ΅ ΠΎΠ½ΠΈ ΡΠΎΠ·Π΄Π°ΡŽΡ‚, Π΅ΡΡ‚ΡŒ Π»ΠΈ способ матСматичСски Π΄ΠΎΠΊΠ°Π·Π°Ρ‚ΡŒ, Ρ‡Ρ‚ΠΎ Π²ΠΎ врСмя самоиндукции двиТущиСся заряды ΠΏΡ€ΠΈΠ½ΡƒΠΆΠ΄Π°ΡŽΡ‚ΡΡ создаваСмым ΠΈΠΌΠΈ ΠΌΠ°Π³Π½ΠΈΡ‚Π½Ρ‹ΠΌ ΠΏΠΎΠ»Π΅ΠΌ?

Π― совсСм запутался; я Ρ‡Ρ‚ΠΎ-Ρ‚ΠΎ пропустил здСсь? ΠŸΠΎΠΆΠ°Π»ΡƒΠΉΡΡ‚Π° ΠΏΠΎΠΌΠΎΠ³ΠΈ.

  • ΠΌΠ°Π³Π½ΠΈΡ‚Π½Ρ‹Π΅ поля
  • элСктромагнитныС ΠΈΠ½Π΄ΡƒΠΊΡ†ΠΈΠΎΠ½Π½Ρ‹Π΅

$\endgroup$

12

$\begingroup$

Ни уравнСния ΠšΡƒΠ»ΠΎΠ½Π°, Π½ΠΈ уравнСния Π‘ΠΈΠΎ-Π‘Π°Π²Π°Ρ€Π° Π½Π΅ ΡΠ²Π»ΡΡŽΡ‚ΡΡ ΠΏΡ€Π°Π²ΠΈΠ»ΡŒΠ½Ρ‹ΠΌΠΈ уравнСниями для элСктромагнитного поля, Π·Π° ΠΈΡΠΊΠ»ΡŽΡ‡Π΅Π½ΠΈΠ΅ΠΌ статики. Π‘ΡƒΡ‰Π΅ΡΡ‚Π²ΡƒΡŽΡ‚ зависящиС ΠΎΡ‚ Π²Ρ€Π΅ΠΌΠ΅Π½ΠΈ обобщСния, Ρ‚Π°ΠΊΠΈΠ΅ ΠΊΠ°ΠΊ 93\vec r’$$ Π³Π΄Π΅ $t_r$ Π½Π° самом Π΄Π΅Π»Π΅ являСтся Ρ„ΡƒΠ½ΠΊΡ†ΠΈΠ΅ΠΉ $\vec r’$, Π° ΠΈΠΌΠ΅Π½Π½ΠΎ $t_r=t-\frac{|\vec r-\vec r’|}{c}.$

Они сводятся ΠΊ ΠšΡƒΠ»ΠΎΠ½Ρƒ ΠΈ Π‘ΠΈΠΎ- Π‘Π°Π²Π°Ρ€Π° Ρ‚ΠΎΠ»ΡŒΠΊΠΎ Ρ‚ΠΎΠ³Π΄Π°, ΠΊΠΎΠ³Π΄Π° эти ΠΏΡ€ΠΎΠΈΠ·Π²ΠΎΠ΄Π½Ρ‹Π΅ ΠΏΠΎ Π²Ρ€Π΅ΠΌΠ΅Π½ΠΈ Ρ‚ΠΎΡ‡Π½ΠΎ Ρ€Π°Π²Π½Ρ‹ Π½ΡƒΠ»ΡŽ, Ρ‡Ρ‚ΠΎ являСтся статикой. Π’Π°ΠΊΠΈΠΌ ΠΎΠ±Ρ€Π°Π·ΠΎΠΌ, Π•Ρ„ΠΈΠΌΠ΅Π½ΠΊΠΎ являСтся ΠΏΡ€ΠΈΠΌΠ΅Ρ€ΠΎΠΌ собствСнных Π²Ρ€Π΅ΠΌΠ΅Π½Π½Ρ‹Ρ… Π·Π°ΠΊΠΎΠ½ΠΎΠ² для элСктромагнитного поля. ΠžΠ±Ρ€Π°Ρ‚ΠΈΡ‚Π΅ Π²Π½ΠΈΠΌΠ°Π½ΠΈΠ΅, Ρ‡Ρ‚ΠΎ ΠΈ элСктричСская, ΠΈ магнитная части элСктромагнитного поля ΠΈΠΌΠ΅ΡŽΡ‚ части, ΠΊΠΎΡ‚ΠΎΡ€Ρ‹Π΅ зависят ΠΎΡ‚ измСнСния Ρ‚ΠΎΠΊΠ° Π²ΠΎ Π²Ρ€Π΅ΠΌΠ΅Π½ΠΈ. Π—Π°ΠΊΠΎΠ½ ЀарадСя связываСт ΠΈΡ… вмСстС.

И ΡΠ»Π΅Π΄ΡƒΡŽΡ‰ΠΈΠΉ Ρ„Π°ΠΊΡ‚ Ρ‚Π°ΠΊΠΆΠ΅ являСтся ΠΊΠ»ΡŽΡ‡Π΅Π²Ρ‹ΠΌ: Π­Π”Π‘ Π²ΠΎΠΊΡ€ΡƒΠ³ Π½Π΅ΠΏΠΎΠ΄Π²ΠΈΠΆΠ½ΠΎΠ³ΠΎ ΠΏΡ€ΠΎΠ²ΠΎΠ΄Π° составляСт 100% ΠΈΠ·-Π·Π° элСктричСского поля. И Ссли Ρƒ вас Π΅ΡΡ‚ΡŒ всСлСнная с Π½Π΅ΠΉΡ‚Ρ€Π°Π»ΡŒΠ½Ρ‹ΠΌΠΈ ΠΎΠ±ΡŠΠ΅ΠΊΡ‚Π°ΠΌΠΈ ΠΏΠΎΠ²ΡΡŽΠ΄Ρƒ, Ρ‚ΠΎ Π•Ρ„ΠΈΠΌΠ΅Π½ΠΊΠΎ прСдсказываСт, Ρ‡Ρ‚ΠΎ элСктричСскоС ΠΏΠΎΠ»Π΅ ΠΈΡΠΊΠ»ΡŽΡ‡ΠΈΡ‚Π΅Π»ΡŒΠ½ΠΎ ΠΈ Π½Π° 100% Π²Ρ‹Π·Π²Π°Π½ΠΎ Π²Ρ€Π΅ΠΌΠ΅Π½Π½Ρ‹ΠΌΠΈ вариациями Ρ‚ΠΎΠΊΠ°. И я Π΄Π΅ΠΉΡΡ‚Π²ΠΈΡ‚Π΅Π»ΡŒΠ½ΠΎ имСю Π² Π²ΠΈΠ΄Ρƒ ΠΏΡ€ΠΈΡ‡ΠΈΠ½Ρƒ, ΠΊΠ°ΠΊ Π² ΠΏΡ€ΠΈΡ‡ΠΈΠ½Π΅ ΠΈ слСдствии. УравнСния ΡΠ²Π»ΡΡŽΡ‚ΡΡ ΠΏΡ€ΠΈΡ‡ΠΈΠ½Π½ΠΎ-слСдствСнными, ΠΏΠΎΡΠΊΠΎΠ»ΡŒΠΊΡƒ заряд (ΠΈ ΠΈΠ·ΠΌΠ΅Π½Π΅Π½ΠΈΠ΅ заряда) ΠΈ Ρ‚ΠΎΠΊ (ΠΈ ΠΈΠ·ΠΌΠ΅Π½Π΅Π½ΠΈΠ΅ Ρ‚ΠΎΠΊΠ°) относятся ΠΊ ΠΏΡ€ΠΎΡˆΠ»ΠΎΠΌΡƒ ($t_r\leq t$), ΠΊΠΎΡ‚ΠΎΡ€Ρ‹Π΅ Π²Π»ΠΈΡΡŽΡ‚ Π½Π° настоящСС.

Π˜Ρ‚Π°ΠΊ, Π΄Π°Π²Π°ΠΉΡ‚Π΅ посмотрим. ΠšΡƒΠ»ΠΎΠ½ ΠΈ Π‘ΠΈΠΎ-Π‘Π°Π²Π°Ρ€ Π½Π΅Π²Π΅Ρ€Π½Ρ‹, Ссли Π½Π΅ Π² статикС. Бамоиндукция Π½Π΅ΠΏΠΎΠ΄Π²ΠΈΠΆΠ½ΠΎΠ³ΠΎ ΠΏΡ€ΠΎΠ²ΠΎΠ΄Π° вызываСтся элСктричСскими полями, ΡΠΎΠ·Π΄Π°ΡŽΡ‰ΠΈΠΌΠΈ Π½Π΅Π½ΡƒΠ»Π΅Π²ΡƒΡŽ Π­Π”Π‘. Π­Ρ‚ΠΈ элСктричСскиС поля Π²Ρ‹Π·Π²Π°Π½Ρ‹ ΠΈΠ·ΠΌΠ΅Π½ΡΡŽΡ‰ΠΈΠΌΠΈΡΡ Π²ΠΎ Π²Ρ€Π΅ΠΌΠ΅Π½ΠΈ Ρ‚ΠΎΠΊΠ°ΠΌΠΈ . И Π΄Π°, эти ΠΈΠ·ΠΌΠ΅Π½ΡΡŽΡ‰ΠΈΠ΅ΡΡ Π²ΠΎ Π²Ρ€Π΅ΠΌΠ΅Π½ΠΈ Ρ‚ΠΎΠΊΠΈ ΠΈ Π²Ρ‹Π·Ρ‹Π²Π°ΡŽΡ‚ (ΠΈ Π΄Π°, я имСю Π² Π²ΠΈΠ΄Ρƒ ΠΏΡ€ΠΈΡ‡ΠΈΠ½Ρƒ) ΠΌΠ°Π³Π½ΠΈΡ‚Π½Ρ‹Π΅ поля.

И ΠΊΠΎΠ³Π΄Π° люди говорят, Ρ‡Ρ‚ΠΎ ΠΏΠ΅Ρ€Π΅ΠΌΠ΅Π½Π½Ρ‹Π΅ элСктричСскиС поля Π²Ρ‹Π·Ρ‹Π²Π°ΡŽΡ‚ ΠΌΠ°Π³Π½ΠΈΡ‚Π½Ρ‹Π΅ поля ΠΈ Π½Π°ΠΎΠ±ΠΎΡ€ΠΎΡ‚, это Π½Π΅ такая ΠΏΡ€ΠΈΡ‡ΠΈΠ½Π½ΠΎΡΡ‚ΡŒ, Π³Π΄Π΅ ΠΎΠ΄Π½ΠΎ Π²Ρ‹Π·Ρ‹Π²Π°Π΅Ρ‚ Π΄Ρ€ΡƒΠ³ΠΎΠ΅, это просто равСнство , Ρ‚Π°ΠΊ ΠΊΠ°ΠΊ ΠΎΠ½ΠΈ ΡΠ²ΡΠ·Ρ‹Π²Π°ΡŽΡ‚ Π΄Π²Π΅ Π²Π΅Ρ‰ΠΈ, происходящиС ΠΎΠ΄Π½ΠΎΠ²Ρ€Π΅ΠΌΠ΅Π½Π½ΠΎ. ΠŸΡ€ΠΈΡ‡ΠΈΠ½Π½ΠΎΡΡ‚ΡŒ связываСт настоящСС (слСдствия) с ΠΏΡ€ΠΎΡˆΠ»Ρ‹ΠΌ (ΠΏΡ€ΠΈΡ‡ΠΈΠ½Ρ‹). Π¦ΠΈΡ‚ΠΈΡ€ΠΎΠ²Π°Ρ‚ΡŒ Π•Ρ„ΠΈΠΌΠ΅Π½ΠΊΠΎ:

Π½ΠΈ уравнСния МаксвСлла, Π½ΠΈ ΠΈΡ… Ρ€Π΅ΡˆΠ΅Π½ΠΈΡ Π½Π΅ ΡƒΠΊΠ°Π·Ρ‹Π²Π°ΡŽΡ‚ Π½Π° Π½Π°Π»ΠΈΡ‡ΠΈΠ΅ ΠΏΡ€ΠΈΡ‡ΠΈΠ½Π½ΠΎΠΉ связи ΠΌΠ΅ΠΆΠ΄Ρƒ элСктричСскими ΠΈ ΠΌΠ°Π³Π½ΠΈΡ‚Π½Ρ‹ΠΌΠΈ полями. Π‘Π»Π΅Π΄ΠΎΠ²Π°Ρ‚Π΅Π»ΡŒΠ½ΠΎ, ΠΌΡ‹ Π΄ΠΎΠ»ΠΆΠ½Ρ‹ Π·Π°ΠΊΠ»ΡŽΡ‡ΠΈΡ‚ΡŒ, Ρ‡Ρ‚ΠΎ элСктромагнитноС ΠΏΠΎΠ»Π΅ Π΅ΡΡ‚ΡŒ двойствСнная ΡΡƒΡ‰Π½ΠΎΡΡ‚ΡŒ, всСгда ΠΈΠΌΠ΅ΡŽΡ‰Π°Ρ ΡΠ»Π΅ΠΊΡ‚Ρ€ΠΈΡ‡Π΅ΡΠΊΡƒΡŽ ΠΈ ΠΌΠ°Π³Π½ΠΈΡ‚Π½ΡƒΡŽ ΡΠΎΡΡ‚Π°Π²Π»ΡΡŽΡ‰ΠΈΠ΅ ΠΎΠ΄Π½ΠΎΠ²Ρ€Π΅ΠΌΠ΅Π½Π½ΠΎ , создаваСмыС ΠΈΡ… ΠΎΠ±Ρ‰ΠΈΠΌΠΈ источниками: ΠΏΠ΅Ρ€Π΅ΠΌΠ΅Π½Π½Ρ‹ΠΌΠΈ Π²ΠΎ Π²Ρ€Π΅ΠΌΠ΅Π½ΠΈ элСктричСскими зарядами ΠΈ Ρ‚ΠΎΠΊΠ°ΠΌΠΈ (Π²Ρ‹Π΄Π΅Π»Π΅Π½ΠΎ ΠΌΠ½ΠΎΠΉ).

Π’Π΅ΠΏΠ΅Ρ€ΡŒ ΠΏΠ΅Ρ€Π΅ΠΉΠ΄Π΅ΠΌ ΠΊ полям self. Π›Π΅Π³ΠΊΠΎ ΠΏΡ€Π΅Π΄ΡΡ‚Π°Π²ΠΈΡ‚ΡŒ, Ρ‡Ρ‚ΠΎ ΠΊΠ°ΠΆΠ΄Ρ‹ΠΉ заряд Ρ€Π΅Π°Π³ΠΈΡ€ΡƒΠ΅Ρ‚ Ρ‚ΠΎΠ»ΡŒΠΊΠΎ Π½Π° поля Π΄Ρ€ΡƒΠ³ΠΈΡ… зарядов. Но Π½Π΅ слСдуСт слишком ΡƒΠΏΡ€ΠΎΡ‰Π°Ρ‚ΡŒ ΠΏΡ€ΠΈ рассмотрСнии Π»ΡƒΡ‡Π΅Π²ΠΎΠΉ Ρ€Π΅Π°ΠΊΡ†ΠΈΠΈ ΠΈ Π΄Ρ€ΡƒΠ³ΠΈΡ… ΠΏΠΎΠ΄ΠΎΠ±Π½Ρ‹Ρ… ослоТнСний.

На самом Π΄Π΅Π»Π΅ Ρƒ вас Π΅ΡΡ‚ΡŒ заряды ΠΈ поля. И Π²Ρ‹ Π΄ΠΎΠ»ΠΆΠ½Ρ‹ ΡƒΠΊΠ°Π·Π°Ρ‚ΡŒ ΠΎΠ±Π°. Π£ ΠΊΠ°ΠΆΠ΄ΠΎΠ³ΠΎ Π΅ΡΡ‚ΡŒ энСргия, Ρƒ ΠΊΠ°ΠΆΠ΄ΠΎΠ³ΠΎ Π΅ΡΡ‚ΡŒ ΠΈΠΌΠΏΡƒΠ»ΡŒΡ, ΠΈ ΠΎΠ½ΠΈ ΠΎΠ±ΠΌΠ΅Π½ΠΈΠ²Π°ΡŽΡ‚ΡΡ Π΄Ρ€ΡƒΠ³ с Π΄Ρ€ΡƒΠ³ΠΎΠΌ энСргиСй ΠΈ ΠΈΠΌΠΏΡƒΠ»ΡŒΡΠΎΠΌ. Поля β€” это Π½Π΅ просто матСматичСскиС Π²Ρ‹Π΄ΡƒΠΌΠΊΠΈ для вычислСния сил ΠΌΠ΅ΠΆΠ΄Ρƒ частицами, это Ρ€Π΅Π°Π»ΡŒΠ½Ρ‹Π΅ Π²Π΅Ρ‰ΠΈ со своими стСпСнями свободы, собствСнной энСргиСй, ΠΈΠΌΠΏΡƒΠ»ΡŒΡΠΎΠΌ ΠΈ Π΄Π°ΠΆΠ΅ напряТСниСм.

Π’ΠΎΠ³Π΄Π° ΠΌΠΎΠΆΠ½ΠΎ ΡƒΠ·Π½Π°Ρ‚ΡŒ ΡΠ½Π΅Ρ€Π³ΠΈΡŽ ΠΈ ΠΈΠΌΠΏΡƒΠ»ΡŒΡ ΠΏΠΎΠ»Π΅ΠΉ ΠΈ зарядов ΠΈ ΠΏΠΎΡ‚ΠΎΠΌ ΡƒΠ·Π½Π°Ρ‚ΡŒ, ΠΊΠ°ΠΊ ΠΎΠ½ΠΈ Π²Π·Π°ΠΈΠΌΠ½ΠΎ ΠΎΠ±ΠΌΠ΅Π½ΠΈΠ²Π°ΡŽΡ‚ΡΡ энСргиСй ΠΈ ΠΈΠΌΠΏΡƒΠ»ΡŒΡΠΎΠΌ ΠΌΠ΅ΠΆΠ΄Ρƒ собой.

Но, ΠΊΠ°ΠΊ всС Π³ΠΎΠ²ΠΎΡ€ΠΈΠ»ΠΈ, самосила ΠΌΠΎΠΆΠ΅Ρ‚ Π±Ρ‹Ρ‚ΡŒ, Ρ‚ΠΎΠ»ΡŒΠΊΠΎ Ссли заряды ΠΈΠ·Π»ΡƒΡ‡Π°ΡŽΡ‚.

НС всС Ρ‚Π°ΠΊ говорят. БобствСнная сила ΠΈΠΌΠ΅Π΅Ρ‚ Π³ΠΎΡ€Π°Π·Π΄ΠΎ Π±ΠΎΠ»Π΅Π΅ ΠΎΠ±Ρ‰Π΅Π΅ Π·Π½Π°Ρ‡Π΅Π½ΠΈΠ΅, Ρ‡Π΅ΠΌ силы Ρ€Π°Π΄ΠΈΠ°Ρ†ΠΈΠΎΠ½Π½ΠΎΠΉ Ρ€Π΅Π°ΠΊΡ†ΠΈΠΈ. Π’ Π½Π°ΠΈΠ±ΠΎΠ»Π΅Π΅ распространСнном случаС гармоничСского двиТСния частица ΠΏΠΎΠ»ΡƒΡ‡Π°Π΅Ρ‚ ΡΡ‚ΠΎΠ»ΡŒΠΊΠΎ ΠΆΠ΅ энСргии, сколько тСряСт ΠΎΡ‚ ΠΈΠ½Π΄ΡƒΠΊΡ‚ΠΈΠ²Π½Ρ‹Ρ… ΠΏΠΎΠ»Π΅ΠΉ (поля Π¨ΠΎΡ‚Ρ‚Π°, ΠΊΠΎΡ‚ΠΎΡ€Ρ‹Π΅ ΠΎΡΠ»Π°Π±Π΅Π²Π°ΡŽΡ‚ быстрСС, Ρ‡Π΅ΠΌ поля излучСния, Π½ΠΎ ΡΠΎΡ…Ρ€Π°Π½ΡΡŽΡ‚ ΡΠ½Π΅Ρ€Π³ΠΈΡŽ) ΠΈ Ρ‚ΠΎΠ»ΡŒΠΊΠΎ Π² срСднСм тСряСт ΡΠ½Π΅Ρ€Π³ΠΈΡŽ Ρ€Π°Π΄ΠΈΠ°Ρ†ΠΈΠΎΠ½Π½Ρ‹ΠΌ полям.

МногиС люди ΠΈΠ·ΡƒΡ‡Π°ΡŽΡ‚ это. Π‘Ρ‚ΠΈΠ²Π΅Π½ Π›Π°ΠΉΠ» написал Ρ†Π΅Π»ΡƒΡŽ ΠΊΠ½ΠΈΠ³Ρƒ Ρ‚ΠΎΠ»ΡŒΠΊΠΎ ΠΎ Ρ€Π°Π²Π½ΠΎΠΌΠ΅Ρ€Π½ΠΎ ΡƒΡΠΊΠΎΡ€ΡΡŽΡ‰Π΅ΠΉΡΡ заряТСнной частицС ΠΈ Ρ†Π΅Π»ΡƒΡŽ ΠΊΠ½ΠΈΠ³Ρƒ ΠΎ Π²Π½ΡƒΡ‚Ρ€Π΅Π½Π½ΠΈΡ… силах. Π“Π΅Ρ€Π±Π΅Ρ€Ρ‚ Π¨ΠΏΠΎΠ½ написал Ρ†Π΅Π»ΡƒΡŽ ΠΊΠ½ΠΈΠ³Ρƒ ΠΎ Π²Π·Π°ΠΈΠΌΠ½ΠΎ связанной Π΄ΠΈΠ½Π°ΠΌΠΈΠΊΠ΅ элСктромагнитных ΠΏΠΎΠ»Π΅ΠΉ ΠΈ зарядов. А Π€Ρ€ΠΈΡ† Π ΠΎΡ€Π»ΠΈΡ… написал ΠΊΠ»Π°ΡΡΠΈΡ‡Π΅ΡΠΊΡƒΡŽ ΠΊΠ½ΠΈΠ³Ρƒ ΠΎ классичСских заряТСнных частицах. И это всСго лишь ΠΊΠ½ΠΈΠ³ΠΈ, ΠΏΠΎ этой Ρ‚Π΅ΠΌΠ΅ Π΅ΠΆΠ΅Π³ΠΎΠ΄Π½ΠΎ публикуСтся, Π½Π°Π²Π΅Ρ€Π½ΠΎΠ΅, Π½Π΅ ΠΌΠ΅Π½Π΅Π΅ дСсятка статСй, ΠΏΠΎ ΠΊΡ€Π°ΠΉΠ½Π΅ΠΉ ΠΌΠ΅Ρ€Π΅, Π·Π° послСдниС сто Π»Π΅Ρ‚.

Π˜Ρ‚Π°ΠΊ. Π•ΡΡ‚ΡŒ ΠΌΠ½ΠΎΠ³ΠΎ собствСнных сил. Π”Π΅Π±Π°Ρ‚Ρ‹ ΠΏΡ€ΠΎΠ΄ΠΎΠ»ΠΆΠ°ΡŽΡ‚ΡΡ ΠΈ ΠΏΠΎ сСй дСнь ΠΎ Ρ‚ΠΎΠΌ, Π²Ρ‹Π·Ρ‹Π²Π°ΡŽΡ‚ Π»ΠΈ собствСнныС силы ΠΈΠ·ΠΌΠ΅Π½Π΅Π½ΠΈΠ΅ сил, Ρ€Ρ‹Π²ΠΎΠΊ (ΠΈΠ·ΠΌΠ΅Π½Π΅Π½ΠΈΠ΅ ускорСния) ΠΈΠ»ΠΈ само ускорСниС. Или это Ρ‡Ρ‚ΠΎ-Ρ‚ΠΎ совсСм Π΄Ρ€ΡƒΠ³ΠΎΠ΅.

Π˜ΠΌΠ΅ΠΉΡ‚Π΅ Π² Π²ΠΈΠ΄Ρƒ, Ρ‡Ρ‚ΠΎ Π² ΡƒΡ‡Π΅Π±Π½ΠΈΠΊΠ΅ ΠΎ ΠΏΡ€ΠΎΠ²ΠΎΠ΄Π΅ классичСского Ρ€Π°Π·ΠΌΠ΅Ρ€Π° ΠΎΠ±Ρ‹Ρ‡Π½ΠΎ ΠΎΠ±ΡΡƒΠΆΠ΄Π°ΡŽΡ‚ΡΡ макроскопичСскиС поля, Π³Π΄Π΅ Π²Ρ‹ усрСдняСтС достаточно большоС количСство Π°Ρ‚ΠΎΠΌΠΎΠ², Ρ‡Ρ‚ΠΎΠ±Ρ‹ поля, заряд ΠΈ Ρ‚ΠΎΠΊ стали Π±ΠΎΠ»Π΅Π΅ Π³Π»Π°Π΄ΠΊΠΈΠΌΠΈ полями, ΠΊΠΎΡ‚ΠΎΡ€Ρ‹Π΅ Π½Π΅ ΠΏΡ€Ρ‹Π³Π°ΡŽΡ‚ ΠΈΠ·-Π·Π° Ρ‚Π΅ΠΏΠ»ΠΎΠ²Ρ‹Ρ… эффСктов. ΠΈΠ»ΠΈ Π² зависимости ΠΎΡ‚ Ρ‚ΠΎΠ³ΠΎ, Π³Π΄Π΅ Π²Ρ‹ Π½Π°Ρ…ΠΎΠ΄ΠΈΡ‚Π΅ΡΡŒ Π² Ρ€Π΅ΡˆΠ΅Ρ‚ΠΊΠ΅ Ρ‚Π²Π΅Ρ€Π΄ΠΎΠ³ΠΎ Ρ‚Π΅Π»Π°. И ΠΊΠΎΠ³Π΄Π° ΠΊΠΎΠ³ΠΎ-Ρ‚ΠΎ Π²ΠΎΠ»Π½ΡƒΡŽΡ‚ эти эффСкты, ΠΎΠ½ΠΈ ΠΈΡΠΏΠΎΠ»ΡŒΠ·ΡƒΡŽΡ‚ ΡΡ‚Π°Ρ‚ΠΈΡΡ‚ΠΈΡ‡Π΅ΡΠΊΡƒΡŽ Ρ„ΠΈΠ·ΠΈΠΊΡƒ ΠΈ/ΠΈΠ»ΠΈ ΠΊΠ²Π°Π½Ρ‚ΠΎΠ²ΡƒΡŽ Ρ„ΠΈΠ·ΠΈΠΊΡƒ. НС Ρ‚ΠΎΠ»ΡŒΠΊΠΎ чистый МаксвСлл.

$\endgroup$

28

$\begingroup$

Π—Π°Π΄ΡƒΠΌΡ‹Π²Π°Π»ΠΈΡΡŒ Π»ΠΈ Π²Ρ‹ ΠΊΠΎΠ³Π΄Π°-Π½ΠΈΠ±ΡƒΠ΄ΡŒ сами, Ρ‡Ρ‚ΠΎ ΠΌΠ°Π³Π½ΠΈΡ‚Π½Ρ‹Π΅ Π΄ΠΈΠΏΠΎΠ»ΡŒΠ½Ρ‹Π΅ ΠΌΠΎΠΌΠ΅Π½Ρ‚Ρ‹ ΡΠ²Π»ΡΡŽΡ‚ΡΡ ΠΏΡ€ΠΈΡ‡ΠΈΠ½ΠΎΠΉ любого ΠΌΠ°Π³Π½ΠΈΡ‚Π½ΠΎΠ³ΠΎ поля? Ссли ΠΎΠ½ΠΈ Π²Ρ‹Ρ€ΠΎΠ²Π½Π΅Π½Ρ‹, ΠΎΠ½ΠΈ ΠΎΠ±Ρ€Π°Π·ΡƒΡŽΡ‚ внСшнСС ΠΏΠΎΠ»Π΅. Π’Ρ‹Ρ€Π°Π²Π½ΠΈΠ²Π°Π½ΠΈΠ΅ происходит Π·Π° счСт ускорСния, ΠΏΠΎΡ‚ΠΎΠΌΡƒ Ρ‡Ρ‚ΠΎ ΠΌΠ°Π³Π½ΠΈΡ‚Π½Ρ‹ΠΉ Π΄ΠΈΠΏΠΎΠ»ΡŒΠ½Ρ‹ΠΉ ΠΌΠΎΠΌΠ΅Π½Ρ‚ связан с собствСнным спином (Π½Π° самом Π΄Π΅Π»Π΅ это Π²Ρ€Π°Ρ‰Π΅Π½ΠΈΠ΅ согласно Ρ€Π΅Π°Π»ΡŒΠ½ΠΎ ΠΏΡ€ΠΎΠ²Π΅Π΄Π΅Π½Π½ΠΎΠΌΡƒ экспСримСнту Π­ΠΉΠ½ΡˆΡ‚Π΅ΠΉΠ½Π° ΠΈ Π΄Π΅ Π₯ааса). И Ρ‡Ρ‚ΠΎ ΡƒΡ‚Π²Π΅Ρ€ΠΆΠ΄Π΅Π½ΠΈΠ΅ ΠΎ двиТущихся зарядах являСтся Π½Π΅ΠΏΡ€Π°Π²ΠΈΠ»ΡŒΠ½Ρ‹ΠΌ Ρ‚ΠΎΠ»ΠΊΠΎΠ²Π°Π½ΠΈΠ΅ΠΌ, заряды Π΄ΠΎΠ»ΠΆΠ½Ρ‹ Π±Ρ‹Ρ‚ΡŒ ускорСны (ΠΊΠ°ΠΊ Π² ΠΊΠ°Ρ‚ΡƒΡˆΠΊΠ΅). ЭкспСримСнт с ΠΌΠ°Π³Π½ΠΈΡ‚Π½Ρ‹ΠΌ ΠΏΠΎΠ»Π΅ΠΌ ΠΌΠ΅ΠΆΠ΄Ρƒ двумя ΠΏΡ€ΠΎΠ²ΠΎΠ΄Π°ΠΌΠΈ проводился сначала ΠΏΡ€ΠΈ Π²ΠΊΠ»ΡŽΡ‡Π΅Π½ΠΈΠΈ Ρ‚ΠΎΠΊΠΎΠ² (Ρ€Π°Π·Π³ΠΎΠ½ зарядов).

$\endgroup$

4

$\begingroup$

ΠŸΡƒΡΡ‚ΡŒ сгусток зарядов двиТСтся с постоянной ΡΠΊΠΎΡ€ΠΎΡΡ‚ΡŒΡŽ $\mathbf v\;.$ ΠŸΠΎΡΠΊΠΎΠ»ΡŒΠΊΡƒ заряды двиТутся, ΠΎΠ½ΠΈ Π±ΡƒΠ΄ΡƒΡ‚ ΡΠΎΠ·Π΄Π°Π²Π°Ρ‚ΡŒ ΠΌΠ°Π³Π½ΠΈΡ‚Π½ΠΎΠ΅ ΠΏΠΎΠ»Π΅ $\bf B$, Ρ‚Π°ΠΊ ΠΊΠ°ΠΊ ΠΈΠΌΠ΅Π½Π½ΠΎ Ρ‚ΠΎΠΊ создаСт ΠΌΠ°Π³Π½ΠΈΡ‚Π½ΠΎΠ΅ ΠΏΠΎΠ»Π΅.

ΠœΡ‹ всС Π·Π½Π°ΠΊΠΎΠΌΡ‹ с Ρ‚ΠΎΠΊΠΎΠΌ Π² ΠΏΡ€ΠΎΠ²ΠΎΠ΄Π΅, ΠΊΠ°ΠΊ ΠΏΠΎΠΊΠ°Π·Π°Π½ΠΎ Π½Π° рисункС Π½ΠΈΠΆΠ΅, Π½ΠΎ двиТущиСся заряды Π½Π° самом Π΄Π΅Π»Π΅ Π½Π΅ Β«ΡΠΎΠ·Π΄Π°ΡŽΡ‚Β» ΠΌΠ°Π³Π½ΠΈΡ‚Π½ΠΎΠ΅ ΠΏΠΎΠ»Π΅. Π‘ΠΌ. Π½Π΅ΠΌΠ½ΠΎΠ³ΠΎ ΠΎΠ± этом Ρƒ Π•Ρ„ΠΈΠΌΠ΅Π½ΠΊΠΎ: «. ..Π½ΠΈ уравнСния МаксвСлла, Π½ΠΈ ΠΈΡ… Ρ€Π΅ΡˆΠ΅Π½ΠΈΡ Π½Π΅ ΡƒΠΊΠ°Π·Ρ‹Π²Π°ΡŽΡ‚ Π½Π° Π½Π°Π»ΠΈΡ‡ΠΈΠ΅ ΠΏΡ€ΠΈΡ‡ΠΈΠ½Π½ΠΎ-слСдствСнной связи ΠΌΠ΅ΠΆΠ΄Ρƒ элСктричСским ΠΈ ΠΌΠ°Π³Π½ΠΈΡ‚Π½Ρ‹ΠΌ полями. Π‘Π»Π΅Π΄ΠΎΠ²Π°Ρ‚Π΅Π»ΡŒΠ½ΠΎ, ΠΌΡ‹ Π΄ΠΎΠ»ΠΆΠ½Ρ‹ Π·Π°ΠΊΠ»ΡŽΡ‡ΠΈΡ‚ΡŒ, Ρ‡Ρ‚ΠΎ элСктромагнитноС ΠΏΠΎΠ»Π΅ являСтся двойствСнной ΡΡƒΡ‰Π½ΠΎΡΡ‚ΡŒΡŽ, всСгда ΠΈΠΌΠ΅ΡŽΡ‰Π΅ΠΉ ΡΠ»Π΅ΠΊΡ‚Ρ€ΠΈΡ‡Π΅ΡΠΊΡƒΡŽ ΠΈ ΠΌΠ°Π³Π½ΠΈΡ‚Π½ΡƒΡŽ ΡΠΎΡΡ‚Π°Π²Π»ΡΡŽΡ‰ΠΈΠ΅ ΠΎΠ΄Π½ΠΎΠ²Ρ€Π΅ΠΌΠ΅Π½Π½ΠΎ…»

Π’Ρ‹ ΠΌΠΎΠΆΠ΅Ρ‚Π΅ ΠΏΠΎΠ½ΡΡ‚ΡŒ это, прСдставив, Ρ‡Ρ‚ΠΎ Π²Ρ‹ ΠΏΠΎΠ·ΠΈΡ‚Ρ€ΠΎΠ½, ΠΈ я саТаю вас ΠΏΠ΅Ρ€Π΅Π΄ ΠΏΡ€ΠΈΠ±ΠΈΡ‚Ρ‹ΠΌ ΠΊ Π·Π΅ΠΌΠ»Π΅ элСктроном. Π­Ρ‚ΠΎΡ‚ элСктрон ΠΈΠΌΠ΅Π΅Ρ‚ элСктромагнитноС ΠΏΠΎΠ»Π΅. Π£ Π½Π΅Π³ΠΎ Π½Π΅Ρ‚ элСктричСского поля ΠΈΠ»ΠΈ ΠΌΠ°Π³Π½ΠΈΡ‚Π½ΠΎΠ³ΠΎ поля, Ρƒ Π½Π΅Π³ΠΎ Π΅ΡΡ‚ΡŒ элСктромагнитноС ΠΏΠΎΠ»Π΅ . Как ΠΈ ΠΏΠΎΠ·ΠΈΡ‚Ρ€ΠΎΠ½. Из-Π·Π° этого, ΠΊΠΎΠ³Π΄Π° я ΠΎΡ‚ΠΏΡƒΡΠΊΠ°ΡŽ тСбя, Ρ‚Ρ‹ прямолинСйно двиТСшься ΠΊ элСктрону. Π—Π°Ρ‚Π΅ΠΌ, ΠΏΠΎΡΠΊΠΎΠ»ΡŒΠΊΡƒ Π²Ρ‹ Π²ΠΈΠ΄ΠΈΡ‚Π΅ Ρ‚ΠΎΠ»ΡŒΠΊΠΎ Π»ΠΈΠ½Π΅ΠΉΠ½ΠΎΠ΅ Π΄Π²ΠΈΠΆΠ΅Π½ΠΈΠ΅, Π²Ρ‹ Π΄ΡƒΠΌΠ°Π΅Ρ‚Π΅ ΠΎ Ρ€Π°Π΄ΠΈΠ°Π»ΡŒΠ½ΠΎΠΌ элСктричСском ΠΏΠΎΠ»Π΅ согласно курсу Π­Π½Π΄Ρ€ΡŽ Π”Π°Ρ„Ρ„ΠΈ ΠΏΠΎ Ρ„ΠΈΠ·ΠΈΠΊΠ΅ 106:

Но Ρ‡Ρ‚ΠΎ ΠΏΡ€ΠΎΠΈΠ·ΠΎΠΉΠ΄Π΅Ρ‚, Ссли я Π±Ρ€ΠΎΡˆΡƒ вас ΠΌΠΈΠΌΠΎ элСктрона? Π’Ρ‹ ΠΏΠΎ-ΠΏΡ€Π΅ΠΆΠ½Π΅ΠΌΡƒ Π΄Π²ΠΈΠΆΠ΅Ρ‚Π΅ΡΡŒ ΠΊ Π½Π΅ΠΌΡƒ, Π½ΠΎ Π²Ρ‹ Ρ‚Π°ΠΊΠΆΠ΅ испытываСтС Π²Ρ€Π°Ρ‰Π°Ρ‚Π΅Π»ΡŒΠ½ΠΎΠ΅ Π΄Π²ΠΈΠΆΠ΅Π½ΠΈΠ΅, ΠΊΠΎΡ‚ΠΎΡ€ΠΎΠ΅ Π²Ρ‹ приписываСтС ΠΌΠ°Π³Π½ΠΈΡ‚Π½ΠΎΠΌΡƒ полю элСктрона. Но Π·Π°ΠΌΠ΅Ρ‚ΡŒΡ‚Π΅, Ρ‡Ρ‚ΠΎ Π²Ρ‹ создали ΠΌΠ°Π³Π½ΠΈΡ‚Π½ΠΎΠ΅ ΠΏΠΎΠ»Π΅ элСктрона Π½Π΅ Ρ‚ΠΎΠ»ΡŒΠΊΠΎ ΠΏΠΎΡ‚ΠΎΠΌΡƒ, Ρ‡Ρ‚ΠΎ Π²Ρ‹ ΠΏΠ΅Ρ€Π΅Π΅Ρ…Π°Π»ΠΈ. 2} \;;$$, Ρ‚Π°ΠΊ ΠΊΠ°ΠΊ $d\mathbf l$ & $\hat{\ mathbf r}$ Π½Π°ΠΏΡ€Π°Π²Π»Π΅Π½Ρ‹ Π² ΠΎΠ΄Π½ΠΎΠΌ Π½Π°ΠΏΡ€Π°Π²Π»Π΅Π½ΠΈΠΈ, Ρ‡ΠΈΡΠ»ΠΈΡ‚Π΅Π»ΡŒ Ρ€Π°Π²Π΅Π½ Π½ΡƒΠ»ΡŽ, Π° Π·Π½Π°ΠΌΠ΅Π½Π°Ρ‚Π΅Π»ΡŒ $r$ Ρ€Π°Π²Π΅Π½ Π½ΡƒΠ»ΡŽ, Ρ‡Ρ‚ΠΎ снова Π΄Π΅Π»Π°Π΅Ρ‚ силу бСсконСчной. Π­Ρ‚ΠΎ Π½Π΅Π²ΠΎΠ·ΠΌΠΎΠΆΠ½ΠΎ.

БогласСн. Π’ΠΎΡ‡Π΅Ρ‡Π½Ρ‹Π΅ частицы ΡƒΠΏΡ€ΠΎΡ‰Π°ΡŽΡ‚ ΠΌΠ°Ρ‚Π΅ΠΌΠ°Ρ‚ΠΈΠΊΡƒ, Π½ΠΎ это всСго лишь ΠΏΡ€ΠΈΠ±Π»ΠΈΠΆΠ΅Π½ΠΈΠ΅, Π° ΡƒΠΏΡ€ΠΎΡ‰Π΅Π½ΠΈΠ΅ ΠΌΠ°Ρ‚Π΅ΠΌΠ°Ρ‚ΠΈΠΊΠΈ Π½Π΅ Π΄ΠΎΠ»ΠΆΠ½ΠΎ ΡΠΊΡ€Ρ‹Π²Π°Ρ‚ΡŒ ΠΎΡ‚ людСй Ρ‚Π²Π΅Ρ€Π΄Ρ‹Π΅ Π½Π°ΡƒΡ‡Π½Ρ‹Π΅ Π΄ΠΎΠΊΠ°Π·Π°Ρ‚Π΅Π»ΡŒΡΡ‚Π²Π° Ρ‚Π°ΠΊΠΈΡ… Π²Π΅Ρ‰Π΅ΠΉ, ΠΊΠ°ΠΊ дифракция элСктронов ΠΈ эффСкт Π­ΠΉΠ½ΡˆΡ‚Π΅ΠΉΠ½Π°-Π΄Π΅-Π“Π°Π°Π·Π°. Но это Ρ‚Π°ΠΊ.

Π’Ρ‹Π²ΠΎΠ΄: На заряды Π½Π΅ ΠΌΠΎΠΆΠ΅Ρ‚ Π²Π»ΠΈΡΡ‚ΡŒ ΠΈΡ… собствСнноС элСктричСскоС ΠΏΠΎΠ»Π΅ ΠΈ ΠΌΠ°Π³Π½ΠΈΡ‚Π½ΠΎΠ΅ ΠΏΠΎΠ»Π΅.

БогласСн.

Но Ρ‚ΠΎΠ³Π΄Π° ΠΊΠ°ΠΊ ΠΌΠΎΠΆΠ΅Ρ‚ Π±Ρ‹Ρ‚ΡŒ самоиндукция , Ссли Π½Π° двиТущиСся заряды Π½Π΅ Π΄Π΅ΠΉΡΡ‚Π²ΡƒΡŽΡ‚ ΠΈΡ… собствСнныС ΠΌΠ°Π³Π½ΠΈΡ‚Π½Ρ‹Π΅ поля?

Бамоиндукция — это «ΠΈΠ½Π΄ΡƒΠΊΡ†ΠΈΡ напряТСния Π² ΠΏΡ€ΠΎΠ²ΠΎΠ΄Π΅ с Ρ‚ΠΎΠΊΠΎΠΌ ΠΏΡ€ΠΈ ΠΈΠ·ΠΌΠ΅Π½Π΅Π½ΠΈΠΈ Ρ‚ΠΎΠΊΠ° Π² самом ΠΏΡ€ΠΎΠ²ΠΎΠ΄Π΅» . И этот Ρ‚ΠΎΠΊ мСняСтся, ΠΏΠΎΡ‚ΠΎΠΌΡƒ Ρ‡Ρ‚ΠΎ происходит ΠΊΠ°ΠΊΠΎΠ΅-Ρ‚ΠΎ взаимодСйствиС с Π΄Ρ€ΡƒΠ³ΠΈΠΌΠΈ заряТСнными частицами. Как цСлая ΠΊΡƒΡ‡Π° заряТСнных частиц Π² ΠΎΠ±Π»ΠΈΠΊΠ΅ ΠΌΠΎΠ΅ΠΉ Ρ€ΡƒΠΊΠΈ, Ρ‰Π΅Π»ΠΊΠ°ΡŽΡ‰Π΅ΠΉ Π²Ρ‹ΠΊΠ»ΡŽΡ‡Π°Ρ‚Π΅Π»Π΅ΠΌ.

Π­Π”Π‘ самоиндукции, вызванная ΠΈΠ·ΠΌΠ΅Π½Π΅Π½ΠΈΠ΅ΠΌ ΠΏΠΎΡ‚ΠΎΠΊΠ° ΠΌΠ°Π³Π½ΠΈΡ‚Π½ΠΎΠ³ΠΎ поля, создаваСмого зарядами, дСйствуСт Π½Π° заряды; ΠΊΠ°ΠΆΠ΄Ρ‹ΠΉ заряд испытываСт силу $\mathbf F= q\; mathbf v\times \mathbf B $, Π³Π΄Π΅ $\mathbf B$ создаСтся ΠΈΠΌΠΈ самими. Π˜Ρ‚Π°ΠΊ, это ΠΎΠ·Π½Π°Ρ‡Π°Π΅Ρ‚, Ρ‡Ρ‚ΠΎ заряды двиТутся ΠΈ ΡΠΎΠ·Π΄Π°ΡŽΡ‚ ΠΌΠ°Π³Π½ΠΈΡ‚Π½ΠΎΠ΅ ΠΏΠΎΠ»Π΅, ΠΊΠΎΡ‚ΠΎΡ€ΠΎΠ΅ снова дСйствуСт Π½Π° заряды, создавшиС это ΠΏΠΎΠ»Π΅.

ΠŸΡ€ΠΎΡΡ‚ΠΎ упроститС этот Ρ‚ΠΎΠΊ Π΄ΠΎ ΠΎΠ΄Π½ΠΎΠ³ΠΎ элСктрона, Π·Π°Ρ‚Π΅ΠΌ ΠΏΡ€Π΅Π΄ΡΡ‚Π°Π²ΡŒΡ‚Π΅, Ρ‡Ρ‚ΠΎ ΠΎΠ½ Π½Π΅ двиТСтся, Π° вмСсто этого Π΄Π²ΠΈΠ³Π°Π΅Ρ‚Π΅ΡΡŒ Π²Ρ‹. Никто Π½Π΅ создавал ΠΌΠ°Π³Π½ΠΈΡ‚Π½ΠΎΠ΅ ΠΏΠΎΠ»Π΅ для элСктрона Ρ‚ΠΎΠ»ΡŒΠΊΠΎ ΠΏΠΎΡ‚ΠΎΠΌΡƒ, Ρ‡Ρ‚ΠΎ ΠΎΠ½ΠΈ двигались. ΠŸΠΎΡ‚ΠΎΠΌΡƒ Ρ‡Ρ‚ΠΎ Ρƒ Π½Π΅Π³ΠΎ элСктромагнитноС ΠΏΠΎΠ»Π΅ . А взаимодСйствия элСктромагнитного поля приводят ΠΊ Π»ΠΈΠ½Π΅ΠΉΠ½ΠΎΠΌΡƒ ΠΈ/ΠΈΠ»ΠΈ Π²Ρ€Π°Ρ‰Π°Ρ‚Π΅Π»ΡŒΠ½ΠΎΠΌΡƒ двиТСнию. Когда ΠΌΡ‹ Π²ΠΈΠ΄ΠΈΠΌ Ρ‚ΠΎΠ»ΡŒΠΊΠΎ ΠΏΠ΅Ρ€Π²ΠΎΠ΅, ΠΌΡ‹ Π½Π°Π·Ρ‹Π²Π°Π΅ΠΌ Π΅Π³ΠΎ элСктричСским ΠΏΠΎΠ»Π΅ΠΌ, ΠΊΠΎΠ³Π΄Π° ΠΌΡ‹ Π²ΠΈΠ΄ΠΈΠΌ Ρ‚ΠΎΠ»ΡŒΠΊΠΎ Π²Ρ‚ΠΎΡ€ΠΎΠ΅, ΠΌΡ‹ Π½Π°Π·Ρ‹Π²Π°Π΅ΠΌ Π΅Π³ΠΎ ΠΌΠ°Π³Π½ΠΈΡ‚Π½Ρ‹ΠΌ ΠΏΠΎΠ»Π΅ΠΌ, Π½ΠΎ рассматриваСмыС поля β€” это элСктромагнитныС поля. И самый простой ΠΏΡ€ΠΈΠΌΠ΅Ρ€ взаимодСйствия элСктромагнитного поля связан с ΠΏΠΎΠ·ΠΈΡ‚Ρ€ΠΎΠ½ΠΈΠ΅ΠΌ. Π•ΡΡ‚ΡŒ Π»ΠΈΠ½Π΅ΠΉΠ½ΠΎΠ΅ Π΄Π²ΠΈΠΆΠ΅Π½ΠΈΠ΅, ΠΊΠΎΠ³Π΄Π° элСктрон ΠΈ ΠΏΠΎΠ·ΠΈΡ‚Ρ€ΠΎΠ½ двиТутся навстрСчу Π΄Ρ€ΡƒΠ³ Π΄Ρ€ΡƒΠ³Ρƒ ΠΈ Π°Π½Π½ΠΈΠ³ΠΈΠ»ΠΈΡ€ΡƒΡŽΡ‚, Π° Ρ‚Π°ΠΊΠΆΠ΅ Π΅ΡΡ‚ΡŒ Π²Ρ€Π°Ρ‰Π°Ρ‚Π΅Π»ΡŒΠ½ΠΎΠ΅ Π΄Π²ΠΈΠΆΠ΅Π½ΠΈΠ΅:

Π˜Π·ΠΎΠ±Ρ€Π°ΠΆΠ΅Π½ΠΈΠ΅ CCASA ΠΎΡ‚ Manticorp/Rubber Duck, см. Wikipedia

Но ΠΊΠ°ΠΊ ΠΆΠ΅ Ρ‚Π°ΠΊ? Π­Ρ‚ΠΎ Π΄Π°Π»ΠΎ Π±Ρ‹ Π±Π΅ΡΠΊΠΎΠ½Π΅Ρ‡Π½ΡƒΡŽ силу (Π΄Π°ΠΆΠ΅ Ссли Π±Ρ‹ ΠΎΠ½Π° Π½Π΅ Π±Ρ‹Π»Π° бСсконСчной, ΠΎΠ½Π° Π΄ΠΎΠ»ΠΆΠ½Π° Π±Ρ‹Π»Π° Π±Ρ‹ Π±Ρ‹Ρ‚ΡŒ Ρ€Π°Π²Π½Π° Π½ΡƒΠ»ΡŽ, ΠΏΠΎΡΠΊΠΎΠ»ΡŒΠΊΡƒ $d\mathbf l$ ΠΈ $\hat{\mathbf r}$ ΠΏΠ°Ρ€Π°Π»Π»Π΅Π»ΡŒΠ½Ρ‹).

БСсконСчных сил Π½Π΅ сущСствуСт, элСктрон ΠΈΠΌΠ΅Π΅Ρ‚ свою ΠΊΠΎΠΌΠΏΡ‚ΠΎΠ½ΠΎΠ²ΡΠΊΡƒΡŽ Π΄Π»ΠΈΠ½Ρƒ Π²ΠΎΠ»Π½Ρ‹ ΠΈ ΠΈΠΌΠ΅Π΅Ρ‚ «ΡΠΏΠΈΠ½ΠΎΡ€Π½ΡƒΡŽ» ΠΏΡ€ΠΈΡ€ΠΎΠ΄Ρƒ.

Π˜Ρ‚Π°ΠΊ, вопрос Π½Π° ΠΌΠΈΠ»Π»ΠΈΠΎΠ½ Π΄ΠΎΠ»Π»Π°Ρ€ΠΎΠ²: влияСт Π»ΠΈ Π½Π° двиТущиСся заряды ΠΌΠ°Π³Π½ΠΈΡ‚Π½ΠΎΠ΅ ΠΏΠΎΠ»Π΅, ΠΊΠΎΡ‚ΠΎΡ€ΠΎΠ΅ ΠΎΠ½ΠΈ ΡΠΎΠ·Π΄Π°ΡŽΡ‚ ΠΏΡ€ΠΈ Π΄Π²ΠΈΠΆΠ΅Π½ΠΈΠΈ, создавая Ρ‚ΠΎΠΊ? Как ΠΌΠΎΠΆΠ΅Ρ‚ Π±Ρ‹Ρ‚ΡŒ, Ссли Ρ„ΠΎΡ€ΠΌΡƒΠ»Π° Π³ΠΎΠ²ΠΎΡ€ΠΈΡ‚ ΠΎΠ± ΠΎΠ±Ρ€Π°Ρ‚Π½ΠΎΠΌ?

НСт, ΠΏΠΎΡ‚ΠΎΠΌΡƒ Ρ‡Ρ‚ΠΎ вашС Π΄Π²ΠΈΠΆΠ΅Π½ΠΈΠ΅ ΠΎΡ‚Π½ΠΎΡΠΈΡ‚Π΅Π»ΡŒΠ½ΠΎ заряТСнной частицы Π½Π΅ создаСт для Π½Π΅Π΅ ΠΌΠ°Π³Π½ΠΈΡ‚Π½ΠΎΠ³ΠΎ поля. Π­Ρ‚ΠΎ просто ΠΏΡ€ΠΈΠ²ΠΎΠ΄ΠΈΡ‚ ΠΊ Π²Ρ€Π°Ρ‰Π°Ρ‚Π΅Π»ΡŒΠ½ΠΎΠΉ силС Π² Π΄ΠΎΠΏΠΎΠ»Π½Π΅Π½ΠΈΠ΅ ΠΊ Π»ΠΈΠ½Π΅ΠΉΠ½ΠΎΠΉ силС ΠΈΠ·-Π·Π° «спинорной» ΠΏΡ€ΠΈΡ€ΠΎΠ΄Ρ‹ заряТСнных частиц.

$\endgroup$

0

Π—Π°Ρ€Π΅Π³ΠΈΡΡ‚Ρ€ΠΈΡ€ΡƒΠΉΡ‚Π΅ΡΡŒ ΠΈΠ»ΠΈ Π²ΠΎΠΉΠ΄ΠΈΡ‚Π΅ Π² систСму

Π—Π°Ρ€Π΅Π³ΠΈΡΡ‚Ρ€ΠΈΡ€ΡƒΠΉΡ‚Π΅ΡΡŒ с ΠΏΠΎΠΌΠΎΡ‰ΡŒΡŽ Google

Π—Π°Ρ€Π΅Π³ΠΈΡΡ‚Ρ€ΠΈΡ€ΠΎΠ²Π°Ρ‚ΡŒΡΡ Ρ‡Π΅Ρ€Π΅Π· Facebook

Π—Π°Ρ€Π΅Π³ΠΈΡΡ‚Ρ€ΠΈΡ€ΡƒΠΉΡ‚Π΅ΡΡŒ, ΠΈΡΠΏΠΎΠ»ΡŒΠ·ΡƒΡ ΡΠ»Π΅ΠΊΡ‚Ρ€ΠΎΠ½Π½ΡƒΡŽ ΠΏΠΎΡ‡Ρ‚Ρƒ ΠΈ ΠΏΠ°Ρ€ΠΎΠ»ΡŒ

ΠžΠΏΡƒΠ±Π»ΠΈΠΊΠΎΠ²Π°Ρ‚ΡŒ ΠΊΠ°ΠΊ Π³ΠΎΡΡ‚ΡŒ

ЭлСктронная ΠΏΠΎΡ‡Ρ‚Π°

ВрСбуСтся, Π½ΠΎ Π½Π΅ отобраТаСтся

ΠžΠΏΡƒΠ±Π»ΠΈΠΊΠΎΠ²Π°Ρ‚ΡŒ ΠΊΠ°ΠΊ Π³ΠΎΡΡ‚ΡŒ

ЭлСктронная ΠΏΠΎΡ‡Ρ‚Π°

ВрСбуСтся, Π½ΠΎ Π½Π΅ отобраТаСтся

НаТимая Β«ΠžΠΏΡƒΠ±Π»ΠΈΠΊΠΎΠ²Π°Ρ‚ΡŒ свой ΠΎΡ‚Π²Π΅Ρ‚Β», Π²Ρ‹ ΡΠΎΠ³Π»Π°ΡˆΠ°Π΅Ρ‚Π΅ΡΡŒ с нашими условиями обслуТивания, ΠΏΠΎΠ»ΠΈΡ‚ΠΈΠΊΠΎΠΉ ΠΊΠΎΠ½Ρ„ΠΈΠ΄Π΅Π½Ρ†ΠΈΠ°Π»ΡŒΠ½ΠΎΡΡ‚ΠΈ ΠΈ ΠΏΠΎΠ»ΠΈΡ‚ΠΈΠΊΠΎΠΉ использования Ρ„Π°ΠΉΠ»ΠΎΠ² cookie

ΠœΠ°Π³Π½ΠΈΡ‚Π½ΠΎΠ΅ ΠΏΠΎΠ»Π΅ состоит ΠΈΠ· Ρ„ΠΎΡ‚ΠΎΠ½ΠΎΠ² | Π€ΠΈΠ·ΠΈΠΊΠ° Π€ΡƒΡ€Π³ΠΎΠ½

ΠšΠ°Ρ‚Π΅Π³ΠΎΡ€ΠΈΡ Π’Ρ‹Π±Π΅Ρ€ΠΈΡ‚Π΅ ΠΊΠ°Ρ‚Π΅Π³ΠΎΡ€ΠΈΡŽΠž Ρ„ΡƒΡ€Π³ΠΎΠ½Π΅ физикиЭлСктричСство ΠΈ магнитыВсС ΠΎΡΡ‚Π°Π»ΡŒΠ½ΠΎΠ΅Π‘Π²Π΅Ρ‚ ΠΈ Π·Π²ΡƒΠΊΠ”Π²ΠΈΠΆΠ΅Π½ΠΈΠ΅ вСщСйНовая ΠΈ Π·Π°Ρ…Π²Π°Ρ‚Ρ‹Π²Π°ΡŽΡ‰Π°Ρ физикаБостояния вСщСства ΠΈ ΡΠ½Π΅Ρ€Π³ΠΈΠΈΠšΠΎΡΠΌΠΎΡΠŸΠΎΠ΄ Π²ΠΎΠ΄ΠΎΠΉ ΠΈ Π² Π²ΠΎΠ·Π΄ΡƒΡ…Π΅

ΠŸΠΎΠ΄ΠΊΠ°Ρ‚Π΅Π³ΠΎΡ€ΠΈΡ

Поиск

Π—Π°Π΄Π°ΠΉΡ‚Π΅ вопрос

ПослСдний ΠΎΡ‚Π²Π΅Ρ‚: 22. 10.2007

Q:

Мой вопрос: ΠΈΠ· Ρ‡Π΅Π³ΠΎ состоит ΠΌΠ°Π³Π½ΠΈΡ‚Π½ΠΎΠ΅ ΠΏΠΎΠ»Π΅? Π― ΠΌΠ½ΠΎΠ³ΠΎ Ρ‡ΠΈΡ‚Π°Π» ΠΎ ΠΌΠ°Π³Π½ΠΈΡ‚Π°Ρ… ΠΈ ​​полях, ΠΊΠΎΡ‚ΠΎΡ€Ρ‹Π΅ ΠΎΠ½ΠΈ ΡΠΎΠ·Π΄Π°ΡŽΡ‚, ΠΈ Π΄Π°ΠΆΠ΅ ΠΎ Ρ‚ΠΎΠΌ, Ρ‡Ρ‚ΠΎ Π²ΠΎΠΊΡ€ΡƒΠ³ самих элСктронов Π΅ΡΡ‚ΡŒ ΠΌΠ°Π³Π½ΠΈΡ‚Π½Ρ‹Π΅ поля, Π½ΠΎ я ΠΏΠΎΠΊΠ° Π½Π΅ встрСчал Π½ΠΈΡ‡Π΅Π³ΠΎ, Ρ‡Ρ‚ΠΎ Π³ΠΎΠ²ΠΎΡ€ΠΈΠ»ΠΎ Π±Ρ‹ ΠΎ Ρ‚ΠΎΠΌ, ΠΈΠ· Ρ‡Π΅Π³ΠΎ состоит само ΠΏΠΎΠ»Π΅. Π­Ρ‚ΠΎ матСрия, поэтому ΠΎΠ½Π° Π΄ΠΎΠ»ΠΆΠ½Π° Π±Ρ‹Ρ‚ΡŒ сдСлана ΠΈΠ· Ρ‡Π΅Π³ΠΎ-Ρ‚ΠΎ. Π•ΡΡ‚ΡŒ Π»ΠΈ Π½Π°Π·Π²Π°Π½ΠΈΠ΅ для этих «частиц»? Или ΠΎΠ½ΠΈ сами просто элСктроны?
— Дуглас (35 Π»Π΅Ρ‚)
Π›ΡƒΠΈΠ·ΠΈΠ°Π½Π° БША

A:

ΠŸΡ€ΠΈΠ²Π΅Ρ‚, Дуглас!

Π­Π»Π΅ΠΊΡ‚Ρ€ΠΎΠΌΠ°Π³Π½ΠΈΡ‚Π½ΠΎΠ΅ взаимодСйствиС опосрСдовано константой ΠΎΠ±ΠΌΠ΅Π½ Ρ„ΠΎΡ‚ΠΎΠ½Π°ΠΌΠΈ ΠΎΡ‚ ΠΎΠ΄Π½ΠΎΠ³ΠΎ заряТСнного ΠΎΠ±ΡŠΠ΅ΠΊΡ‚Π° ΠΊ Π΄Ρ€ΡƒΠ³ΠΎΠΌΡƒ. ΠœΠ°Π³Π½ΠΈΡ‚Π½Ρ‹ΠΉ ΠΏΠΎΠ»Π΅ Π½Π° самом Π΄Π΅Π»Π΅ просто классичСскоС ΠΏΡ€ΠΈΠ±Π»ΠΈΠΆΠ΅Π½ΠΈΠ΅ ΠΊ Ρ„ΠΎΡ‚ΠΎΠ½Π½ΠΎΠΌΡƒ ΠΎΠ±ΠΌΠ΅Π½Ρƒ ΠΊΠ°Ρ€Ρ‚ΠΈΠ½Π°. Π’ двиТущСйся систСмС отсчСта вмСсто Π½Π΅Π΅ появляСтся ΠΌΠ°Π³Π½ΠΈΡ‚Π½ΠΎΠ΅ ΠΏΠΎΠ»Π΅ ΠΊΠ°ΠΊ комбинация ΠΌΠ°Π³Π½ΠΈΡ‚Π½ΠΎΠ³ΠΎ поля ΠΈ элСктричСского поля, поэтому элСктричСскоС Π° ΠΌΠ°Π³Π½ΠΈΡ‚Π½Ρ‹Π΅ поля состоят ΠΈΠ· ΠΎΠ΄Π½ΠΎΠ³ΠΎ ΠΈ Ρ‚ΠΎΠ³ΠΎ ΠΆΠ΅ «вСщСства» (Ρ„ΠΎΡ‚ΠΎΠ½ΠΎΠ²).

Π’ Π½Π΅ΠΊΠΎΡ‚ΠΎΡ€Ρ‹Ρ… элСктромагнитных взаимодСйствиях ΡƒΡ‡Π°ΡΡ‚Π²ΡƒΡŽΡ‚ «настоящиС» Ρ„ΠΎΡ‚ΠΎΠ½Ρ‹ с ΠΎΠΏΡ€Π΅Π΄Π΅Π»Π΅Π½Π½Ρ‹Π΅ частоты, энСргии ΠΈ ΠΈΠΌΠΏΡƒΠ»ΡŒΡΡ‹. ЭлСктростатичСскиС ΠΈ ΠΌΠ°Π³Π½ΠΈΡ‚Π½Ρ‹Π΅ вмСсто этого поля Π²ΠΊΠ»ΡŽΡ‡Π°ΡŽΡ‚ ΠΎΠ±ΠΌΠ΅Π½ Β«Π²ΠΈΡ€Ρ‚ΡƒΠ°Π»ΡŒΠ½Ρ‹ΠΌΠΈΒ» Ρ„ΠΎΡ‚ΠΎΠ½Π°ΠΌΠΈ. ΠžΡ‡Π΅Π½ΡŒ Π±Π»ΠΈΠ·ΠΊΠΎ ΠΊ элСктрон прСдставляСт собой ΠΏΠ»ΠΎΡ‚Π½ΠΎΠ΅ ΠΎΠ±Π»Π°ΠΊΠΎ Π²ΠΈΡ€Ρ‚ΡƒΠ°Π»ΡŒΠ½Ρ‹Ρ… Ρ„ΠΎΡ‚ΠΎΠ½ΠΎΠ², ΠΊΠΎΡ‚ΠΎΡ€Ρ‹Π΅ постоянно испускаСтся ΠΈ вновь поглощаСтся элСктроном. НСкоторыС ΠΈΠ· этих Ρ„ΠΎΡ‚ΠΎΠ½ΠΎΠ² Ρ€Π°ΡΡ‰Π΅ΠΏΠ»ΡΡŽΡ‚ΡΡ Π½Π° элСктрон-ΠΏΠΎΠ·ΠΈΡ‚Ρ€ΠΎΠ½Π½Ρ‹Π΅ ΠΏΠ°Ρ€Ρ‹ (ΠΈΠ»ΠΈ ΠΏΠ°Ρ€Ρ‹ Π΅Ρ‰Π΅ Π±ΠΎΠ»Π΅Π΅ тяТСлых вСщСств), ΠΊΠΎΡ‚ΠΎΡ€Ρ‹Π΅ Ρ€Π΅ΠΊΠΎΠΌΠ±ΠΈΠ½ΠΈΡ€ΡƒΡŽΡ‚ Π² Ρ„ΠΎΡ‚ΠΎΠ½Ρ‹, ΠΊΠΎΡ‚ΠΎΡ€Ρ‹Π΅ ΠΏΠΎΠ²Ρ‚ΠΎΡ€Π½ΠΎ ΠΏΠΎΠ³Π»ΠΎΡ‰Π°ΡŽΡ‚ΡΡ исходным элСктрон. Π­Ρ‚ΠΈ Π²ΠΈΡ€Ρ‚ΡƒΠ°Π»ΡŒΠ½Ρ‹Π΅ ΠΏΠ΅Ρ‚Π»ΠΈ частиц ΡΠΊΡ€Π°Π½ΠΈΡ€ΡƒΡŽΡ‚ заряд элСктрон Ρ‚Π°ΠΊ, Ρ‡Ρ‚ΠΎ Π²Π΄Π°Π»ΠΈ ΠΎΡ‚ элСктрона каТСтся, Ρ‡Ρ‚ΠΎ ΠΎΠ½ ΠΈΠΌΠ΅Π΅Ρ‚ мСньшС Π·Π°Ρ€ΡΠ΄ΠΈΡ‚ΡŒ Ρ‡Π΅ΠΌ рядом. 92 (Π² систСмС отсчСта, Π² ΠΊΠΎΡ‚ΠΎΡ€ΠΎΠΉ элСктрон Π½Π΅ ΠΈΠΌΠ΅Π΅Ρ‚ ΠΈΠΌΠΏΡƒΠ»ΡŒΡ).

Π’ΠΎΠΌ

(ΠΎΠΏΡƒΠ±Π»ΠΈΠΊΠΎΠ²Π°Π½ΠΎ 22.10.2007)

Π”ΠΎΠΏΠΎΠ»Π½Π΅Π½ΠΈΠ΅ β„–1: прСдставлСниС ΠΎ ΠΌΠ°Π³Π½Π΅Ρ‚ΠΈΠ·ΠΌΠ΅

Вопрос:

МоТно Π»ΠΈ Ρ‡Π΅Ρ‚ΠΊΠΎ ΠΎΡ‚Π²Π΅Ρ‚ΠΈΡ‚ΡŒ Π½Π° вопрос, ΠΈΡΠΏΠΎΠ»ΡŒΠ·ΡƒΡ Π±ΠΎΠ»Π΅Π΅ элСмСнтарныС Ρ‚Π΅Ρ€ΠΌΠΈΠ½Ρ‹ для элСмСнтарных частиц? Или, ΠΌΠΎΠΆΠ΅Ρ‚ Π±Ρ‹Ρ‚ΡŒ, схСма частиц ΠΊΠ°ΠΊ каскада Π² Ρ€Π°Π·Π»ΠΈΡ‡Π½Ρ‹Ρ… энСргСтичСских состояниях, производящих Π²Π·Π°ΠΈΠΌΠ½Ρ‹Π΅ силы ΠΈΠ»ΠΈ дСйствия Π½Π° Π΄Ρ€ΡƒΠ³ΡƒΡŽ ΠΌΠ°Ρ‚Π΅Ρ€ΠΈΡŽ?
— Π”ΠΆ. Π‘. Π‘Π­Π Π” (67 Π»Π΅Ρ‚)
Davao City,Philippines

A:

ΠŸΠΎΡ…ΠΎΠΆΠ΅, Π²Ρ‹ проситС Π΄ΠΈΠ°Π³Ρ€Π°ΠΌΠΌΡƒ Π€Π΅ΠΉΠ½ΠΌΠ°Π½Π° для прСдставлСния элСктромагнитных взаимодСйствий. Π’Ρ‹ ΠΌΠΎΠΆΠ΅Ρ‚Π΅ ΠΏΠΎΠ»ΡƒΡ‡ΠΈΡ‚ΡŒ это Π² обсуТдСнии Π² Π’ΠΈΠΊΠΈΠΏΠ΅Π΄ΠΈΠΈ:Β https://en.wikipedia.org/wiki/Quantum_electrodynamics.

А ΠΏΠΎΠΊΠ° я Π²ΠΎΡΠΏΠΎΠ»ΡŒΠ·ΡƒΡŽΡΡŒ случаСм, Ρ‡Ρ‚ΠΎΠ±Ρ‹ Π½Π΅ΠΌΠ½ΠΎΠ³ΠΎ ΠΈΠ·ΠΌΠ΅Π½ΠΈΡ‚ΡŒ ΠΏΡ€Π΅Π·Π΅Π½Ρ‚Π°Ρ†ΠΈΡŽ Π’ΠΎΠΌΠ°. ΠœΡ‹ ΠΎΠ±Ρ‹Ρ‡Π½ΠΎ Π³ΠΎΠ²ΠΎΡ€ΠΈΠΌ Ρ‡Ρ‚ΠΎ-Ρ‚ΠΎ Π²Ρ€ΠΎΠ΄Π΅ Β«Π²ΠΈΡ€Ρ‚ΡƒΠ°Π»ΡŒΠ½Ρ‹Π΅ фотоны… постоянно ΠΈΡΠΏΡƒΡΠΊΠ°ΡŽΡ‚ΡΡ ΠΈ ΠΏΠΎΠ²Ρ‚ΠΎΡ€Π½ΠΎ ΠΏΠΎΠ³Π»ΠΎΡ‰Π°ΡŽΡ‚ΡΡ элСктроном», Π½ΠΎ Π½Π° самом Π΄Π΅Π»Π΅ ΠΌΡ‹ ΠΈΠΌΠ΅Π΅ΠΌ Π² Π²ΠΈΠ΄Ρƒ Π½Π΅ это. Π”Π²Π΅ частицы, Π²Π·Π°ΠΈΠΌΠΎΠ΄Π΅ΠΉΡΡ‚Π²ΡƒΡŽΡ‰ΠΈΠ΅ элСктромагнитным ΠΎΠ±Ρ€Π°Π·ΠΎΠΌ, Π΄Π΅ΠΉΡΡ‚Π²ΠΈΡ‚Π΅Π»ΡŒΠ½ΠΎ ΠΎΠΊΡ€ΡƒΠΆΠ΅Π½Ρ‹ Π²ΠΈΡ€Ρ‚ΡƒΠ°Π»ΡŒΠ½Ρ‹ΠΌ Ρ„ΠΎΡ‚ΠΎΠ½Π½Ρ‹ΠΌ ΠΎΠ±Π»Π°ΠΊΠΎΠΌ. Однако Π² извСстных случаях (Π½Π°ΠΏΡ€ΠΈΠΌΠ΅Ρ€, Π°Ρ‚ΠΎΠΌ Π²ΠΎΠ΄ΠΎΡ€ΠΎΠ΄Π°) это ΠΎΠ±Π»Π°ΠΊΠΎ ΡΠΎΠ²Π΅Ρ€ΡˆΠ΅Π½Π½ΠΎ Π½Π΅ΠΈΠ·ΠΌΠ΅Π½Π½ΠΎ Π²ΠΎ Π²Ρ€Π΅ΠΌΠ΅Π½ΠΈ. Π’ΠΎΠΎΠ±Ρ‰Π΅ Π½ΠΈΡ‡Π΅Π³ΠΎ Π½Π΅ происходит. Π‘Π»ΠΎΠ²Π° ΠΎ ΠΊΠΎΠ»Π΅Π±Π»ΡŽΡ‰ΠΈΡ…ΡΡ Π²ΠΎΠΊΡ€ΡƒΠ³ Π²Π΅Ρ‰Π°Ρ… β€” Π³Ρ€ΡƒΠ±Ρ‹ΠΉ способ ΠΏΠ΅Ρ€Π΅Π΄Π°Ρ‚ΡŒ ΠΎΠ΄Π½ΠΎ ΠΈΠ· спСцифичСских свойств ΠΊΠ²Π°Π½Ρ‚ΠΎΠ²Ρ‹Ρ… ΠΏΠΎΠ»Π΅ΠΉ. ЭлСктричСскиС ΠΈ ΠΌΠ°Π³Π½ΠΈΡ‚Π½Ρ‹Π΅ поля ΠΈΠΌΠ΅ΡŽΡ‚ Π½Π΅ Ρ‚ΠΎΠ»ΡŒΠΊΠΎ срСдниС значСния, Π½ΠΎ ΠΈ Π΄ΠΈΠ°ΠΏΠ°Π·ΠΎΠ½Ρ‹ Π²ΠΎΠ·ΠΌΠΎΠΆΠ½Ρ‹Ρ… Π·Π½Π°Ρ‡Π΅Π½ΠΈΠΉ Π²ΠΎΠΊΡ€ΡƒΠ³ срСднСго. Π’ΠΎΡ‚ Ρ‡Ρ‚ΠΎ Ρ‚Π°ΠΊ отличаСтся ΠΎΡ‚ классичСских ΠΏΠΎΠ»Π΅ΠΉ. (Π­Ρ‚ΠΎ ΠΏΠΎΡ…ΠΎΠΆΠ΅ Π½Π° полоТСния ΠΊΠ²Π°Π½Ρ‚ΠΎΠ²Ρ‹Ρ… частиц, ΠΊΠΎΡ‚ΠΎΡ€Ρ‹Π΅ ΠΈΠΌΠ΅ΡŽΡ‚ Π΄ΠΈΠ°ΠΏΠ°Π·ΠΎΠ½ ΠΎΠΊΠΎΠ»ΠΎ срСднСго полоТСния, Π² ΠΎΡ‚Π»ΠΈΡ‡ΠΈΠ΅ ΠΎΡ‚ классичСских частиц. ) Π’Ρ‹ ΠΌΠΎΠΆΠ΅Ρ‚Π΅ ΠΏΠ΅Ρ€Π΅Π΄Π°Ρ‚ΡŒ ΠΎΠ±Ρ€Π°Π· этого Π΄ΠΈΠ°ΠΏΠ°Π·ΠΎΠ½Π°, ΠΏΡ€ΠΈΡ‚Π²ΠΎΡ€ΠΈΠ²ΡˆΠΈΡΡŒ, Ρ‡Ρ‚ΠΎ поля ΠΏΡ€Ρ‹Π³Π°ΡŽΡ‚ ΠΌΠ΅ΠΆΠ΄Ρƒ Ρ€Π°Π·Π»ΠΈΡ‡Π½Ρ‹ΠΌΠΈ Π²ΠΎΠ·ΠΌΠΎΠΆΠ½Ρ‹ΠΌΠΈ значСниями, Ρ‚ΠΎΡ‡Π½ΠΎ Ρ‚Π°ΠΊ ΠΆΠ΅, ΠΊΠ°ΠΊ Π²Ρ‹ ΠΌΠΎΠΆΠ΅Ρ‚Π΅ ΠΏΡ€Π΅Π΄ΡΡ‚Π°Π²ΠΈΡ‚ΡŒ Ρ‡Ρ‚ΠΎ частица ΠΏΡ€Ρ‹Π³Π°Π΅Ρ‚ ΠΌΠ΅ΠΆΠ΄Ρƒ Ρ€Π°Π·Π»ΠΈΡ‡Π½Ρ‹ΠΌΠΈ полоТСниями Π² своСм ΠΎΠ±Π»Π°ΠΊΠ΅. Однако поля (Π²ΠΊΠ»ΡŽΡ‡Π°Ρ ΠΈΡ… разброс Π·Π½Π°Ρ‡Π΅Π½ΠΈΠΉ) Π΄ΠΎΠ»ΠΆΠ½Ρ‹ ΠΏΡ€Ρ‹Π³Π°Ρ‚ΡŒ Π½Π΅ большС, Ρ‡Π΅ΠΌ частицы Π² пространствС. НапримСр, Π² Ρ…ΠΎΡ€ΠΎΡˆΠ΅ΠΌ ΡΡ‚Π°Π±ΠΈΠ»ΡŒΠ½ΠΎΠΌ Π°Ρ‚ΠΎΠΌΠ½ΠΎΠΌ состоянии Π½ΠΈΡ‡Π΅Π³ΠΎ Π½Π΅ мСняСтся Π²ΠΎ Π²Ρ€Π΅ΠΌΠ΅Π½ΠΈ. БтатичСский Π΄ΠΈΠ°ΠΏΠ°Π·ΠΎΠ½ возмоТностСй прСвращаСтся Π² Π΄Π΅ΠΉΡΡ‚Π²ΠΈΡ‚Π΅Π»ΡŒΠ½Ρ‹ΠΉ Π΄ΠΈΠ°ΠΏΠ°Π·ΠΎΠ½ Ρ€Π΅Π·ΡƒΠ»ΡŒΡ‚Π°Ρ‚ΠΎΠ² Ρ‚ΠΎΠ»ΡŒΠΊΠΎ Ρ‚ΠΎΠ³Π΄Π°, ΠΊΠΎΠ³Π΄Π° систСма ΠΎΠΏΡ€Π΅Π΄Π΅Π»Π΅Π½Π½Ρ‹ΠΌ ΠΎΠ±Ρ€Π°Π·ΠΎΠΌ взаимодСйствуСт с большим внСшним ΠΌΠΈΡ€ΠΎΠΌ.

Майк Π’.

(ΠΎΠΏΡƒΠ±Π»ΠΈΠΊΠΎΠ²Π°Π½ΠΎ 27.04.2011)

Π”ΠΎΠΏΠΎΠ»Π½Π΅Π½ΠΈΠ΅ β„–2: вСрнСмся ΠΊ мСханичСским полям МаксвСлла

Вопрос:

Π― Π΄Π΅ΠΉΡΡ‚Π²ΠΈΡ‚Π΅Π»ΡŒΠ½ΠΎ Ρ…ΠΎΡ‡Ρƒ Π·Π½Π°Ρ‚ΡŒ, ΠΈΠ· Ρ‡Π΅Π³ΠΎ состоит ΠΌΠ°Π³Π½ΠΈΡ‚Π½ΠΎΠ΅ ΠΏΠΎΠ»Π΅ ? ΠŸΠΎΠΆΠ°Π»ΡƒΠΉΡΡ‚Π°, Π½Π΅ ΠΈΡΠΏΠΎΠ»ΡŒΠ·ΡƒΠΉΡ‚Π΅ Ρ„ΠΎΡ‚ΠΎΠ½Ρ‹ Π² своСм ΠΎΡ‚Π²Π΅Ρ‚Π΅, ΠΏΠΎΡΠΊΠΎΠ»ΡŒΠΊΡƒ всС ΠΌΡ‹ Π·Π½Π°Π΅ΠΌ, Ρ‡Ρ‚ΠΎ ΠΌΠ°Π³Π½ΠΈΡ‚Π½Ρ‹Π΅ поля Π½Π΅ состоят ΠΈΠ· Ρ„ΠΎΡ‚ΠΎΠ½ΠΎΠ². На вопрос Дугласа Ρ‚Π°ΠΊ ΠΈ Π½Π΅ ΠΎΡ‚Π²Π΅Ρ‚ΠΈΠ»ΠΈ. Дуглас Π½Π΅ ΡΠΏΡ€Π°ΡˆΠΈΠ²Π°Π» ΠΎΠ± элСктромагнитном ΠΏΠΎΠ»Π΅, ΠΊΠΎΡ‚ΠΎΡ€ΠΎΠ΅ Π½Π°ΠΈΠ±ΠΎΠ»Π΅Π΅ тСсно связано с Ρ„ΠΎΡ‚ΠΎΠ½ΠΎΠΌ. ΠŸΡ€Π΅Π΄ΠΎΡΡ‚Π°Π²Π»Π΅Π½Π½Ρ‹Π΅ ΠΎΡ‚Π²Π΅Ρ‚Ρ‹ Π±Ρ‹Π»ΠΈ Π½Π΅ΠΏΠΎΠ»Π½Ρ‹ΠΌΠΈ ΠΈ Π½Π΅ ΠΎΡ‚Π½ΠΎΡΠΈΠ»ΠΈΡΡŒ ΠΊ вопросу Дугласа. Из Ρ‡Π΅Π³ΠΎ состоит ΠΌΠ°Π³Π½ΠΈΡ‚Π½ΠΎΠ΅ ΠΏΠΎΠ»Π΅? Π’ΠΎΡ‚ Ρ‡Ρ‚ΠΎ соврСмСнная Π½Π°ΡƒΠΊΠ° Π·Π½Π°Π΅Ρ‚ ΠΎ ΠΌΠ°Π³Π½ΠΈΡ‚Π½Ρ‹Ρ… полях. ЧСстный ΠΎΡ‚Π²Π΅Ρ‚: ΠΌΡ‹ Π½Π΅ Π·Π½Π°Π΅ΠΌ, Ρ‡Ρ‚ΠΎ Ρ‚Π°ΠΊΠΎΠ΅ ΠΌΠ°Π³Π½ΠΈΡ‚Π½ΠΎΠ΅ ΠΏΠΎΠ»Π΅. Π§Ρ‚ΠΎ ΠΌΡ‹ Π·Π½Π°Π΅ΠΌ, Ρ‚Π°ΠΊ это Ρ‚ΠΎ, Ρ‡Ρ‚ΠΎ ΠΏΠΎΠ»Π΅ ΠœΠ°Π³Π½ΠΈΡ‚Π° гСнСрируСтся Π΄Π²ΠΈΠΆΠ΅Π½ΠΈΠ΅ΠΌ элСктростатичСских зарядов Π²Π½ΡƒΡ‚Ρ€ΠΈ самого ΠΌΠ°Π³Π½ΠΈΡ‚Π°. ЭлСктричСскиС заряды — это элСктроны. Π­Π»Π΅ΠΊΡ‚Ρ€ΠΎΠ½Ρ‹ двиТутся ΠΊΠΎΠ³Π΅Ρ€Π΅Π½Ρ‚Π½ΠΎ ΠΈ синхронно, Ρ‡Ρ‚ΠΎ Π²Ρ‹Π·Ρ‹Π²Π°Π΅Ρ‚ выброс сильного ΠΌΠ°Π³Π½ΠΈΡ‚Π½ΠΎΠ³ΠΎ поля ΠΈΠ· ΠΌΠ°Π³Π½ΠΈΡ‚Π°. Π§Π΅Π³ΠΎ ΠΌΡ‹ Π½Π΅ Π·Π½Π°Π΅ΠΌ, Ρ‚Π°ΠΊ это Ρ‚ΠΎΠ³ΠΎ, ΠΈΠ· Ρ‡Π΅Π³ΠΎ состоит это ΠΏΠΎΠ»Π΅. НСкоторыС люди Π³ΠΎΠ²ΠΎΡ€ΠΈΠ»ΠΈ, Ρ‡Ρ‚ΠΎ ΠΎΠ½ состоит ΠΈΠ· ΠΌΠ°Π³Π½ΠΈΡ‚Π½Ρ‹Ρ… ΠΌΠΎΠ½ΠΎΠΏΠΎΠ»Π΅ΠΉ. ΠœΠ°Π³Π½ΠΈΡ‚Π½Ρ‹Π΅ ΠΌΠΎΠ½ΠΎΠΏΠΎΠ»ΠΈ Π½ΠΈΠΊΠΎΠ³Π΄Π° Π½Π΅ ΠΎΡ‚ΠΊΡ€Ρ‹Π²Π°Π»ΠΈΡΡŒ, Ρ‚Π°ΠΊ Ρ‡Ρ‚ΠΎ Π΅ΡΡ‚ΡŒ большая Π²Π΅Ρ€ΠΎΡΡ‚Π½ΠΎΡΡ‚ΡŒ, ΠΎΡ‡Π΅Π½ΡŒ большая Π²Π΅Ρ€ΠΎΡΡ‚Π½ΠΎΡΡ‚ΡŒ Ρ‚ΠΎΠ³ΠΎ, Ρ‡Ρ‚ΠΎ тСория Π½Π΅Π²Π΅Ρ€Π½Π°. Π― ΡΡ‡ΠΈΡ‚Π°ΡŽ, Ρ‡Ρ‚ΠΎ ΠΌΠ°Π³Π½ΠΈΡ‚Π½ΠΎΠ΅ ΠΏΠΎΠ»Π΅ Π²ΠΎΠΎΠ±Ρ‰Π΅ Π½Π΅ состоит ΠΈΠ· поля ΠΊΠ°ΠΊΠΎΠΉ-Π»ΠΈΠ±ΠΎ частицы. Дуглас, Π΄ΡƒΠΌΠ°ΠΉΡ‚Π΅ ΠΎ ΠΌΠ°Π³Π½ΠΈΡ‚Π½ΠΎΠΌ ΠΏΠΎΠ»Π΅ ΠΊΠ°ΠΊ ΠΎ прямой Π΄Π΅Ρ„ΠΎΡ€ΠΌΠ°Ρ†ΠΈΠΈ физичСского пространства. Π’Π°ΠΊ Π΄ΠΎΠ»ΠΆΠ½Ρ‹ Ρ€Π°Π±ΠΎΡ‚Π°Ρ‚ΡŒ всС чистыС поля. Π­Ρ‚ΠΎ Π΄ΠΎΠ»ΠΆΠ½Ρ‹ Π±Ρ‹Ρ‚ΡŒ мСханичСскиС Π΄Π΅Ρ„ΠΎΡ€ΠΌΠ°Ρ†ΠΈΠΈ пространства. Π’Ρ‹ ΠΌΠΎΠΆΠ΅Ρ‚Π΅ Π΄ΡƒΠΌΠ°Ρ‚ΡŒ ΠΎ пространствС ΠΊΠ°ΠΊ ΠΎ Ρ‚Π²Π΅Ρ€Π΄ΠΎΠΉ ΡƒΠΏΡ€ΡƒΠ³ΠΎΠΉ Ρ‚ΠΊΠ°Π½ΠΈ с Π½ΠΈΠ·ΠΊΠΎΠΉ ΠΏΠ»ΠΎΡ‚Π½ΠΎΡΡ‚ΡŒΡŽ ΠΈ высоким натяТСниСм. ΠœΠ°Π³Π½ΠΈΡ‚Π½ΠΎΠ΅ ΠΏΠΎΠ»Π΅ Π΅ΡΡ‚ΡŒ мСханичСская дСформация самого пространства. Π― Π±Ρ‹ Ρ…ΠΎΡ‚Π΅Π», Ρ‡Ρ‚ΠΎΠ±Ρ‹ Π²Ρ‹ Π½Π°ΠΏΠ΅Ρ‡Π°Ρ‚Π°Π»ΠΈ этот вопрос ΠΈ ΠΎΡ‚Π²Π΅Ρ‚ΠΈΠ»ΠΈ, Π½ΠΎ ΠΌΡ‹ ΠΎΠ±Π° Π·Π½Π°Π΅ΠΌ, Ρ‡Ρ‚ΠΎ Π²Ρ‹ этого Π½Π΅ сдСлаСтС.
— ΠœΠ°Ρ€ΠΊ (58 Π»Π΅Ρ‚)
Π€Π»ΠΎΡ€ΠΈΠ΄Π°

A:

Когда МаксвСлл Π²ΠΏΠ΅Ρ€Π²Ρ‹Π΅ ΠΏΡ€ΠΈΠ΄ΡƒΠΌΠ°Π» свои Π·Π½Π°ΠΌΠ΅Π½ΠΈΡ‚Ρ‹Π΅ уравнСния элСктромагнСтизма, ΠΎΠ½ попытался ΡΠ΄Π΅Π»Π°Ρ‚ΡŒ ΠΌΠ΅Ρ…Π°Π½ΠΈΡ‡Π΅ΡΠΊΡƒΡŽ модСль с малСнькими ΡˆΠ΅ΡΡ‚Π΅Ρ€Π½ΡΠΌΠΈ, колСсами ΠΈ ΠΏΡ€ΠΎΡ‡ΠΈΠΌ. ΠžΡ‚ Π½Π΅Π³ΠΎ быстро ΠΎΡ‚ΠΊΠ°Π·Π°Π»ΠΈΡΡŒ, ΠΏΠΎΡΠΊΠΎΠ»ΡŒΠΊΡƒ ΠΎΠ½ Π½Π΅ добавлял Π½ΠΈΡ‡Π΅Π³ΠΎ, ΠΊΡ€ΠΎΠΌΠ΅ услоТнСния ΡƒΡ€Π°Π²Π½Π΅Π½ΠΈΠΉ поля.

Π£ вас Π΅ΡΡ‚ΡŒ Ρ€Π°Π·Π»ΠΈΡ‡Π½Ρ‹Π΅ словСсныС утвСрТдСния ΠΎ Ρ‚ΠΎΠΌ, Ρ‡Ρ‚ΠΎ Ρ‚Π°ΠΊΠΎΠ΅ ΠΏΠΎΠ»Π΅ «Π½Π° самом Π΄Π΅Π»Π΅». Π’Ρ‹ Π³ΠΎΠ²ΠΎΡ€ΠΈΡ‚Π΅, Ρ‡Ρ‚ΠΎ ΠΎΠ½ сдСлан Π½Π΅ ΠΈΠ· ΠΌΠΎΠ½ΠΎΠΏΠΎΠ»Π΅ΠΉ, Π½ΠΎ я Π½ΠΈΠΊΠΎΠ³Π΄Π° Π½Π΅ ΡΠ»Ρ‹ΡˆΠ°Π», Ρ‡Ρ‚ΠΎΠ±Ρ‹ ΠΊΡ‚ΠΎ-Ρ‚ΠΎ ΠΏΡ€Π΅Π΄ΠΏΠΎΠ»Π°Π³Π°Π», Ρ‡Ρ‚ΠΎ это Ρ‚Π°ΠΊ. Π’Ρ‹ Π³ΠΎΠ²ΠΎΡ€ΠΈΡ‚Π΅, Ρ‡Ρ‚ΠΎ это Π½Π΅ ΠΈΠΌΠ΅Π΅Ρ‚ Π½ΠΈΡ‡Π΅Π³ΠΎ ΠΎΠ±Ρ‰Π΅Π³ΠΎ с Ρ„ΠΎΡ‚ΠΎΠ½Π°ΠΌΠΈ, Ρ‚ΠΎ Π΅ΡΡ‚ΡŒ с ΠΊΠ²Π°Π½Ρ‚ΠΎΠ²ΠΎΠΉ ΠΌΠ΅Ρ…Π°Π½ΠΈΠΊΠΎΠΉ. Π’Π°ΡˆΠ° ΠΊΠ°Ρ€Ρ‚ΠΈΠ½Π° Π·Π²ΡƒΡ‡ΠΈΡ‚ Ρ‚Π°ΠΊ, Π±ΡƒΠ΄Ρ‚ΠΎ Π½Π΅ соотвСтствуСт ΡΠΏΠ΅Ρ†ΠΈΠ°Π»ΡŒΠ½ΠΎΠΉ Ρ‚Π΅ΠΎΡ€ΠΈΠΈ ΠΎΡ‚Π½ΠΎΡΠΈΡ‚Π΅Π»ΡŒΠ½ΠΎΡΡ‚ΠΈ.

Π‘ΠΎΡ‡Π΅Ρ‚Π°Π½ΠΈΠ΅ ΡΠΏΠ΅Ρ†ΠΈΠ°Π»ΡŒΠ½ΠΎΠΉ Ρ‚Π΅ΠΎΡ€ΠΈΠΈ ΠΎΡ‚Π½ΠΎΡΠΈΡ‚Π΅Π»ΡŒΠ½ΠΎΡΡ‚ΠΈ ΠΈ ΠΊΠ²Π°Π½Ρ‚ΠΎΠ²ΠΎΠΉ ΠΌΠ΅Ρ…Π°Π½ΠΈΠΊΠΈ позволяСт ΠΏΡ€ΠΎΠ²ΠΎΠ΄ΠΈΡ‚ΡŒ расчСты Π²Π΅Ρ‰Π΅ΠΉ, ΠΊΠΎΡ‚ΠΎΡ€Ρ‹Π΅ ΠΌΠΎΠΆΠ½ΠΎ ΠΈΠ·ΠΌΠ΅Ρ€ΠΈΡ‚ΡŒ. НапримСр, ΠΎΠ½ Π΄Π°Π΅Ρ‚ прСдсказаниС Π³ΠΈΡ€ΠΎΠΌΠ°Π³Π½ΠΈΡ‚Π½ΠΎΠ³ΠΎ ΠΎΡ‚Π½ΠΎΡˆΠ΅Π½ΠΈΡ элСктронов. Π­ΠΊΡΠΏΠ΅Ρ€ΠΈΠΌΠ΅Π½Ρ‚Π°Π»ΡŒΠ½ΠΎ Π·Π½Π°Ρ‡Π΅Π½ΠΈΠ΅ Ρ€Π°Π²Π½ΠΎ 2,00231930462 с нСбольшой ΠΏΠΎΠ³Ρ€Π΅ΡˆΠ½ΠΎΡΡ‚ΡŒΡŽ Π² послСднСм дСсятичном разрядС. «ΠŸΡ€Π΅Π΄ΡΠΊΠ°Π·Π°Π½ΠΈΠ΅ ΠšΠ­Π” согласуСтся с ΡΠΊΡΠΏΠ΅Ρ€ΠΈΠΌΠ΅Π½Ρ‚Π°Π»ΡŒΠ½ΠΎ ΠΈΠ·ΠΌΠ΅Ρ€Π΅Π½Π½Ρ‹ΠΌ Π·Π½Π°Ρ‡Π΅Π½ΠΈΠ΅ΠΌ Π±ΠΎΠ»Π΅Π΅ Ρ‡Π΅ΠΌ Π½Π° 10Β Π·Π½Π°Ρ‡Π°Ρ‰ΠΈΡ… Ρ†ΠΈΡ„Ρ€…» Β Β 

КакоС Π·Π½Π°Ρ‡Π΅Π½ΠΈΠ΅ Π΄Π°Π΅Ρ‚ ваша модСль?

Майк Π’.

ΡˆΡ‚. Π― Π½Π΅ ΠΌΠΎΠ³Ρƒ ΡƒΠ΄Π΅Ρ€ΠΆΠ°Ρ‚ΡŒΡΡ ΠΎΡ‚ Ρ‚ΠΎΠ³ΠΎ, Ρ‡Ρ‚ΠΎΠ±Ρ‹ Π½Π΅ привСсти ΠΏΠ΅Ρ€Π²Ρ‹Π΅ Π΄Π²Π΅ строчки ΠΈΠ· ΠΏΡ€ΠΈΠΌΠ΅Ρ€Π½ΠΎ 100-страничного ΠΈΠ½Ρ‚Π΅Ρ€Π²ΡŒΡŽ, ΠΊΠΎΡ‚ΠΎΡ€ΠΎΠ΅ Π€ΠΎΠ½Π΄ химичСского наслСдия ΠΏΡ€ΠΎΠ²Π΅Π» с ΡƒΡ‡Π΅Π½Ρ‹ΠΌ (ΠΌΠΎΠΉ ΠΎΡ‚Π΅Ρ†), ΠΊΠΎΡ‚ΠΎΡ€Ρ‹ΠΉ Ρ€Π°Π±ΠΎΡ‚Π°Π» с ΠΌΠ°Π³Π½ΠΈΡ‚Π°ΠΌΠΈ Π² Ρ‚Π΅Ρ‡Π΅Π½ΠΈΠ΅ ~ 9 Π»Π΅Ρ‚.0 Π»Π΅Ρ‚. Β«Π Π΅Π±Π΅Π½ΠΊΠΎΠΌ я ΠΎΠ±Π½Π°Ρ€ΡƒΠΆΠΈΠ», Ρ‡Ρ‚ΠΎ ΠΌΠΎΠ³Ρƒ Π·Π°ΡΡ‚Π°Π²ΠΈΡ‚ΡŒ Π±ΡƒΠ»Π°Π²ΠΊΠΈ Π² ΠΊΠΎΡ€ΠΎΠ±ΠΊΠ΅ ΡΡ‚ΠΎΡΡ‚ΡŒ, ΠΈ, пСрСмСщая ΠΌΠ°Π³Π½ΠΈΡ‚, я ΠΌΠΎΠ³Ρƒ Π·Π°ΡΡ‚Π°Π²ΠΈΡ‚ΡŒ ΠΈΡ… Π΄Π²ΠΈΠ³Π°Ρ‚ΡŒΡΡ. Π― понятия Π½Π΅ ΠΈΠΌΠ΅Π», Ρ‡Ρ‚ΠΎ Ρ‚Π°ΠΊΠΎΠ΅ ΠΌΠ°Π³Π½ΠΈΡ‚Π½ΠΎΠ΅ ΠΏΠΎΠ»Π΅, ΠΈ ΠΏΠΎΠ΄ΠΎΠ·Ρ€Π΅Π²Π°ΡŽ, Ρ‡Ρ‚ΠΎ Π΄ΠΎ сих ΠΏΠΎΡ€ понятия Π½Π΅ имСю, Ρ‡Ρ‚ΠΎ Ρ‚Π°ΠΊΠΎΠ΅ ΠΌΠ°Π³Π½ΠΈΡ‚Π½ΠΎΠ΅ ΠΏΠΎΠ»Π΅. ΠΌΠ°Π³Π½ΠΈΡ‚Π½ΠΎΠ΅ ΠΏΠΎΠ»Π΅ Π΅ΡΡ‚ΡŒ, Π·Π° ΠΈΡΠΊΠ»ΡŽΡ‡Π΅Π½ΠΈΠ΅ΠΌ Π½Π΅ΠΊΠΎΡ‚ΠΎΡ€Ρ‹Ρ… Π²Π΅Ρ‰Π΅ΠΉ, ΠΊΠΎΡ‚ΠΎΡ€Ρ‹Π΅ ΠΎΠ½ΠΎ Π΄Π΅Π»Π°Π΅Ρ‚Β».

(ΠΎΠΏΡƒΠ±Π»ΠΈΠΊΠΎΠ²Π°Π½ΠΎ 04.09.2013)

Π”ΠΎΠΏΠΎΠ»Π½Π΅Π½ΠΈΠ΅ β„–3: элСктричСскиС поля ΠΊΠ°ΠΊ Π²ΠΈΡ€Ρ‚ΡƒΠ°Π»ΡŒΠ½Ρ‹Π΅ Ρ„ΠΎΡ‚ΠΎΠ½Π½Ρ‹Π΅ ΠΎΠ±Π»Π°ΠΊΠ°

Вопрос:

ΠšΠ°ΠΆΠ΅Ρ‚ΡΡ, Ρ‡Ρ‚ΠΎ Π²ΠΈΡ€Ρ‚ΡƒΠ°Π»ΡŒΠ½ΠΎΠ΅ Ρ„ΠΎΡ‚ΠΎΠ½Π½ΠΎΠ΅ ΠΎΠ±Π»Π°ΠΊΠΎ сущСствуСт Ρ‚ΠΎΠ»ΡŒΠΊΠΎ ΠΎΡ‡Π΅Π½ΡŒ Π±Π»ΠΈΠ·ΠΊΠΎ ΠΊ статичСскому заряду. Как ΠΌΡ‹ ΠΏΡ€ΠΈΠΌΠΈΡ€ΠΈΠΌ это с (Π°) Ρ‚Π΅ΠΌ Ρ„Π°ΠΊΡ‚ΠΎΠΌ, Ρ‡Ρ‚ΠΎ силовыС Π»ΠΈΠ½ΠΈΠΈ элСктричСского поля тянутся Π²Π΅Ρ‡Π½ΠΎ ΠΈ (Π±) Ρ‚Π΅ΠΌ Ρ„Π°ΠΊΡ‚ΠΎΠΌ, Ρ‡Ρ‚ΠΎ Π²ΠΈΡ€Ρ‚ΡƒΠ°Π»ΡŒΠ½Ρ‹Π΅ Ρ„ΠΎΡ‚ΠΎΠ½Ρ‹ ΠΎΠ±ΠΌΠ΅Π½ΠΈΠ²Π°ΡŽΡ‚ΡΡ Π½Π° расстояниях, ΠΏΡ€Π΅Π²Ρ‹ΡˆΠ°ΡŽΡ‰ΠΈΡ… расстояниС Π²ΠΈΡ€Ρ‚ΡƒΠ°Π»ΡŒΠ½ΠΎΠ³ΠΎ Ρ„ΠΎΡ‚ΠΎΠ½Π½ΠΎΠ³ΠΎ ΠΎΠ±Π»Π°ΠΊΠ°? Π― ΠΏΡ€Π΅Π΄ΡΡ‚Π°Π²Π»ΡΡŽ сСбС Π²ΠΈΡ€Ρ‚ΡƒΠ°Π»ΡŒΠ½ΠΎΠ΅ Ρ„ΠΎΡ‚ΠΎΠ½Π½ΠΎΠ΅ ΠΎΠ±Π»Π°ΠΊΠΎ, ΠΎΠΊΡ€ΡƒΠΆΠ°ΡŽΡ‰Π΅Π΅ ΠΈΠ·ΠΎΠ»ΠΈΡ€ΠΎΠ²Π°Π½Π½Ρ‹ΠΉ элСктрон. сдача. Π’Π½Π΅Π·Π°ΠΏΠ½ΠΎ ΠΏΡ€ΠΎΡ‚ΠΎΠ½ оказываСтся рядом с элСктроном. ΠŸΡ€Π΅ΠΆΠ΄Π΅ Ρ‡Π΅ΠΌ ΠΏΡ€ΠΎΡ‚ΠΎΠ½ ΠΏΡ€ΠΈΠ±ΡƒΠ΄Π΅Ρ‚ Π½Π° сцСну, Π²ΠΈΡ€Ρ‚ΡƒΠ°Π»ΡŒΠ½ΠΎΠ΅ Ρ„ΠΎΡ‚ΠΎΠ½Π½ΠΎΠ΅ ΠΎΠ±Π»Π°ΠΊΠΎ ΠΏΠ»ΠΎΡ‚Π½ΠΎ упаковываСтся Π²ΠΎΠΊΡ€ΡƒΠ³ элСктрона. (Π’ΠΈΡ€Ρ‚ΡƒΠ°Π»ΡŒΠ½ΠΎΠ΅ Ρ„ΠΎΡ‚ΠΎΠ½Π½ΠΎΠ΅ ΠΎΠ±Π»Π°ΠΊΠΎ прСдставляСт собой статичСскоС ΠΊΠ²Π°Π½Ρ‚ΠΎΠ²ΠΎΠ΅ состояниС, Π½ΠΎ это Π½Π΅ ΠΎΠ·Π½Π°Ρ‡Π°Π΅Ρ‚, Ρ‡Ρ‚ΠΎ Π²ΠΈΡ€Ρ‚ΡƒΠ°Π»ΡŒΠ½Ρ‹Π΅ Ρ„ΠΎΡ‚ΠΎΠ½Ρ‹ Π½Π΅ двиТутся β€” ΠΏΠΎΠ΄ΠΎΠ±Π½ΠΎ элСктронному ΠΎΠ±Π»Π°ΠΊΡƒ Π²ΠΎΠΊΡ€ΡƒΠ³ Π°Ρ‚ΠΎΠΌΠ° Π²ΠΎΠ΄ΠΎΡ€ΠΎΠ΄Π°). Π²ΠΈΡ€Ρ‚ΡƒΠ°Π»ΡŒΠ½ΠΎΠ΅ Ρ„ΠΎΡ‚ΠΎΠ½Π½ΠΎΠ΅ ΠΎΠ±Π»Π°ΠΊΠΎ мСняСт Ρ„ΠΎΡ€ΠΌΡƒ? НапримСр, «растягиваСтся Π»ΠΈΒ» Π²ΠΈΡ€Ρ‚ΡƒΠ°Π»ΡŒΠ½ΠΎΠ΅ Ρ„ΠΎΡ‚ΠΎΠ½Π½ΠΎΠ΅ ΠΎΠ±Π»Π°ΠΊΠΎ, Ρ‡Ρ‚ΠΎΠ±Ρ‹ Π΄ΠΎΡΡ‚ΠΈΡ‡ΡŒ ΠΏΡ€ΠΎΡ‚ΠΎΠ½Π°, прСдставляя ΠΏΠΎΠ²Ρ‹ΡˆΠ΅Π½Π½ΠΎΠ΅ присутствиС Π²ΠΈΡ€Ρ‚ΡƒΠ°Π»ΡŒΠ½Ρ‹Ρ… Ρ„ΠΎΡ‚ΠΎΠ½ΠΎΠ² Π½Π° ΠΏΡƒΡ‚ΠΈ, Π³Π΄Π΅ ΠΎΠ½ΠΈ ΠΎΠ±ΠΌΠ΅Π½ΠΈΠ²Π°ΡŽΡ‚ΡΡ ΠΌΠ΅ΠΆΠ΄Ρƒ двумя заряТСнными частицами?
— JD (29 Π»Π΅Ρ‚)
Луисвилл, ΠšΠ΅Π½Ρ‚ΡƒΠΊΠΊΠΈ, БША

A:

ΠšΠ»Π°ΡΡΠΈΡ‡Π΅ΡΠΊΠΎΠ΅ Π²Ρ‹Ρ€Π°ΠΆΠ΅Π½ΠΈΠ΅ поля Π³ΠΎΠ²ΠΎΡ€ΠΈΡ‚ Π²Π°ΠΌ, ΠΊΠ°ΠΊ распространяСтся Π²ΠΈΡ€Ρ‚ΡƒΠ°Π»ΡŒΠ½ΠΎΠ΅ Ρ„ΠΎΡ‚ΠΎΠ½Π½ΠΎΠ΅ ΠΎΠ±Π»Π°ΠΊΠΎ. Π’Π°ΠΊΠΈΠΌ ΠΎΠ±Ρ€Π°Π·ΠΎΠΌ, эти Π²ΠΈΡ€Ρ‚ΡƒΠ°Π»ΡŒΠ½Ρ‹Π΅ Ρ„ΠΎΡ‚ΠΎΠ½Ρ‹ Π½Π΅ Π±ΠΎΠ»Π΅Π΅ ΠΈ Π½Π΅ ΠΌΠ΅Π½Π΅Π΅ сконцСнтрированы Π½Π° зарядС, Ρ‡Π΅ΠΌ классичСскиС поля. ИзмСнСния Ρ„ΠΎΡ€ΠΌΡ‹, ΠΊΠΎΠ³Π΄Π° ΠΏΡ€ΠΈΡΡƒΡ‚ΡΡ‚Π²ΡƒΡŽΡ‚ два заряда, ΠΎΠΏΡ€Π΅Π΄Π΅Π»ΡΡŽΡ‚ΡΡ суммированиСм классичСских Π²Π΅ΠΊΡ‚ΠΎΡ€Π½Ρ‹Ρ… ΠΏΠΎΠ»Π΅ΠΉ. Π’Π°ΠΊ Ρ‡Ρ‚ΠΎ Π΄Π°, ΠΌΠ΅ΠΆΠ΄Ρƒ зарядами сущСствуСт особСнно сильноС ΠΏΠΎΠ»Π΅, Π½ΠΎ Π΄Π°Π»Π΅ΠΊΠΎ ΠΎΡ‚ Π½ΠΈΡ… поля ΠΈΠΌΠ΅ΡŽΡ‚ Ρ‚Π΅Π½Π΄Π΅Π½Ρ†ΠΈΡŽ Π³Π°ΡΠΈΡ‚ΡŒΡΡ.

Майк Π’.

(ΠΎΠΏΡƒΠ±Π»ΠΈΠΊΠΎΠ²Π°Π½ΠΎ 11.04.2015)

Π”ΠΎΠΏΠΎΠ»Π½Π΅Π½ΠΈΠ΅ β„–4: Ρ„ΠΎΡ‚ΠΎΠ½Ρ‹ ΠΈ ΠΌΠ°Π³Π½ΠΈΡ‚Π½Ρ‹Π΅ поля

Вопрос:

ΠœΠ°Π³Π½ΠΈΡ‚Π½Ρ‹Π΅ поля нСльзя ΠΎΠ±ΡŠΡΡΠ½ΠΈΡ‚ΡŒ, просто сказав, Ρ‡Ρ‚ΠΎ ΠΎΠ½ΠΈ состоят ΠΈΠ· Β«Ρ„ΠΎΡ‚ΠΎΠ½ΠΎΠ²Β». Какая Π΄Π°Ρ‚Π° ΠΈΡ… физичСского макияТа?
— Π ΠΎΠ±Π΅Ρ€Ρ‚ ПонсС (69 Π»Π΅Ρ‚)
Port Hueneme

A:

Π’ ΠΎΠ΄Π½ΠΎΠΌ смыслС Π²Ρ‹ ΠΏΡ€Π°Π²Ρ‹, Ρ‡Ρ‚ΠΎ Π½Π΅ Π΄ΠΎΠ»ΠΆΠ½Ρ‹ Π΄ΡƒΠΌΠ°Ρ‚ΡŒ, Ρ‡Ρ‚ΠΎ ΠΌΠ°Π³Π½ΠΈΡ‚Π½Ρ‹Π΅ поля состоят ΠΈΠ· Ρ„ΠΎΡ‚ΠΎΠ½ΠΎΠ². Если Ρƒ вас Π΅ΡΡ‚ΡŒ ΠΎΠΏΡ€Π΅Π΄Π΅Π»Π΅Π½Π½ΠΎΠ΅ количСство Ρ„ΠΎΡ‚ΠΎΠ½ΠΎΠ² ΠΊΠ°ΠΆΠ΄ΠΎΠ³ΠΎ Ρ‚ΠΈΠΏΠ°, ΠΎΠΆΠΈΠ΄Π°Π΅ΠΌΠΎΠ΅ ΠΌΠ°Π³Π½ΠΈΡ‚Π½ΠΎΠ΅ ΠΏΠΎΠ»Π΅ Ρ€Π°Π²Π½ΠΎ Π½ΡƒΠ»ΡŽ. Π§Ρ‚ΠΎΠ±Ρ‹ ΠΏΠΎΠ»ΡƒΡ‡ΠΈΡ‚ΡŒ Ρ‡Ρ‚ΠΎ-Ρ‚ΠΎ Π²Ρ€ΠΎΠ΄Π΅ классичСского Ρ‡Π΅Ρ‚ΠΊΠΎ ΠΎΠΏΡ€Π΅Π΄Π΅Π»Π΅Π½Π½ΠΎΠ³ΠΎ ΠΌΠ°Π³Π½ΠΈΡ‚Π½ΠΎΠ³ΠΎ поля, Π²Π°ΠΌ Π½ΡƒΠΆΠ΅Π½ большой разброс Π²ΠΎΠ·ΠΌΠΎΠΆΠ½Ρ‹Ρ… чисСл Ρ„ΠΎΡ‚ΠΎΠ½ΠΎΠ².* Π­Ρ‚ΠΎ сильно отличаСтся ΠΎΡ‚ прСдполоТСния, Ρ‡Ρ‚ΠΎ ΠΊΠ°ΠΆΠ΄Ρ‹ΠΉ Ρ„ΠΎΡ‚ΠΎΠ½ вносит ΠΊΠ°ΠΊΠΎΠ΅-Ρ‚ΠΎ ΠΏΠΎΠ»Π΅.

Π‘ Π΄Ρ€ΡƒΠ³ΠΎΠΉ стороны, Π²ΠΎΠ·ΠΌΠΎΠΆΠ½ΠΎ, Π²Ρ‹ Π΄ΡƒΠΌΠ°Π΅Ρ‚Π΅, Ρ‡Ρ‚ΠΎ ΠΏΠΎΠΌΠΈΠΌΠΎ Ρ„ΠΎΡ‚ΠΎΠ½ΠΎΠ² Π΅ΡΡ‚ΡŒ Π΅Ρ‰Π΅ ΠΊΠ°ΠΊΠΎΠΉ-Ρ‚ΠΎ ΠΈΠ½Π³Ρ€Π΅Π΄ΠΈΠ΅Π½Ρ‚. НСт.

Майк Π’.

*Если Π²Ρ‹ посмотритС, Π½Π°ΠΏΡ€ΠΈΠΌΠ΅Ρ€, Π½Π° ΡƒΡ€Π°Π²Π½Π΅Π½ΠΈΠ΅Β 21 Π² этой ΡΡ‚Π°Ρ‚ΡŒΠ΅ (https://www.phys.ksu.edu/personal/wysin/notes/quantumEM.pdf), Π²Ρ‹ Π½Π°ΠΉΠ΄Π΅Ρ‚Π΅ Π²Ρ‹Ρ€Π°ΠΆΠ΅Π½ΠΈΠ΅ для ΠΌΠ°Π³Π½ΠΈΡ‚Π½ΠΎΠ΅ ΠΏΠΎΠ»Π΅ Π² Ρ‚Π΅Ρ€ΠΌΠΈΠ½Π°Ρ… ΠΎΠΏΠ΅Ρ€Π°Ρ‚ΠΎΡ€ΠΎΠ² роТдСния ΠΈ уничтоТСния Ρ„ΠΎΡ‚ΠΎΠ½ΠΎΠ².

(ΠΎΠΏΡƒΠ±Π»ΠΈΠΊΠΎΠ²Π°Π½ΠΎ 13.12.2015)

Π”ΠΎΠΏΠΎΠ»Π½Π΅Π½ΠΈΠ΅ β„– 5: Ρ„ΠΎΡ‚ΠΎΠ½Ρ‹ ΠΈ ΠΌΠ°Π³Π½Π΅Ρ‚ΠΈΠ·ΠΌ ΠΏΠΎΠ»Π΅ Ρ€Π°Π²Π½ΠΎ Π½ΡƒΠ»ΡŽ. Π§Ρ‚ΠΎΠ±Ρ‹ ΠΏΠΎΠ»ΡƒΡ‡ΠΈΡ‚ΡŒ Ρ‡Ρ‚ΠΎ-Ρ‚ΠΎ Π²Ρ€ΠΎΠ΄Π΅ классичСского Ρ‡Π΅Ρ‚ΠΊΠΎ ΠΎΠΏΡ€Π΅Π΄Π΅Π»Π΅Π½Π½ΠΎΠ³ΠΎ ΠΌΠ°Π³Π½ΠΈΡ‚Π½ΠΎΠ³ΠΎ поля, Π²Π°ΠΌ Π½ΡƒΠΆΠ΅Π½ большой разброс Π²ΠΎΠ·ΠΌΠΎΠΆΠ½Ρ‹Ρ… чисСл Ρ„ΠΎΡ‚ΠΎΠ½ΠΎΠ²Β». ΠΌΠ΅Ρ…Π°Π½ΠΈΠΊΠ°. Π­Ρ‚Π° Ρ‚Π΅ΠΌΠ° сбиваСт мСня с Ρ‚ΠΎΠ»ΠΊΡƒ, ΠΏΠΎΡ‚ΠΎΠΌΡƒ Ρ‡Ρ‚ΠΎ я Π΄ΡƒΠΌΠ°ΡŽ, Ρ‡Ρ‚ΠΎ Ρƒ мСня Π΅ΡΡ‚ΡŒ Π±Π°Π·ΠΎΠ²ΠΎΠ΅ ΠΏΠΎΠ½ΠΈΠΌΠ°Π½ΠΈΠ΅ Β«ΠΎΠ±Ρ‹Ρ‡Π½Ρ‹Ρ…Β» Ρ„ΠΎΡ‚ΠΎΠ½ΠΎΠ² (Π½Π°ΠΏΡ€ΠΈΠΌΠ΅Ρ€, Π²ΠΈΠ΄ΠΈΠΌΠΎΠ³ΠΎ свСта), Π½ΠΎ Ρ‚Π°ΠΊΠΈΠ΅ Ρ„ΠΎΡ‚ΠΎΠ½Ρ‹ ΠΈΠΌΠ΅ΡŽΡ‚ ΠΎΠΏΡ€Π΅Π΄Π΅Π»Π΅Π½Π½Ρ‹Π΅ энСргии ΠΈ Π΄Π»ΠΈΠ½Ρ‹ Π²ΠΎΠ»Π½, ΠΈ ΠΎΠ½ΠΈ Π½Π΅ двиТутся ΠΏΠΎ Π·Π°ΠΌΠΊΠ½ΡƒΡ‚ΠΎΠΌΡƒ ΠΊΠΎΠ½Ρ‚ΡƒΡ€Ρƒ, ΠΊΠ°ΠΊ ΠΌΠ°Π³Π½ΠΈΡ‚Π½ΠΎΠ΅ ΠΏΠΎΠ»Π΅. ΠΊ. Β«ΠœΠ°Π³Π½ΠΈΡ‚Π½Ρ‹ΠΉΒ» Ρ„ΠΎΡ‚ΠΎΠ½ каТСтся Π΄Ρ€ΡƒΠ³ΠΈΠΌ, Ρ‡Π΅ΠΌ Β«ΠΎΠ±Ρ‹Ρ‡Π½Ρ‹ΠΉΒ» Ρ„ΠΎΡ‚ΠΎΠ½, ΠΈ я Ρ…ΠΎΡ‚Π΅Π» Π±Ρ‹ ΠΏΠΎΠ½ΡΡ‚ΡŒ, ΠΏΠΎΡ‡Π΅ΠΌΡƒ, Ссли это Π²ΠΎΠ·ΠΌΠΎΠΆΠ½ΠΎ Π±Π΅Π· понимания ΠΌΠ°Ρ‚Π΅ΠΌΠ°Ρ‚ΠΈΠΊΠΈ. Бпасибо!


— ДТСймс (30 Π»Π΅Ρ‚)

A:

Π― Π½Π΅ смогу это ΠΎΠ±ΡŠΡΡΠ½ΠΈΡ‚ΡŒ, Π½ΠΎ ΠΌΠΎΠ³Ρƒ ΠΏΡ€ΠΎΡΡΠ½ΠΈΡ‚ΡŒ Π½Π΅ΠΊΠΎΡ‚ΠΎΡ€Ρ‹Π΅ ΠΌΠΎΠΌΠ΅Π½Ρ‚Ρ‹.

Π€ΠΎΡ‚ΠΎΠ½Ρ‹, вносящиС Π²ΠΊΠ»Π°Π΄ Π² ΠΌΠ°Π³Π½ΠΈΡ‚Π½Ρ‹Π΅ поля , Π½ΠΈΡ‡Π΅ΠΌ Π½Π΅ ΠΎΡ‚Π»ΠΈΡ‡Π°ΡŽΡ‚ΡΡ ΠΎΡ‚ Ρ„ΠΎΡ‚ΠΎΠ½ΠΎΠ², вносящих Π²ΠΊΠ»Π°Π΄ Π² элСктричСскиС поля. ΠšΠΎΠ½ΠΊΡ€Π΅Ρ‚Π½Ρ‹ΠΉ ΠΎΠ±Ρ€Π°Π·Π΅Ρ† Ρ‚ΠΎΠ³ΠΎ, ΠΊΠ°ΠΊΠΈΠ΅ Ρ„Π°Π·Ρ‹ ΠΏΡ€ΠΈΡΡƒΡ‚ΡΡ‚Π²ΡƒΡŽΡ‚ для Ρ€Π°Π·Π½ΠΎΠ³ΠΎ количСства Ρ„ΠΎΡ‚ΠΎΠ½ΠΎΠ², опрСдСляСт, ΠΊΠ°ΠΊΠΈΠ΅ классичСскиС поля ΠΏΡ€ΠΈΡΡƒΡ‚ΡΡ‚Π²ΡƒΡŽΡ‚.

Π§Ρ‚ΠΎ касаСтся ΠΎΡ‚Π½ΠΎΡˆΠ΅Π½ΠΈΡ количСства Ρ„ΠΎΡ‚ΠΎΠ½ΠΎΠ² ΠΊ классичСским полям, я ΠΌΠΎΠ³Ρƒ ΠΏΡ€Π΅Π΄Π»ΠΎΠΆΠΈΡ‚ΡŒ аналогию, ΠΊΠΎΡ‚ΠΎΡ€ΡƒΡŽ Π²Ρ‹ ΠΌΠΎΠ³Π»ΠΈ Π±Ρ‹ ΠΈΠ·ΡƒΡ‡ΠΈΡ‚ΡŒ, ΠΈ ΠΊΠΎΡ‚ΠΎΡ€ΡƒΡŽ Π±Ρ‹Π»ΠΎ Π±Ρ‹ Π»Π΅Π³Ρ‡Π΅ ΠΈΠ·ΠΎΠ±Ρ€Π°Π·ΠΈΡ‚ΡŒ. ΠŸΠΎΡΠΌΠΎΡ‚Ρ€ΠΈΡ‚Π΅ Π½Π° Π²ΠΎΠ»Π½ΠΎΠ²Ρ‹Π΅ Ρ„ΡƒΠ½ΠΊΡ†ΠΈΠΈ, ΠΊΠΎΡ‚ΠΎΡ€Ρ‹Π΅ ΠΏΡ€Π΅Π΄ΡΡ‚Π°Π²Π»ΡΡŽΡ‚ состояния простого гармоничСского осциллятора (масса Π½Π° ΠΏΡ€ΡƒΠΆΠΈΠ½Π΅). (https://en.wikipedia.org/wiki/Quantum_harmonic_oscillator) Π’Π΅, Ρƒ ΠΊΠΎΠ³ΠΎ опрСдСлСнная энСргия, ΠΏΠΎΠ΄ΠΎΠ±Π½Ρ‹ состояниям Ρ„ΠΎΡ‚ΠΎΠ½ΠΎΠ² с ΠΎΠΏΡ€Π΅Π΄Π΅Π»Π΅Π½Π½Ρ‹ΠΌΠΈ числами ΠΈ, ΡΠ»Π΅Π΄ΠΎΠ²Π°Ρ‚Π΅Π»ΡŒΠ½ΠΎ, ΠΎΠΏΡ€Π΅Π΄Π΅Π»Π΅Π½Π½Ρ‹ΠΌΠΈ энСргиями. ΠžΠ΄Π½ΠΎΡΠ½Π΅Ρ€Π³Π΅Ρ‚ΠΈΡ‡Π΅ΡΠΊΠΈΠ΅ состояния SHO всСгда Ρ€Π°Π²Π½ΠΎΠΌΠ΅Ρ€Π½ΠΎ распрСдСлСны Π²ΠΎΠΊΡ€ΡƒΠ³ срСднСй Ρ‚ΠΎΡ‡ΠΊΠΈ Π±Π΅Π· срСднСго смСщСния ΠΈ срСднСй скорости. По Ρ‚ΠΎΡ‡Π½ΠΎΠΉ Π°Π½Π°Π»ΠΎΠ³ΠΈΠΈ, Ρ„ΠΎΡ‚ΠΎΠ½Π½Ρ‹Π΅ состояния с ΠΎΠΏΡ€Π΅Π΄Π΅Π»Π΅Π½Π½Ρ‹ΠΌ числом Π½Π΅ ΠΈΠΌΠ΅ΡŽΡ‚ срСднСго ΠΌΠ°Π³Π½ΠΈΡ‚Π½ΠΎΠ³ΠΎ поля ΠΈΠ»ΠΈ элСктричСского поля. Π§Ρ‚ΠΎΠ±Ρ‹ Π·Π°ΡΡ‚Π°Π²ΠΈΡ‚ΡŒ SHO Π²Ρ€Π°Ρ‰Π°Ρ‚ΡŒΡΡ классичСски, Π²Π°ΠΌ Π½ΡƒΠΆΠ½ΠΎ ΡΠΎΠ·Π΄Π°Ρ‚ΡŒ состояния, Π² ΠΊΠΎΡ‚ΠΎΡ€Ρ‹Ρ… интСрфСрСнция ΠΌΠ΅ΠΆΠ΄Ρƒ частями с Ρ€Π°Π·Π½Ρ‹ΠΌΠΈ энСргиями Π²Ρ‹Π·Ρ‹Π²Π°Π΅Ρ‚ ΠΎΡ‚ΠΌΠ΅Π½Ρƒ Π² ΠΎΠ΄Π½ΠΈΡ… частях ΠΈ усилСниС Π² Π΄Ρ€ΡƒΠ³ΠΈΡ…. По ΠΌΠ΅Ρ€Π΅ Ρ‚ΠΎΠ³ΠΎ, ΠΊΠ°ΠΊ Π²ΠΎΠ»Π½Π° измСняСтся Π²ΠΎ Π²Ρ€Π΅ΠΌΠ΅Π½ΠΈ, полоТСния этих Ρ‚ΠΎΡ‡Π΅ΠΊ с Π½ΠΈΠ·ΠΊΠΎΠΉ ΠΈ высокой ΠΏΠ»ΠΎΡ‚Π½ΠΎΡΡ‚ΡŒΡŽ ΠΊΠΎΠ»Π΅Π±Π»ΡŽΡ‚ΡΡ Π²Π·Π°Π΄ ΠΈ Π²ΠΏΠ΅Ρ€Π΅Π΄, Ρ‡Ρ‚ΠΎ ΠΎΠ·Π½Π°Ρ‡Π°Π΅Ρ‚ ΠΈΠ·ΠΌΠ΅Π½Π΅Π½ΠΈΠ΅ полоТСния ΠΈ скорости. Аналогом для Ρ„ΠΎΡ‚ΠΎΠ½ΠΎΠ² являСтся ΠΈΠ·ΠΌΠ΅Π½Π΅Π½ΠΈΠ΅ ΠΏΠΎΠ»Π΅ΠΉ.

Майк Π’.

(ΠΎΠΏΡƒΠ±Π»ΠΈΠΊΠΎΠ²Π°Π½ΠΎ 16.01.2017)

Π”ΠΎΠΏΠΎΠ»Π½Π΅Π½ΠΈΠ΅ β„–6: ΠΈΠ½Π³Ρ€Π΅Π΄ΠΈΠ΅Π½Ρ‚Ρ‹ ΠΌΠ°Π³Π½Π΅Ρ‚ΠΈΠ·ΠΌΠ°?

Q:

Майк Π’. сказал: «Π‘ Π΄Ρ€ΡƒΠ³ΠΎΠΉ стороны, ΠΌΠΎΠΆΠ΅Ρ‚ Π±Ρ‹Ρ‚ΡŒ, Π²Ρ‹ Π΄ΡƒΠΌΠ°Π΅Ρ‚Π΅, Ρ‡Ρ‚ΠΎ ΠΊΡ€ΠΎΠΌΠ΅ Ρ„ΠΎΡ‚ΠΎΠ½ΠΎΠ² Π΅ΡΡ‚ΡŒ ΠΊΠ°ΠΊΠΎΠΉ-Ρ‚ΠΎ Π΄Ρ€ΡƒΠ³ΠΎΠΉ ΠΈΠ½Π³Ρ€Π΅Π΄ΠΈΠ΅Π½Ρ‚. НСт, ΠΌΠ°Π»ΡŒΡ‡ΠΈΠΊ! Π­Ρ‚ΠΎ смСлоС заявлСниС. ΠšΠΎΠ½Π΅Ρ‡Π½ΠΎ, Π²Ρ‹ ΠΈΠΌΠ΅Π΅Ρ‚Π΅ Π² Π²ΠΈΠ΄Ρƒ Β«ΠΌΡ‹ Π½Π΅ Π·Π½Π°Π΅ΠΌ Π½ΠΈ ΠΎ ΠΊΠ°ΠΊΠΎΠΌ Π΄Ρ€ΡƒΠ³ΠΎΠΌ ΠΈΠ½Π³Ρ€Π΅Π΄ΠΈΠ΅Π½Ρ‚Π΅Β» β€” ΠΊΠ°ΠΊ ΠΌΡ‹ Π½ΠΈΡ‡Π΅Π³ΠΎ Π½Π΅ Π·Π½Π°Π΅ΠΌ ΠΎ Ρ‚Π΅ΠΌΠ½ΠΎΠΉ энСргии ΠΈ ΠΎΡ‡Π΅Π½ΡŒ ΠΌΠ°Π»ΠΎ ΠΎ Ρ‚Π΅ΠΌΠ½ΠΎΠΉ ΠΌΠ°Ρ‚Π΅Ρ€ΠΈΠΈ. Π˜Π½Ρ‚Π΅Ρ€Π΅ΡΠ½ΠΎ, Ρ‡Ρ‚ΠΎ Π±Ρ‹ ΠΎΠ½ сказал Π΄ΠΎ постулирования Ρ„ΠΎΡ‚ΠΎΠ½ΠΎΠ². ΠŸΡ€ΠΎΡ‡ΠΈΡ‚Π°Π² всю Π΄ΠΈΡΠΊΡƒΡΡΠΈΡŽ, ΠΌΠ½Π΅ каТСтся, Ρ‡Ρ‚ΠΎ СдинствСнный Π²Π΅Ρ€Π½Ρ‹ΠΉ ΠΎΡ‚Π²Π΅Ρ‚: Β«ΠœΡ‹ Π½Π΅ Π·Π½Π°Π΅ΠΌΒ». Бколько Π°Ρ‚ΠΎΠΌΠΎΠ²/см Π² мСТгалактичСском пространствС? НС Ρ‚Π°ΠΊ ΡƒΠΆ ΠΈ ΠΌΠ½ΠΎΠ³ΠΎ, Π³Π°Ρ€Π°Π½Ρ‚ΠΈΡ€ΡƒΡŽ, поэтому ΠΌΠΎΠΆΠ½ΠΎ Π±Ρ‹Π»ΠΎ Π±Ρ‹ ΠΎΠΆΠΈΠ΄Π°Ρ‚ΡŒ, Ρ‡Ρ‚ΠΎ Π±ΡƒΠ΄Π΅Ρ‚ мСньшС этих ΠΆΠΈΠ·Π½Π΅Π½Π½ΠΎ Π²Π°ΠΆΠ½Ρ‹Ρ… Ρ„ΠΎΡ‚ΠΎΠ½ΠΎΠ², входящих ΠΈ выходящих ΠΈΠ· элСктронов Π² срСдС, Π³Π΄Π΅ мСньшС элСктронов, Π² ΠΊΠΎΡ‚ΠΎΡ€Ρ‹Π΅ ΠΌΠΎΠΆΠ½ΠΎ Π²Ρ…ΠΎΠ΄ΠΈΡ‚ΡŒ ΠΈ Π²Ρ‹Ρ…ΠΎΠ΄ΠΈΡ‚ΡŒ, β€” Π½ΠΎ ΠΊΠ°ΠΊΠΎΠ²ΠΎ Π±ΡƒΠ΄Π΅Ρ‚ притяТСниС ΠΌΠ΅ΠΆΠ΄Ρƒ двумя ΠΌΠ°Π³Π½ΠΈΡ‚Ρ‹ Π² этой срСдС? Π­Ρ‚ΠΎ Π±Ρ‹Π»ΠΎ Π±Ρ‹ Ρ‚ΠΎ ΠΆΠ΅ самоС, Π½Π΅ Ρ‚Π°ΠΊ Π»ΠΈ? ΠšΠΎΠ½Π΅Ρ‡Π½ΠΎ, ΡƒΠ΄ΠΈΠ²ΠΈΡ‚Π΅Π»ΡŒΠ½ΠΎ, Ρ‡Ρ‚ΠΎ Ссли Π΄Π²Π° ΡΠΈΠ»ΡŒΠ½Ρ‹Ρ… ΠΌΠ°Π³Π½ΠΈΡ‚Π° ΠΏΠΎΠΌΠ΅ΡΡ‚ΠΈΡ‚ΡŒ Π² ΡΠ°ΠΌΡƒΡŽ Ρ€Π°Π·Ρ€Π΅ΠΆΠ΅Π½Π½ΡƒΡŽ ΠΎΠ±Π»Π°ΡΡ‚ΡŒ пространства, ΠΎΠ½ΠΈ всС Ρ€Π°Π²Π½ΠΎ Π±ΡƒΠ΄ΡƒΡ‚ ΠΎΠΊΠ°Π·Ρ‹Π²Π°Ρ‚ΡŒ ΠΌΠΎΡ‰Π½ΠΎΠ΅ воздСйствиС Π΄Ρ€ΡƒΠ³ Π½Π° Π΄Ρ€ΡƒΠ³Π° (ΠΏΡ€ΠΈΡ‚ΡΠ³ΠΈΠ²Π°ΡŽΡ‰Π΅Π΅ ΠΈΠ»ΠΈ ΠΎΡ‚Ρ‚Π°Π»ΠΊΠΈΠ²Π°ΡŽΡ‰Π΅Π΅), Π΄Π°ΠΆΠ΅ Ссли ΠΎΠ½ΠΈ ΠΌΠΎΠ³ΡƒΡ‚ Π±Ρ‹Ρ‚ΡŒ Π½Π΅ΠΏΠΎΠ΄Π²ΠΈΠΆΠ½Ρ‹ ΠΎΡ‚Π½ΠΎΡΠΈΡ‚Π΅Π»ΡŒΠ½ΠΎ Π΄Ρ€ΡƒΠ³ Π΄Ρ€ΡƒΠ³Π°. ΠœΠ΅ΠΆΠ΄Ρƒ Π½ΠΈΠΌΠΈ явно Ρ‡Ρ‚ΠΎ-Ρ‚ΠΎ Π΅ΡΡ‚ΡŒ. ΠšΡ‚ΠΎ-Ρ‚ΠΎ Π²Ρ‹ΡˆΠ΅ сказал, Ρ‡Ρ‚ΠΎ ΠΌΠ°Π³Π½ΠΈΡ‚Π½ΠΎΠ΅ ΠΏΠΎΠ»Π΅ Π½Π΅ ΠΌΠΎΠΆΠ΅Ρ‚ ΡΡƒΡ‰Π΅ΡΡ‚Π²ΠΎΠ²Π°Ρ‚ΡŒ само ΠΏΠΎ сСбС β€” это ΡΠΎΡΡ‚Π°Π²Π»ΡΡŽΡ‰Π°Ρ элСктромагнитного поля, ΠΈ мСня это интСрСсуСт, ΠΏΠΎΡ‚ΠΎΠΌΡƒ Ρ‡Ρ‚ΠΎ я ΡΡ‡ΠΈΡ‚Π°ΡŽ, Ρ‡Ρ‚ΠΎ это ΠΏΠΎΠ»Π΅ Π΄ΠΎΠ»ΠΆΠ½ΠΎ ΠΏΡ€ΠΈΡΡƒΡ‚ΡΡ‚Π²ΠΎΠ²Π°Ρ‚ΡŒ, ΠΊΠΎΠ³Π΄Π° ΠΌΠ°Π³Π½ΠΈΡ‚Ρ‹ Π½Π΅Ρ‚. Π― Π΄ΡƒΠΌΠ°ΡŽ, Π²ΠΏΠΎΠ»Π½Π΅ вСроятно, Ρ‡Ρ‚ΠΎ Ρ‚ΠΎ, Ρ‡Ρ‚ΠΎ составляСт это ΠΏΠΎΠ»Π΅, Ρ‚Π°ΠΊΠΆΠ΅ составляСт Ρ‚ΠΎ, Π² Ρ‡Π΅ΠΌ свСт создаСт Π²ΠΎΠ»Π½Ρƒ, Π° свСт, ΠΊΠ°ΠΊ Π½Π°ΠΌ говорят, являСтся элСктромагнитной Π²ΠΎΠ»Π½ΠΎΠΉ, Ρ‚Π°ΠΊΠΎΠΉ ΠΆΠ΅, ΠΊΠ°ΠΊ высокочастотная Ρ€Π°Π΄ΠΈΠΎΠ²ΠΎΠ»Π½Π°. МнС (ΠΏΠΎΠ»Π½ΠΎΠΌΡƒ Π΄ΠΈΠ»Π΅Ρ‚Π°Π½Ρ‚Ρƒ) каТСтся бСссмыслСнным ΡƒΡ‚Π²Π΅Ρ€ΠΆΠ΄Π΅Π½ΠΈΠ΅, Ρ‡Ρ‚ΠΎ свСт β€” это Π²ΠΎΠ»Π½Π°, проходящая сквозь Π½ΠΈΡ‡Ρ‚ΠΎ. Π”Ρ€ΡƒΠ³ΠΈΠ΅ Π²ΠΎΠ»Π½Ρ‹ β€” это всС Π²ΠΎΠ»Π½Ρ‹ Π² Ρ‡Π΅ΠΌ-Ρ‚ΠΎ β€” Π² Π²ΠΎΠ·Π΄ΡƒΡ…Π΅, Π² Π²ΠΎΠ΄Π΅, Π² Π²Π΅Ρ€Π΅Π²ΠΊΠ΅ ΠΈ Ρ‚. Π΄. Π‘Π²Π΅Ρ‚ распространяСтся Π² пространствС с ΠΎΡ‡Π΅Π½ΡŒ Ρ‚ΠΎΡ‡Π½ΠΎ ΠΎΠΏΡ€Π΅Π΄Π΅Π»Π΅Π½Π½ΠΎΠΉ ΡΠΊΠΎΡ€ΠΎΡΡ‚ΡŒΡŽ. НСсомнСнно, Ссли Π±Ρ‹ структура пространства Π±Ρ‹Π»Π° Π½Π΅ΠΌΠ½ΠΎΠ³ΠΎ Π΄Ρ€ΡƒΠ³ΠΎΠΉ, свСт двигался Π±Ρ‹ Π½Π΅ΠΌΠ½ΠΎΠ³ΠΎ ΠΌΠ΅Π΄Π»Π΅Π½Π½Π΅Π΅ ΠΈΠ»ΠΈ Π½Π΅ΠΌΠ½ΠΎΠ³ΠΎ быстрСС, Ρ‚Π°ΠΊ Ρ‡Ρ‚ΠΎ это ΠΏΠΎΠ»Π΅ Π΄ΠΎΠ»ΠΆΠ½ΠΎ Π±Ρ‹Ρ‚ΡŒ ΡƒΠ΄ΠΈΠ²ΠΈΡ‚Π΅Π»ΡŒΠ½ΠΎ постоянным Π²ΠΎ всСм пространствС ΠΈ Π½Π΅ Π·Π°Π²ΠΈΡΠ΅Ρ‚ΡŒ ΠΎΡ‚ плотности Π°Ρ‚ΠΎΠΌΠΎΠ². Π­Ρ‚ΠΎ, бСзусловно, Π΄ΠΎΠ»ΠΆΠ½Π° Π±Ρ‹Ρ‚ΡŒ ΠΎΡ‡Π΅Π½ΡŒ эффСктивная транспортная срСда, ΠΏΠΎΡ‚ΠΎΠΌΡƒ Ρ‡Ρ‚ΠΎ свСт ΠΌΠΎΠΆΠ΅Ρ‚ ΠΏΡƒΡ‚Π΅ΡˆΠ΅ΡΡ‚Π²ΠΎΠ²Π°Ρ‚ΡŒ ΠΌΠΈΠ»Π»ΠΈΠ°Ρ€Π΄Ρ‹ Π»Π΅Ρ‚ ΠΈ всС ΠΆΠ΅ Π΄ΠΎΡΡ‚ΠΈΠ³Π°Ρ‚ΡŒ нас, Π½Π΅ Ρ€Π°ΡΡΠ΅ΠΈΠ²Π°ΡΡΡŒ большС, Ρ‡Π΅ΠΌ Π½Π° ΠΊΠ²Π°Π΄Ρ€Π°Ρ‚ расстояния. Π’Π°ΠΊ Ρ‡Π΅Ρ€Π΅Π· Ρ‡Ρ‚ΠΎ ΠΎΠ½ ΠΏΡƒΡ‚Π΅ΡˆΠ΅ΡΡ‚Π²ΡƒΠ΅Ρ‚? Π•ΡΡ‚ΡŒ Π»ΠΈ какая-Π½ΠΈΠ±ΡƒΠ΄ΡŒ подсказка Π² Ρ‚ΠΎΠΌ, Ρ‡Ρ‚ΠΎ элСктричСскиС ΠΈ ΠΌΠ°Π³Π½ΠΈΡ‚Π½Ρ‹Π΅ ΠΊΠΎΠΌΠΏΠΎΠ½Π΅Π½Ρ‚Ρ‹ Ρ€Π°Π΄ΠΈΠΎΠΏΠ΅Ρ€Π΅Π΄Π°Ρ‡ΠΈ находятся ΠΏΠΎΠ΄ прямым ΡƒΠ³Π»ΠΎΠΌ? (Π― имСю Π² Π²ΠΈΠ΄Ρƒ ΡƒΠ³ΠΎΠ» поляризации). Π― Π²ΠΈΠ΄Π΅Π» объяснСния Ρ‚ΠΎΠ³ΠΎ, ΠΊΠ°ΠΊ свСт ΠΏΡ€ΠΎΡ…ΠΎΠ΄ΠΈΡ‚ Ρ‡Π΅Ρ€Π΅Π· стСкло ΠΈΠ»ΠΈ Π²ΠΎΠ΄Ρƒ (Π³Π΄Π΅ Π΅Π³ΠΎ ΡΠΊΠΎΡ€ΠΎΡΡ‚ΡŒ отличаСтся ΠΎΡ‚ скорости Π² космосС), Π½ΠΎ это зависит ΠΎΡ‚ Ρ‚ΠΎΠ³ΠΎ, Ρ‡Ρ‚ΠΎ Π°Ρ‚ΠΎΠΌΡ‹ Π²ΠΎΠ·Π±ΡƒΠΆΠ΄Π°ΡŽΡ‚ΡΡ свСтом ΠΈ ΠΈΡΠΏΡƒΡΠΊΠ°ΡŽΡ‚ Π² ΠΎΡ‚Π²Π΅Ρ‚ ΠΏΡ€ΠΎΡ‚ΠΎΠ½ β€” ΡƒΠ΄ΠΈΠ²ΠΈΡ‚Π΅Π»ΡŒΠ½ΠΎ, всСгда Π² ΠΎΠ΄Π½ΠΎΠΌ ΠΈ Ρ‚ΠΎΠΌ ΠΆΠ΅ Π½Π°ΠΏΡ€Π°Π²Π»Π΅Π½ΠΈΠΈ. Π­Ρ‚ΠΎ объяснСниС ΠΎΠ±ΡŠΡΡΠ½ΡΠ΅Ρ‚ Π΄ΠΈΡ„Ρ€Π°ΠΊΡ†ΠΈΡŽ, ΠΎΠ΄Π½Π°ΠΊΠΎ Ρ‚ΠΎΡ‚ ΠΆΠ΅ ΠΌΠ΅Ρ…Π°Π½ΠΈΠ·ΠΌ нСльзя ΠΏΡ€ΠΈΠΌΠ΅Π½ΠΈΡ‚ΡŒ ΠΊ пустому пространству. Π•ΡΡ‚ΡŒ Π»ΠΈ ΠΊΠ°ΠΊΠΎΠΉ-Π»ΠΈΠ±ΠΎ ΠΎΡ‚Π²Π΅Ρ‚, Π·Π° ΠΊΠΎΡ‚ΠΎΡ€Ρ‹ΠΉ ΠΌΠΎΠΆΠ΅Ρ‚ ΡƒΡ…Π²Π°Ρ‚ΠΈΡ‚ΡŒΡΡ нСспСциалист, ΠΈΠ»ΠΈ это ΠΎΠ΄Π½Π° ΠΈΠ· Ρ‚Π΅Ρ… Π²Π΅Ρ‰Π΅ΠΉ, ΠΊΠ°ΠΊ sqrt(-1), ΠΊΠΎΡ‚ΠΎΡ€Ρ‹Π΅ ΠΌΠΎΠ³ΡƒΡ‚ Π±Ρ‹Ρ‚ΡŒ Ρ‚ΠΎΠ»ΡŒΠΊΠΎ ΠΏΠΎΠ½ΠΈΠΌΠ°Π΅Ρ‚Π΅ Π² области ΠΌΠ°Ρ‚Π΅ΠΌΠ°Ρ‚ΠΈΠΊΠΈ? Π”Π°ΠΆΠ΅ Ρ‚Π°ΠΌ Π΅ΡΡ‚ΡŒ Ρ‚Π²Π΅Ρ€Π΄ΠΎΠ΅ ΠΏΠΎΠ½ΠΈΠΌΠ°Π½ΠΈΠ΅ Ρ‚ΠΎΠ³ΠΎ, Ρ‡Ρ‚ΠΎ это Ρ‚Π°ΠΊΠΎΠ΅, Π° Π½Π΅ ΠΊΠ°ΠΊΠΎΠ²Ρ‹ Π΅Π³ΠΎ свойства?
— Майк Коллинз (71 Π³ΠΎΠ΄)
Π“ΡƒΠΈΠ½Π΅Ρ‚, Уэльс

A:

Π’Ρ‹ ΠΏΡ€Π°Π²Ρ‹, Ρ‡Ρ‚ΠΎ Π² Ρ†Π΅Π»ΠΎΠΌ ΠΌΡ‹ Π½Π΅ Π·Π½Π°Π΅ΠΌ всСх ΠΈΠ½Π³Ρ€Π΅Π΄ΠΈΠ΅Π½Ρ‚ΠΎΠ² ΠΌΠΈΡ€Π°. ΠœΡ‹, вСроятно, Π΄Π°ΠΆΠ΅ Π½Π΅ Π·Π½Π°Π΅ΠΌ основной Ρ„ΠΎΡ€ΠΌΡ‹ Ρ‚Π΅ΠΎΡ€ΠΈΠΈ, Ρ‚ΠΎΠ³ΠΎ, ΠΊΠ°ΠΊ пространство-врСмя Π²ΠΎΠ·Π½ΠΈΠΊΠ°Π΅Ρ‚ ΠΈΠ· Π½Π΅ΠΊΠΎΡ‚ΠΎΡ€Ρ‹Ρ… Π±ΠΎΠ»Π΅Π΅ Π³Π»ΡƒΠ±ΠΎΠΊΠΈΡ… Ρ„ΠΎΡ€ΠΌ ΠΈ Ρ‚. Π΄. Π’Π΅ΠΌ Π½Π΅ ΠΌΠ΅Π½Π΅Π΅, Π² контСкстС Ρ‚ΠΎΠ³ΠΎ, Ρ‡Ρ‚ΠΎ ΠΌΡ‹ Π·Π½Π°Π΅ΠΌ, Π² ΠΌΠ°Π³Π½Π΅Ρ‚ΠΈΠ·ΠΌΠ΅ Π½Π΅Ρ‚ особой Ρ‚Π°ΠΉΠ½Ρ‹. На самом Π΄Π΅Π»Π΅ ΠΌΠ°Π³Π½Π΅Ρ‚ΠΈΠ·ΠΌ являСтся Ρ‡Π°ΡΡ‚ΡŒΡŽ элСктрослабой Ρ‚Π΅ΠΎΡ€ΠΈΠΈ, которая являСтся самой извСстной ΠΈΠ· ΠΈΠΌΠ΅ΡŽΡ‰ΠΈΡ…ΡΡ Ρƒ нас Ρ‚Π΅ΠΎΡ€ΠΈΠΉ. Он прСдсказываСт, Π½Π°ΠΏΡ€ΠΈΠΌΠ΅Ρ€, ΠΌΠ°Π³Π½Π΅Ρ‚ΠΈΠ·ΠΌ элСктрона с Ρ‚ΠΎΡ‡Π½ΠΎΡΡ‚ΡŒΡŽ Π±ΠΎΠ»Π΅Π΅ ΠΎΠ΄Π½ΠΎΠΉ стомиллиардной.

Π’ΠΎΠ»Π½Ρ‹, ΠΊΠΎΡ‚ΠΎΡ€Ρ‹Π΅ Π²Ρ‹ упомянули, ΠΌΠΎΠΆΠ½ΠΎ ΠΎΠΏΠΈΡΠ°Ρ‚ΡŒ ΠΊΠ°ΠΊ Ρ‚ΠΈΠΏ повСдСния Π½ΠΈΠΆΠ΅Π»Π΅ΠΆΠ°Ρ‰ΠΈΡ… срСд β€” Π²ΠΎΠ΄Ρ‹, Π²ΠΎΠ·Π΄ΡƒΡ…Π° ΠΈ Ρ‚. Π΄. Однако Π½Π° Π±ΠΎΠ»Π΅Π΅ Π³Π»ΡƒΠ±ΠΎΠΊΠΎΠΌ ΡƒΡ€ΠΎΠ²Π½Π΅ ΠΈΠ½Π³Ρ€Π΅Π΄ΠΈΠ΅Π½Ρ‚Ρ‹ ВсСлСнной (Ρ„ΠΎΡ‚ΠΎΠ½Ρ‹, ΠΊΠ²Π°Ρ€ΠΊΠΈ, Π½Π΅ΠΉΡ‚Ρ€ΠΈΠ½ΠΎ, Π³Π»ΡŽΠΎΠ½Ρ‹…) Π² настоящСС врСмя ΠΌΠΎΠ³ΡƒΡ‚ Π±Ρ‹Ρ‚ΡŒ описаны Ρ‚ΠΎΠ»ΡŒΠΊΠΎ ΠΊΠ°ΠΊ чистыС ΠΊΠ²Π°Π½Ρ‚ΠΎΠ²Ρ‹Π΅ Π²ΠΎΠ»Π½Ρ‹ сами ΠΏΠΎ сСбС. Π€ΠΎΡ‚ΠΎΠ½Ρ‹ Ρ‚Π°ΠΊΠΈΠ΅ ΠΆΠ΅ основныС, ΠΊΠ°ΠΊ ΠΈ Π»ΡŽΠ±Ρ‹Π΅ Π΄Ρ€ΡƒΠ³ΠΈΠ΅ ΠΈΠ½Π³Ρ€Π΅Π΄ΠΈΠ΅Π½Ρ‚Ρ‹, ΠΊΠΎΡ‚ΠΎΡ€Ρ‹Π΅ Ρƒ нас Π΅ΡΡ‚ΡŒ.

Π’ΠΎΠ·ΠΌΠΎΠΆΠ½ΠΎ, ΠΊΠΎΠ³Π΄Π°-Π½ΠΈΠ±ΡƒΠ΄ΡŒ Π±ΡƒΠ΄ΡƒΡ‚ Π½Π°ΠΉΠ΄Π΅Π½Ρ‹ ΠΊΠ°ΠΊΠΈΠ΅-Ρ‚ΠΎ Π±ΠΎΠ»Π΅Π΅ Π³Π»ΡƒΠ±ΠΎΠΊΠΈΠ΅ ΠΈΠ½Π³Ρ€Π΅Π΄ΠΈΠ΅Π½Ρ‚Ρ‹, ΠΈ всС наши Π½Ρ‹Π½Π΅ΡˆΠ½ΠΈΠ΅ Ρ„ΡƒΠ½Π΄Π°ΠΌΠ΅Π½Ρ‚Π°Π»ΡŒΠ½Ρ‹Π΅ поля частиц Π±ΡƒΠ΄ΡƒΡ‚ Ρ€Π°ΡΡΠΌΠ°Ρ‚Ρ€ΠΈΠ²Π°Ρ‚ΡŒΡΡ ΠΊΠ°ΠΊ Π²ΠΎΠ·Π½ΠΈΠΊΠ°ΡŽΡ‰ΠΈΠ΅ ΠΈΠ· повСдСния этой Π±ΠΎΠ»Π΅Π΅ Π³Π»ΡƒΠ±ΠΎΠΊΠΎΠΉ Ρ‚Π΅ΠΎΡ€ΠΈΠΈ. НС окаТСтся Π»ΠΈ Ρ‚ΠΎΠ³Π΄Π°, Ρ‡Ρ‚ΠΎ Π±ΠΎΠ»Π΅Π΅ глубокая тСория Π²ΠΎΠ·Π½ΠΈΠΊΠ½Π΅Ρ‚ ΠΈΠ· Π΅Ρ‰Π΅ Π±ΠΎΠ»Π΅Π΅ Π³Π»ΡƒΠ±ΠΎΠΊΠΎΠΉ? Π‘ΡƒΠ΄Π΅Ρ‚ Π»ΠΈ этот ΠΏΠ°Ρ‚Ρ‚Π΅Ρ€Π½ ΠΏΡ€ΠΎΠ΄ΠΎΠ»ΠΆΠ°Ρ‚ΡŒΡΡ Π²Π΅Ρ‡Π½ΠΎ ΠΈΠ»ΠΈ достигнСт самого Π³Π»ΡƒΠ±ΠΎΠΊΠΎΠ³ΠΎ уровня? ΠœΡ‹ Π½Π΅ Π·Π½Π°Π΅ΠΌ. Пока Π΅ΡΡ‚ΡŒ ΠΎΠ±ΠΎΡ€Π²Π°Π½Π½Ρ‹Π΅ ΠΊΠΎΠ½Ρ†Ρ‹ (нСсоотвСтствиС ΠΌΠ΅ΠΆΠ΄Ρƒ ΠΎΠ±Ρ‰Π΅ΠΉ Ρ‚Π΅ΠΎΡ€ΠΈΠ΅ΠΉ ΠΎΡ‚Π½ΠΎΡΠΈΡ‚Π΅Π»ΡŒΠ½ΠΎΡΡ‚ΠΈ ΠΈ ΠΊΠ²Π°Π½Ρ‚ΠΎΠ²ΠΎΠΉ Ρ‚Π΅ΠΎΡ€ΠΈΠ΅ΠΉ поля, Ρ‚Π΅ΠΌΠ½ΠΎΠΉ энСргиСй ΠΈ Ρ‚Π΅ΠΌΠ½ΠΎΠΉ ΠΌΠ°Ρ‚Π΅Ρ€ΠΈΠ΅ΠΉ…), ΠΌΡ‹ Π·Π½Π°Π΅ΠΌ, Ρ‡Ρ‚ΠΎ ΠΌΡ‹ Π½Π΅ Π² самом Π½ΠΈΠ·Ρƒ стСка. Если какая-Ρ‚ΠΎ Ρ‚ΠΎΡ‡ΠΊΠ° Π±ΡƒΠ΄Π΅Ρ‚ достигнута Π±Π΅Π· основных ΠΎΠ±ΠΎΡ€Π²Π°Π½Π½Ρ‹Ρ… ΠΊΠΎΠ½Ρ†ΠΎΠ², Ρ‚ΠΎ, Π²ΠΎΠ·ΠΌΠΎΠΆΠ½ΠΎ, ΠΌΡ‹ окаТСмся Π½Π° самом Π³Π»ΡƒΠ±ΠΎΠΊΠΎΠΌ ΡƒΡ€ΠΎΠ²Π½Π΅.

Майк Π’.

(ΠΎΠΏΡƒΠ±Π»ΠΈΠΊΠΎΠ²Π°Π½ΠΎ 15.06.2017)

Π”ΠΎΠΏΠΎΠ»Π½Π΅Π½ΠΈΠ΅ β„– 7: Π½Π΅ΠΎΠ±Ρ…ΠΎΠ΄ΠΈΠΌΡ‹ Π»ΠΈ ΠΌΠ°Π³Π½ΠΈΡ‚Π½Ρ‹Π΅ поля?

Q:

Π― Ρ…ΠΎΡ‚Π΅Π» Π±Ρ‹ Π·Π°Π΄Π°Ρ‚ΡŒ, казалось Π±Ρ‹, Π³Π»ΡƒΠΏΡ‹ΠΉ вопрос. ΠšΠ°ΠΊΠΎΠ²Ρ‹ ΡΠΊΡΠΏΠ΅Ρ€ΠΈΠΌΠ΅Π½Ρ‚Π°Π»ΡŒΠ½Ρ‹Π΅ Π΄ΠΎΠΊΠ°Π·Π°Ρ‚Π΅Π»ΡŒΡΡ‚Π²Π° Ρ‚Ρ€Π°Π΄ΠΈΡ†ΠΈΠΎΠ½Π½ΠΎΠΉ ΠΈΠ΄Π΅ΠΈ ΠΌΠ°Π³Π½ΠΈΡ‚Π½Ρ‹Ρ… ΠΏΠΎΠ»Π΅ΠΉ? ΠœΡ‹ Π΄ΠΎΠ»ΠΆΠ½Ρ‹ ΠΏΠΎΠΌΠ½ΠΈΡ‚ΡŒ, Ρ‡Ρ‚ΠΎ идСя Π²Ρ€Π°Ρ‰Π°ΡŽΡ‰Π΅Π³ΠΎΡΡ пСрпСндикулярного ΠΏΠΎΡ‚ΠΎΠΊΠ° Π±Ρ‹Π»Π° основана Π½Π° ΠΈΠ³Π½ΠΎΡ€ΠΈΡ€ΠΎΠ²Π°Π½ΠΈΠΈ ΠΌΠ°Π³Π½Π΅Ρ‚ΠΈΠ·ΠΌΠ° β€” ΠΎΡ€Π±ΠΈΡ‚Π°Π»ΡŒΠ½Ρ‹Π΅ ΠΈ Π²Ρ€Π°Ρ‰Π°ΡŽΡ‰ΠΈΠ΅ΡΡ элСктроны Π±Ρ‹Π»ΠΈ нСизвСстны 150 Π»Π΅Ρ‚ Π½Π°Π·Π°Π΄. Π― Ρ‚Ρ‰Π΅Ρ‚Π½ΠΎ просил Π΄ΠΎΠΊΠ°Π·Π°Ρ‚Π΅Π»ΡŒΡΡ‚Π²Π° этого ΠΏΠΎΡ‚ΠΎΠΊΠ°. ΠšΠ°ΠΆΠ΅Ρ‚ΡΡ, это всСго лишь Π΄ΠΎΠ³Π°Π΄ΠΊΠ°, которая ΠΏΡ€Π΅Π²Ρ€Π°Ρ‚ΠΈΠ»Π°ΡΡŒ Π² Π²Π΅Ρ€Ρƒ. ΠŸΡ€Π΅Π΄ΠΏΠΎΠ»ΠΎΠΆΠΈΠΌ, Ρ‡Ρ‚ΠΎ ΠΌΠ°Π³Π½ΠΈΡ‚Ρ‹ Π±Ρ‹Π»ΠΈ нСизвСстны Π² Ρ‚ΠΎ врСмя. Π’ΠΎΠ³Π΄Π° экспСримСнты с элСктричСством ΠΏΡ€ΠΈΠ²Π΅Π»ΠΈ Π±Ρ‹ ΠΊ простому Π·Π°ΠΊΠΎΠ½Ρƒ: ΠΎΠ΄ΠΈΠ½Π°ΠΊΠΎΠ²Ρ‹Π΅ Ρ‚ΠΎΠΊΠΈ ΠΏΡ€ΠΈΡ‚ΡΠ³ΠΈΠ²Π°ΡŽΡ‚ΡΡ, Π° ΠΏΡ€ΠΎΡ‚ΠΈΠ²ΠΎΠΏΠΎΠ»ΠΎΠΆΠ½Ρ‹Π΅ Ρ‚ΠΎΠΊΠΈ ΠΎΡ‚Ρ‚Π°Π»ΠΊΠΈΠ²Π°ΡŽΡ‚ΡΡ. Π—Π°Ρ‚Π΅ΠΌ этот основной Π·Π°ΠΊΠΎΠ½ ΠΎΠ±ΡŠΡΡΠ½ΡΠ΅Ρ‚ ΠΌΠ°Π³Π½Π΅Ρ‚ΠΈΠ·ΠΌ, Π½Π°ΠΏΡ€ΠΈΠΌΠ΅Ρ€, Π²Ρ‹Ρ€Π°Π²Π½ΠΈΠ²Π°Π½ΠΈΠ΅ ΠΆΠ΅Π»Π΅Π·Π½Ρ‹Ρ… ΠΎΠΏΠΈΠ»ΠΎΠΊ Π²ΠΎΠΊΡ€ΡƒΠ³ ΠΌΠ°Π³Π½ΠΈΡ‚Π°. Π˜ΡΠΏΠΎΠ»ΡŒΠ·ΡƒΡ ΠΏΡ€ΠΈΠ½Ρ†ΠΈΠΏ Оккама, услоТнСниС Ρ†ΠΈΡ€ΠΊΡƒΠ»ΠΈΡ€ΡƒΡŽΡ‰Π΅Π³ΠΎ пСрпСндикулярного ΠΏΠΎΡ‚ΠΎΠΊΠ° каТСтся Π½Π΅ΠΎΠΏΡ€Π°Π²Π΄Π°Π½Π½Ρ‹ΠΌ. (Π’Π°ΠΊΠΈΠΌ ΠΎΠ±Ρ€Π°Π·ΠΎΠΌ, вмСсто Ρ‚ΠΎΠ³ΠΎ, Ρ‡Ρ‚ΠΎΠ±Ρ‹ ΠΈΡΠΏΠΎΠ»ΡŒΠ·ΠΎΠ²Π°Ρ‚ΡŒ Π·Π°ΠΊΠΎΠ½ Π‘ΠΈΠΎ ΠΈ Π‘Π°Π²Π°Ρ€Π° для прСдсказания плотности пСрпСндикулярного ΠΏΠΎΡ‚ΠΎΠΊΠ°, ΠΌΠΎΠΆΠ½ΠΎ Π²ΠΊΠ»ΡŽΡ‡ΠΈΡ‚ΡŒ скалярноС ΠΏΡ€ΠΎΠΈΠ·Π²Π΅Π΄Π΅Π½ΠΈΠ΅ Π²Π΅ΠΊΡ‚ΠΎΡ€ΠΎΠ² Ρ‚ΠΎΠΊΠ° для прСдсказания силы). Π‘Π»Π΅Π΄ΠΎΠ²Π°Ρ‚Π΅Π»ΡŒΠ½ΠΎ, ΠΌΠ°Π³Π½ΠΈΡ‚Π½Ρ‹Π΅ силы просто Π΄Π΅ΠΉΡΡ‚Π²ΡƒΡŽΡ‚ вдоль прямых Π»ΠΈΠ½ΠΈΠΉ ΠΌΠ΅ΠΆΠ΄Ρƒ двиТущимися зарядами. Π­Ρ‚ΠΎ Ρ‚ΠΎΡ‚ ΠΆΠ΅ простой ΠΏΡ€ΠΈΠ½Ρ†ΠΈΠΏ, ΠΊΠΎΡ‚ΠΎΡ€Ρ‹ΠΉ Ρ€Π°Π±ΠΎΡ‚Π°Π΅Ρ‚ для элСктростатичСских сил ΠΌΠ΅ΠΆΠ΄Ρƒ стационарными зарядами. Нам Π½Π΅ Π½ΡƒΠΆΠ½ΠΎ ΠΏΡ€Π΅Π΄ΠΏΠΎΠ»Π°Π³Π°Ρ‚ΡŒ, Ρ‡Ρ‚ΠΎ ВсСлСнная ΠΈΡΠΏΠΎΠ»ΡŒΠ·ΡƒΠ΅Ρ‚ Π΄Π²Π° ΡΠΎΠ²Π΅Ρ€ΡˆΠ΅Π½Π½ΠΎ Ρ€Π°Π·Π½Ρ‹Ρ… силовых ΠΌΠ΅Ρ…Π°Π½ΠΈΠ·ΠΌΠ°. Π”Π²ΠΈΠΆΠ΅Π½ΠΈΠ΅ просто измСняСт элСктростатичСскиС силы. ΠœΠ°Π³Π½ΠΈΡ‚Π½Ρ‹Π΅ поля ΠΎΠΏΡ€Π΅Π΄Π΅Π»ΡΡŽΡ‚ΡΡ ΠΊΠ°ΠΊ Π½Π΅ΠΏΡ€Π΅Ρ€Ρ‹Π²Π½Ρ‹Π΅. Π’Π°ΠΊΠΈΠΌ ΠΎΠ±Ρ€Π°Π·ΠΎΠΌ, ΠΏΠΎΠ»Π΅, исходящСС ΠΎΡ‚ сСвСрного ΠΊΠΎΠ½Ρ†Π° стСрТнСвого ΠΌΠ°Π³Π½ΠΈΡ‚Π°, ΠΎΠ³ΠΈΠ±Π°Π΅Ρ‚ внСшнюю сторону ΠΌΠ°Π³Π½ΠΈΡ‚Π° ΠΊ ΡŽΠΆΠ½ΠΎΠΌΡƒ ΠΏΠΎΠ»ΡŽΡΡƒ ΠΈ возвращаСтся Ρ‡Π΅Ρ€Π΅Π· Ρ‚Π΅Π»ΠΎ ΠΌΠ°Π³Π½ΠΈΡ‚Π° ΠΎΠ±Ρ€Π°Ρ‚Π½ΠΎ ΠΊ сСвСрному ΠΏΠΎΠ»ΡŽΡΡƒ. Π’Π΅ΠΏΠ΅Ρ€ΡŒ ΠΏΡ€Π΅Π΄ΡΡ‚Π°Π²ΡŒΡ‚Π΅ сСбС ΠΌΠ°Π³Π½ΠΈΡ‚, сдСланный ΠΈΠ· ΠΎΡ‡Π΅Π½ΡŒ вязкого ΠΌΠ°Ρ‚Π΅Ρ€ΠΈΠ°Π»Π°, ΠΊΠΎΡ‚ΠΎΡ€Ρ‹ΠΉ позволяСт свободно двиТущСмуся сСвСрному ΠΏΠΎΠ»ΡŽΡΡƒ Π΄Ρ€Π΅ΠΉΡ„ΠΎΠ²Π°Ρ‚ΡŒ Π²Π½ΡƒΡ‚Ρ€ΠΈ Π½Π΅Π³ΠΎ. Π­Ρ‚ΠΎΡ‚ Π²Π½ΡƒΡ‚Ρ€Π΅Π½Π½ΠΈΠΉ сСвСрный полюс отталкиваСтся ΡŽΠΆΠ½Ρ‹ΠΌ полюсом ΠΌΠ°Π³Π½ΠΈΡ‚Π° (?) ΠΈ снова ΡƒΡ…ΠΎΠ΄ΠΈΡ‚ Ρ‡Π΅Ρ€Π΅Π· Π΅Π³ΠΎ сСвСрный полюс. Нас всСх этому ΡƒΡ‡Π°Ρ‚, Π½ΠΎ для мСня это Π½Π΅ ΠΈΠΌΠ΅Π΅Ρ‚ смысла. ΠœΡ‹ Π½Π΅ Π΄ΠΎΠ»ΠΆΠ½Ρ‹ Ρ€Π°ΡΡΠΌΠ°Ρ‚Ρ€ΠΈΠ²Π°Ρ‚ΡŒ ΠΌΠ°Π³Π½ΠΈΡ‚Ρ‹ ΠΊΠ°ΠΊ Π²Π΅Ρ‡Π½Ρ‹Π΅ Π΄Π²ΠΈΠ³Π°Ρ‚Π΅Π»ΠΈ. Π‘ΠΈΠ»Ρ‹ Π½Π°Ρ‡ΠΈΠ½Π°ΡŽΡ‚ΡΡ ΠΈ Π·Π°ΠΊΠ°Π½Ρ‡ΠΈΠ²Π°ΡŽΡ‚ΡΡ Π² Ρ‚ΠΎΡ‡ΠΊΠ°Ρ…: ΠΎΠ½ΠΈ Π½Π΅ Π²Ρ€Π°Ρ‰Π°ΡŽΡ‚ΡΡ ΠΏΠΎ ΠΊΡ€ΡƒΠ³Ρƒ. Π›ΠΈΠ½ΠΈΠΈ ΠΏΠΎΡ‚ΠΎΠΊΠ° ΠΏΠΎΠΊΠ°Π·Ρ‹Π²Π°ΡŽΡ‚, ΠΊΠ°ΠΊ ΠΎΡ‚ΠΊΠ»ΠΎΠ½ΡΡŽΡ‚ΡΡ ΠΌΠ°Π³Π½ΠΈΡ‚Π½Ρ‹Π΅ компасы, Π½ΠΎ Π½ΠΈΡ‡Ρ‚ΠΎ Π½Π΅ Ρ†ΠΈΡ€ΠΊΡƒΠ»ΠΈΡ€ΡƒΠ΅Ρ‚, ΠΊΡ€ΠΎΠΌΠ΅ зарядов. ΠŸΡ€Π΅Π΄ΡΡ‚Π°Π²Π»Π΅Π½ΠΈΠ΅ ΠΎ ΠΊΡ€ΡƒΠ³ΠΎΠ²Ρ‹Ρ… полях, Π²ΠΎΠ·ΠΌΠΎΠΆΠ½ΠΎ, Π²ΠΎΠ·Π½ΠΈΠΊΠ»ΠΎ, ΠΊΠΎΠ³Π΄Π° Π²ΠΎΠΊΡ€ΡƒΠ³ проводящСго ΠΏΡ€ΠΎΠ²ΠΎΠ΄Π° Π±Ρ‹Π»ΠΈ Π·Π°ΠΌΠ΅Ρ‡Π΅Π½Ρ‹ ΠΊΠΎΠ»ΡŒΡ†Π° ΠΈΠ· ΠΆΠ΅Π»Π΅Π·Π½Ρ‹Ρ… ΠΎΠΏΠΈΠ»ΠΎΠΊ, Π½ΠΎ это Π±Ρ‹Π»Π° ΠΎΡ‡Π΅Π½ΡŒ странная идСя. ΠšΡ€ΡƒΠ³ΠΎΠ²ΠΎΠ΅ ΠΌΠ°Π³Π½ΠΈΡ‚Π½ΠΎΠ΅ ΠΏΠΎΠ»Π΅ Π² любой Ρ‚ΠΎΡ‡ΠΊΠ΅ опрСдСляСтся ΠΊΠ°ΠΊ Π²Π΅ΠΊΡ‚ΠΎΡ€, пСрпСндикулярный создаваСмой ΠΈΠΌ ΠΌΠ°Π³Π½ΠΈΡ‚Π½ΠΎΠΉ силС. Однако, Ссли Π²Π΅ΠΊΡ‚ΠΎΡ€ прСдставляСт Ρ‡Ρ‚ΠΎ-Ρ‚ΠΎ, Ρ‡Ρ‚ΠΎ явно сущСствуСт, Π½Π°ΠΏΡ€ΠΈΠΌΠ΅Ρ€. физичСской силы ΠΈΠ»ΠΈ скорости Π²Π΅Ρ‚Ρ€Π°, Π΅Π΅ пСрпСндикулярныС ΡΠΎΡΡ‚Π°Π²Π»ΡΡŽΡ‰ΠΈΠ΅ Ρ€Π°Π²Π½Ρ‹ Π½ΡƒΠ»ΡŽ, Ρ‚. Π΅. ΠΎΠ½Π° Π½Π΅ дСйствуСт Π² пСрпСндикулярном Π½Π°ΠΏΡ€Π°Π²Π»Π΅Π½ΠΈΠΈ. Π’Π°ΠΊΠΈΠΌ ΠΎΠ±Ρ€Π°Π·ΠΎΠΌ, ΠΌΡ‹ ΠΌΠΎΠΆΠ΅ΠΌ ΡΠΊΠ°Π·Π°Ρ‚ΡŒ, Ρ‡Ρ‚ΠΎ Ρ†ΠΈΡ€ΠΊΡƒΠ»ΠΈΡ€ΡƒΡŽΡ‰Π΅Π³ΠΎ ΠΌΠ°Π³Π½ΠΈΡ‚Π½ΠΎΠ³ΠΎ поля, ΠΎΠΊΠ°Π·Ρ‹Π²Π°ΡŽΡ‰Π΅Π³ΠΎ наибольшСС влияниС Π² пСрпСндикулярном Π½Π°ΠΏΡ€Π°Π²Π»Π΅Π½ΠΈΠΈ, Π½Π΅ сущСствуСт. ΠœΠ°Π³Π½ΠΈΡ‚Π½ΠΎΠ΅ ΠΏΠΎΠ»Π΅ Π½Π΅ Π±ΠΎΠ»Π΅Π΅ Π·Π°Π³Π°Π΄ΠΎΡ‡Π½ΠΎ, Ρ‡Π΅ΠΌ элСктричСскоС.
— Π­Π½Π΄Ρ€ΡŽ (67 Π»Π΅Ρ‚)
Π¨Ρ€ΠΎΠΏΡˆΠΈΡ€, Англия

A:

Π’Ρ‹ ΠΏΡ€Π°Π²Ρ‹ Π² Ρ‚ΠΎΠΌ, Ρ‡Ρ‚ΠΎ силы ΠΌΠ΅ΠΆΠ΄Ρƒ классичСскими тСчСниями с достаточно статичным располоТСниСм ΠΌΠΎΠ³ΡƒΡ‚ Π±Ρ‹Ρ‚ΡŒ Π²Ρ‹Ρ€Π°ΠΆΠ΅Π½Ρ‹ нСпосрСдствСнно Ρ‡Π΅Ρ€Π΅Π· тСчСния ΠΈ смСщСния ΠΌΠ΅ΠΆΠ΄Ρƒ Π½ΠΈΠΌΠΈ. Π­Ρ‚ΠΎ Π½Π΅ Ρ‚Π°ΠΊ просто, ΠΊΠ°ΠΊ элСктростатика, ΠΏΠΎΡ‚ΠΎΠΌΡƒ Ρ‡Ρ‚ΠΎ Π½Π°ΠΏΡ€Π°Π²Π»Π΅Π½ΠΈΠ΅ смСщСния ΠΈ направлСния Ρ‚ΠΎΠΊΠΎΠ² входят Π² нСсколько слоТнСС. ΠΏΡƒΡ‚ΡŒ. Но всС ΠΆΠ΅ Π² этом случаС использованиС описаний ΠΌΠ°Π³Π½ΠΈΡ‚Π½ΠΎΠ³ΠΎ поля являСтся лишь Π΄ΠΎΠΏΠΎΠ»Π½ΠΈΡ‚Π΅Π»ΡŒΠ½Ρ‹ΠΌ матСматичСским удобством. Как Ρ‚ΠΎΠ»ΡŒΠΊΠΎ Π²Ρ‹ Π½Π°Ρ‡Π½Π΅Ρ‚Π΅ Π³ΠΎΠ²ΠΎΡ€ΠΈΡ‚ΡŒ ΠΎ ускоряСт классичСских зарядов, ΠΎΠ΄Π½Π°ΠΊΠΎ Π² Π΄Π΅Π»ΠΎ Π²ΡΡ‚ΡƒΠΏΠ°ΡŽΡ‚ ΠΏΠΎΠ»Π½Ρ‹Π΅ уравнСния элСктромагнитного поля. Π‘Ρ‹Π»ΠΎ Π±Ρ‹ ΠΊΡ€Π°ΠΉΠ½Π΅ Π½Π΅ΡƒΠ΄ΠΎΠ±Π½ΠΎ ΠΎΠΏΠΈΡΡ‹Π²Π°Ρ‚ΡŒ элСктромагнитныС Π²ΠΎΠ»Π½Ρ‹ (Π½Π°ΠΏΡ€ΠΈΠΌΠ΅Ρ€, Ρ€Π°Π΄ΠΈΠΎΠ²ΠΎΠ»Π½Ρ‹ ΠΈ свСт) Π² обозначСниях, основанных Π½Π° зарядах ΠΈ Ρ‚ΠΎΠΊΠ°Ρ…, ΠΏΠΎΡ‚ΠΎΠΌΡƒ Ρ‡Ρ‚ΠΎ эти Π²ΠΎΠ»Π½Ρ‹ свободно Ρ€Π°ΡΠΏΡ€ΠΎΡΡ‚Ρ€Π°Π½ΡΡŽΡ‚ΡΡ Π²Π΄Π°Π»ΠΈ ΠΎΡ‚ Π»ΡŽΠ±Ρ‹Ρ… зарядов ΠΈ Ρ‚ΠΎΠΊΠΎΠ².

Как Ρ‚ΠΎΠ»ΡŒΠΊΠΎ Π²Ρ‹ Π·Π°Ρ…ΠΎΡ‚ΠΈΡ‚Π΅ Π²ΠΊΠ»ΡŽΡ‡ΠΈΡ‚ΡŒ ΠΊΠ²Π°Π½Ρ‚ΠΎΠ²Ρ‹Π΅ спины, описания, ΠΊΠΎΡ‚ΠΎΡ€Ρ‹Π΅ Π½Π΅ ΡƒΡ‡ΠΈΡ‚Ρ‹Π²Π°ΡŽΡ‚ ΠΌΠ°Π³Π½ΠΈΡ‚Π½Ρ‹Π΅ поля ΠΈΠ»ΠΈ Π΄Π°ΠΆΠ΅ Π±ΠΎΠ»Π΅Π΅ абстрактныС сущности (Π²Π΅ΠΊΡ‚ΠΎΡ€Π½Ρ‹Π΅ ΠΏΠΎΡ‚Π΅Π½Ρ†ΠΈΠ°Π»Ρ‹,. ..), становятся бСсполСзными. ВсС практичСскиС ΠΌΠ°Π³Π½ΠΈΡ‚Π½Ρ‹Π΅ ΠΌΠ°Ρ‚Π΅Ρ€ΠΈΠ°Π»Ρ‹ Π²ΠΊΠ»ΡŽΡ‡Π°ΡŽΡ‚ Ρ‚Π°ΠΊΠΈΠ΅ спины.

Майк Π’.

(ΠΎΠΏΡƒΠ±Π»ΠΈΠΊΠΎΠ²Π°Π½ΠΎ 26.07.2017)

Π”ΠΎΠΏΠΎΠ»Π½Π΅Π½ΠΈΠ΅ β„–8: ΠΊΠ²Π°Π½Ρ‚ΠΎΠ²Ρ‹Π΅ Ρ„Π»ΡƒΠΊΡ‚ΡƒΠ°Ρ†ΠΈΠΈ Ρ„ΠΎΡ‚ΠΎΠ½Ρ‹ … постоянно ΠΈΡΠΏΡƒΡΠΊΠ°ΡŽΡ‚ΡΡ ΠΈ ΠΏΠΎΠ²Ρ‚ΠΎΡ€Π½ΠΎ ΠΏΠΎΠ³Π»ΠΎΡ‰Π°ΡŽΡ‚ΡΡ элСктроном», Π½ΠΎ Π½Π° самом Π΄Π΅Π»Π΅ ΠΌΡ‹ ΠΈΠΌΠ΅Π΅ΠΌ Π² Π²ΠΈΠ΄Ρƒ Π½Π΅ это. Π”Π²Π΅ элСктромагнитно Π²Π·Π°ΠΈΠΌΠΎΠ΄Π΅ΠΉΡΡ‚Π²ΡƒΡŽΡ‰ΠΈΠ΅ частицы Π΄Π΅ΠΉΡΡ‚Π²ΠΈΡ‚Π΅Π»ΡŒΠ½ΠΎ ΠΎΠΊΡ€ΡƒΠΆΠ΅Π½Ρ‹ Π²ΠΈΡ€Ρ‚ΡƒΠ°Π»ΡŒΠ½Ρ‹ΠΌ Ρ„ΠΎΡ‚ΠΎΠ½Π½Ρ‹ΠΌ ΠΎΠ±Π»Π°ΠΊΠΎΠΌ. Π°Ρ‚ΠΎΠΌ) это ΠΎΠ±Π»Π°ΠΊΠΎ ΡΠΎΠ²Π΅Ρ€ΡˆΠ΅Π½Π½ΠΎ Π½Π΅ΠΈΠ·ΠΌΠ΅Π½Π½ΠΎ Π²ΠΎ Π²Ρ€Π΅ΠΌΠ΅Π½ΠΈ. Π’ΠΎΠΎΠ±Ρ‰Π΅ Π½ΠΈΡ‡Π΅Π³ΠΎ Π½Π΅ происходит. Π‘Π»ΠΎΠ²Π° ΠΎ Π²Π΅Ρ‰Π°Ρ…, Ρ„Π»ΡƒΠΊΡ‚ΡƒΠΈΡ€ΡƒΡŽΡ‰ΠΈΡ… Π²ΠΎΠΊΡ€ΡƒΠ³, β€” Π³Ρ€ΡƒΠ±Ρ‹ΠΉ способ ΠΏΠ΅Ρ€Π΅Π΄Π°Ρ‚ΡŒ ΠΎΠ΄Π½ΠΎ ΠΈΠ· спСцифичСских свойств ΠΊΠ²Π°Π½Ρ‚ΠΎΠ²Ρ‹Ρ… ΠΏΠΎΠ»Π΅ΠΉΒ». Как взаимодСйствиС ΠΌΠ΅ΠΆΠ΄Ρƒ двумя частицами ΠΌΠΎΠΆΠ΅Ρ‚ привСсти ΠΊ ΡΠΎΡΡ‚ΠΎΡΠ½ΠΈΡŽ, Π² ΠΊΠΎΡ‚ΠΎΡ€ΠΎΠΌ Β«Π²ΠΎΠΎΠ±Ρ‰Π΅ Π½ΠΈΡ‡Π΅Π³ΠΎ Π½Π΅ происходит»? Π Π°Π·Π²Π΅ физичСскоС взаимодСйствиС Π½Π΅ ΠΎΠ±ΡΠ·Π°Ρ‚Π΅Π»ΡŒΠ½ΠΎ ΠΏΡ€ΠΈΠ²ΠΎΠ΄ΠΈΡ‚ ΠΊ ΠΊΠ°ΠΊΠΈΠΌ-Ρ‚ΠΎ измСнСниям? Π”Π°ΠΆΠ΅ Π² чистом Π²Π°ΠΊΡƒΡƒΠΌΠ΅ ΠΌΠ½ΠΎΠ³ΠΎΠ΅ происходит, Ρ‚ΠΎ Π΅ΡΡ‚ΡŒ ΡΡƒΡ‰Π΅ΡΡ‚Π²ΡƒΡŽΡ‚ Ρ„Π»ΡƒΠΊΡ‚ΡƒΠ°Ρ†ΠΈΠΈ Π½ΡƒΠ»Π΅Π²ΠΎΠΉ энСргии. НС ΠΌΠΎΠ³Π»ΠΈ Π±Ρ‹ Π²Ρ‹ ΠΏΠΎΠ΄Ρ€ΠΎΠ±Π½Π΅Π΅ Ρ€Π°ΡΡΠΊΠ°Π·Π°Ρ‚ΡŒ ΠΎ Ρ‚ΠΎΠΌ, Ρ‡Ρ‚ΠΎ Π²Ρ‹ ΠΈΠΌΠ΅Π΅Ρ‚Π΅ Π² Π²ΠΈΠ΄Ρƒ ΠΎΡ‚Π½ΠΎΡΠΈΡ‚Π΅Π»ΡŒΠ½ΠΎ взаимодСйствий ΠΌΠ΅ΠΆΠ΄Ρƒ Π°Ρ‚ΠΎΠΌΠ½Ρ‹ΠΌΠΈ частицами ΠΈ Π²ΠΈΡ€Ρ‚ΡƒΠ°Π»ΡŒΠ½Ρ‹ΠΌΠΈ Ρ„ΠΎΡ‚ΠΎΠ½Π°ΠΌΠΈ Π² этом смыслС? ΠšΡ€ΠΎΠΌΠ΅ Ρ‚ΠΎΠ³ΠΎ, ΠΏΠΎΡ‡Π΅ΠΌΡƒ ΠΈΡΠΏΠΎΠ»ΡŒΠ·ΡƒΠ΅Ρ‚ΡΡ язык, ΡƒΠΊΠ°Π·Ρ‹Π²Π°ΡŽΡ‰ΠΈΠΉ Π½Π° ΠΈΠ½Ρ‚Π΅Ρ€Π°ΠΊΡ‚ΠΈΠ²Π½ΠΎΠ΅ Π΄Π²ΠΈΠΆΠ΅Π½ΠΈΠ΅ ΠΌΠ΅ΠΆΠ΄Ρƒ частицами («флуктуация Π²ΠΎΠΊΡ€ΡƒΠ³Β»), Ссли Π² субатомных ΠΎΠ±Π»Π°ΠΊΠ°Ρ…, Ρ‚Π°ΠΊΠΈΡ… ΠΊΠ°ΠΊ Π²ΠΈΡ€Ρ‚ΡƒΠ°Π»ΡŒΠ½Ρ‹Π΅ Ρ„ΠΎΡ‚ΠΎΠ½Ρ‹, Π½Π΅Ρ‚ двиТСния, ΠΈ Π½Π΅ ΠΌΠΎΠ³Π»ΠΈ Π±Ρ‹ Π²Ρ‹ Π±ΠΎΠ»Π΅Π΅ Ρ‡Π΅Ρ‚ΠΊΠΎ ΠΎΠ±ΡŠΡΡΠ½ΠΈΡ‚ΡŒ, ΠΊΠ°ΠΊ Ρ„ΠΈΠ·ΠΈΠΊΠ° двиТСния Π½Π°Ρ€ΡƒΡˆΠ°Π΅Ρ‚ΡΡ Π½Π° субатомном ΡƒΡ€ΠΎΠ²Π½Π΅? -Π°Ρ‚ΠΎΠΌΠ½Ρ‹Ρ… ΠΌΠ°ΡΡˆΡ‚Π°Π±Π°Ρ…, Ссли это Π΄Π΅ΠΉΡΡ‚Π²ΠΈΡ‚Π΅Π»ΡŒΠ½ΠΎ Ρ‚ΠΎ, Ρ‡Ρ‚ΠΎ происходит?


— Π ΠΎΠ΄Ρ€ΠΈ ΠžΡ€Π΄Π΅Π½Ρ (33 Π³ΠΎΠ΄Π°)
Дуглас, остров Мэн, БританскиС острова

A:

ЀизичСскоС взаимодСйствиС Π½Π΅ ΠΎΠ±ΡΠ·Π°Ρ‚Π΅Π»ΡŒΠ½ΠΎ ΠΎΠ·Π½Π°Ρ‡Π°Π΅Ρ‚, Ρ‡Ρ‚ΠΎ Ρ‡Ρ‚ΠΎ-Ρ‚ΠΎ происходит, ΠΏΠΎ ΠΊΡ€Π°ΠΉΠ½Π΅ΠΉ ΠΌΠ΅Ρ€Π΅, Π² ΠΎΠ±Ρ‹Ρ‡Π½ΠΎΠΌ смыслС этого слова. НапримСр, ΠΏΡ€Π΅Π΄ΡΡ‚Π°Π²ΡŒΡ‚Π΅ сСбС ΠΊΠΎΡ€ΠΎΠ±ΠΊΡƒ, ΡΡ‚ΠΎΡΡ‰ΡƒΡŽ Π½Π° ΠΏΠΎΠ»Ρƒ. ΠšΠΎΡ€ΠΎΠ±ΠΊΠ° ΠΈ ΠΏΠΎΠ», бСзусловно, Π²Π·Π°ΠΈΠΌΠΎΠ΄Π΅ΠΉΡΡ‚Π²ΡƒΡŽΡ‚. Но ΠΌΠ°Π»ΠΎ Ρ‡Ρ‚ΠΎ происходит. НичСго Π½Π΅ мСняСтся.

Β«Π”Π°ΠΆΠ΅ Π² чистом Π²Π°ΠΊΡƒΡƒΠΌΠ΅ ΠΌΠ½ΠΎΠ³ΠΎΠ΅ происходит, Ρ‚. Π΅. ΡΡƒΡ‰Π΅ΡΡ‚Π²ΡƒΡŽΡ‚ Ρ„Π»ΡƒΠΊΡ‚ΡƒΠ°Ρ†ΠΈΠΈ Π½ΡƒΠ»Π΅Π²ΠΎΠΉ энСргии». Π­Ρ‚ΠΎ Π²Π΅Ρ€Π½ΠΎ Π² ΡΠ»Π΅Π΄ΡƒΡŽΡ‰Π΅ΠΌ смыслС: значСния Ρ€Π°Π·Π»ΠΈΡ‡Π½Ρ‹Ρ… ΠΏΠΎΠ»Π΅ΠΉ Π½Π΅ Ρ„ΠΈΠΊΡΠΈΡ€ΡƒΡŽΡ‚ΡΡ Ρ‚ΠΎΡ‡Π½ΠΎ Π½Π° Π½ΡƒΠ»Π΅, Π° ΠΈΠΌΠ΅ΡŽΡ‚ распространяСтся Π²ΠΎΠΊΡ€ΡƒΠ³ этого, Ρ‚ΠΎΡ‡Π½ΠΎ Ρ‚Π°ΠΊ ΠΆΠ΅, ΠΊΠ°ΠΊ распространяСтся ΠΏΠΎΠ»ΠΎΠΆΠ΅Π½ΠΈΠ΅ Π²ΠΎΠ»Π½Ρ‹. Π’Π΅ΠΌ Π½Π΅ ΠΌΠ΅Π½Π΅Π΅, разброс поля Π½Π΅ мСняСтся Π²ΠΎ Π²Ρ€Π΅ΠΌΠ΅Π½ΠΈ.

Π― Π΄ΡƒΠΌΠ°ΡŽ, Ρ‡Ρ‚ΠΎ Π½Π΅Π±Ρ€Π΅ΠΆΠ½Ρ‹ΠΉ язык ΠΈΡΠΏΠΎΠ»ΡŒΠ·ΡƒΠ΅Ρ‚ΡΡ, ΠΏΠΎΡ‚ΠΎΠΌΡƒ Ρ‡Ρ‚ΠΎ ΠΌΡ‹ инстинктивно пытаСмся Π²Ρ‚ΠΈΡΠ½ΡƒΡ‚ΡŒ ΠΊΠ²Π°Π½Ρ‚ΠΎΠ²ΡƒΡŽ Ρ€Π΅Π°Π»ΡŒΠ½ΠΎΡΡ‚ΡŒ Π² классичСскиС ΠΊΠ°Ρ€Ρ‚ΠΈΠ½ΠΊΠΈ.

Mike W.

(ΠΎΠΏΡƒΠ±Π»ΠΈΠΊΠΎΠ²Π°Π½ΠΎ 30.09.2017)

Π”ΠΎΠΏΠΎΠ»Π½Π΅Π½ΠΈΠ΅ β„–9: прСдполоТСния ΠΎΠ± элСктромагнитном ΠΏΠΎΠ»Π΅

Q:

ΠŸΡ€Π΅ΠΆΠ΄Π΅ всСго, сущСствуСт большая Ρ€Π°Π·Π½ΠΈΡ†Π° ΠΌΠ΅ΠΆΠ΄Ρƒ статичСскими полями ΠΈ элСктромагнитной ΠΈΠ½Π΄ΡƒΠΊΡ†ΠΈΠ΅ΠΉ . Богласно ΠΎΡ„ΠΈΡ†ΠΈΠ°Π»ΡŒΠ½ΠΎΠΉ Π½Π°ΡƒΠΊΠ΅, ΠΌΠ°Π³Π½Π΅Ρ‚ΠΈΠ·ΠΌ являСтся Β«ΠΏΠΎΠ±ΠΎΡ‡Π½Ρ‹ΠΌ эффСктом» элСктричСского Ρ‚ΠΎΠΊΠ°, ΠΈ ΠΎΠ±Π΅ силы (магнитная ΠΈ элСктричСская) ΡΠ²Π»ΡΡŽΡ‚ΡΡ лишь двумя сторонами ΠΎΠ΄Π½ΠΎΠΉ ΠΌΠ΅Π΄Π°Π»ΠΈ (элСктромагнСтизм). ЭлСктростатичСскоС ΠΈ магнитостатичСскоС поля β€” это Π΄Π²Π° Ρ€Π°Π·Π½Ρ‹Ρ… аспСкта физичСской ΠΌΠ°Ρ‚Π΅Ρ€ΠΈΠΈ. ЭлСктростатичСскоС ΠΏΠΎΠ»Π΅ Π½Π΅ влияСт Π½Π° стрСлку компаса, Π½ΠΎ ΠΎΠ½ΠΎ влияСт Π½Π° ΠΌΠ΅Ρ‚Π°Π»Π»Ρ‹, Ρ‚Π°ΠΊΠΈΠ΅ ΠΊΠ°ΠΊ алюминий, Π½Π° ΠΊΠΎΡ‚ΠΎΡ€Ρ‹Π΅ магнитостатичСскоС ΠΏΠΎΠ»Π΅ Π½Π΅ ΠΎΠΊΠ°Π·Ρ‹Π²Π°Π΅Ρ‚ Π²ΠΈΠ΄ΠΈΠΌΠΎΠ³ΠΎ дСйствия, Π½ΠΎ явно влияСт Π½Π° Π΄Ρ€ΡƒΠ³ΠΈΠ΅ ΠΌΠ°Π³Π½ΠΈΡ‚Π½Ρ‹Π΅ поля. ЭлСктростатичСскоС ΠΏΠΎΠ»Π΅ создаСтся напряТСниСм ΠΌΠ΅ΠΆΠ΄Ρƒ ΠΏΡ€ΠΎΡ‚ΠΈΠ²ΠΎΠΏΠΎΠ»ΠΎΠΆΠ½Ρ‹ΠΌΠΈ элСктричСскими зарядами. ЭлСктричСская сила «питаСтся» Ρ€Π°Π·Π½ΠΎΡΡ‚ΡŒΡŽ Π²Π΅Π»ΠΈΡ‡ΠΈΠ½, ΠΊΠΎΡ‚ΠΎΡ€Ρ‹Π΅ стрСмятся ΠΎΠ±Π½ΡƒΠ»ΠΈΡ‚ΡŒΡΡ, достигнув Π½Π΅ΠΉΡ‚Ρ€Π°Π»ΡŒΠ½ΠΎΠ³ΠΎ значСния, ΠΏΠΎΠ΄ΠΎΠ±Π½ΠΎ ΠΏΡ€ΠΎΡ‚ΠΈΠ²ΠΎΠΏΠΎΠ»ΠΎΠΆΠ½Ρ‹ΠΌ систСмам давлСния Π²ΠΎΠ·Π΄ΡƒΡ…Π° ΠΈΠ»ΠΈ уровня Π²ΠΎΠ΄Ρ‹ Π² соСдинСнных Смкостях. Π’Π°ΠΌ Π½ΡƒΠΆΠ΅Π½ ΠΊΠ°ΠΊΠΎΠΉ-Ρ‚ΠΎ дисбаланс Π½Π΅ΠΉΡ‚Ρ€Π°Π»ΡŒΠ½ΠΎΠΉ ΠΌΠ°Ρ‚Π΅Ρ€ΠΈΠΈ, Ρ‡Ρ‚ΠΎΠ±Ρ‹ ΡΠ΄Π΅Π»Π°Ρ‚ΡŒ Π΅Π΅ элСктричСски заряТСнной, Ρ‡Ρ‚ΠΎ часто связано с Π΄ΠΎΠΏΠΎΠ»Π½ΠΈΡ‚Π΅Π»ΡŒΠ½ΠΎΠΉ Ρ€Π°Π±ΠΎΡ‚ΠΎΠΉ. ΠœΠ°Π³Π½ΠΈΡ‚Π½ΠΎΠ΅ ΠΏΠΎΠ»Π΅ управляСтся ΠΏΡ€ΠΎΡ‚ΠΈΠ²ΠΎΠΏΠΎΠ»ΠΎΠΆΠ½Ρ‹ΠΌΠΈ ориСнтациями ΠΈ ΠΌΠΎΠΆΠ΅Ρ‚ ΡΠΎΠ·Π΄Π°Π²Π°Ρ‚ΡŒΡΡ элСктричСски Π½Π΅ΠΉΡ‚Ρ€Π°Π»ΡŒΠ½ΠΎΠΉ ΠΌΠ°Ρ‚Π΅Ρ€ΠΈΠ΅ΠΉ. ΠŸΠ΅Ρ€Π½Π°ΠΌΠ΅Π½Ρ‚Π½Ρ‹Π΅ ΠΌΠ°Π³Π½ΠΈΡ‚Ρ‹ Π½Π΅ Ρ‚Ρ€Π΅Π±ΡƒΡŽΡ‚ Π΄ΠΎΠΏΠΎΠ»Π½ΠΈΡ‚Π΅Π»ΡŒΠ½ΠΎΠΉ Ρ€Π°Π±ΠΎΡ‚Ρ‹ для создания ΠΌΠ°Π³Π½ΠΈΡ‚Π½Ρ‹Ρ… ΠΏΠΎΠ»Π΅ΠΉ. ΠŸΡ€ΠΎΡ‚ΠΈΠ²ΠΎΠΏΠΎΠ»ΠΎΠΆΠ½Ρ‹Π΅ полярности ΠΏΡ€ΠΈΡ‚ΡΠ³ΠΈΠ²Π°ΡŽΡ‚ΡΡ Π΄Ρ€ΡƒΠ³ ΠΊ Π΄Ρ€ΡƒΠ³Ρƒ, Π½ΠΎ ΠΎΠ½ΠΈ Π½Π΅ Π½Π΅ΠΉΡ‚Ρ€Π°Π»ΠΈΠ·ΡƒΡŽΡ‚ Π΄Ρ€ΡƒΠ³ Π΄Ρ€ΡƒΠ³Π° β€” Ссли Π²Ρ‹ соСдинитС 2 ΠΌΠ°Π³Π½ΠΈΡ‚Π°, ΠΎΠ½ΠΈ Π±ΡƒΠ΄ΡƒΡ‚ Ρ€Π°Π±ΠΎΡ‚Π°Ρ‚ΡŒ ΠΊΠ°ΠΊ ΠΎΠ΄ΠΈΠ½ ΠΌΠ°Π³Π½ΠΈΡ‚, ΠΈ ΠΌΠ°Π³Π½ΠΈΡ‚Π½ΠΎΠ΅ ΠΏΠΎΠ»Π΅ Π±ΡƒΠ΄Π΅Ρ‚ сильнСС. Π˜ΡΡ‚ΠΎΡ‡Π½ΠΈΠΊΠΎΠΌ элСктричСского поля являСтся элСктричСский заряд субатомной частицы, Π° источником ΠΌΠ°Π³Π½ΠΈΡ‚Π½ΠΎΠ³ΠΎ поля — Π΅Π΅ ΠΊΠ²Π°Π½Ρ‚ΠΎΠ²Ρ‹ΠΉ спин. это Π΄Π²Π° Ρ€Π°Π·Π½Ρ‹Ρ… Π²Π½ΡƒΡ‚Ρ€Π΅Π½Π½ΠΈΡ… свойства частицы, ΠΈ ΠΎΠ±Π° ΠΎΠ΄ΠΈΠ½Π°ΠΊΠΎΠ²ΠΎ Π²Π°ΠΆΠ½Ρ‹ … Богласно ΠΎΡ„ΠΈΡ†ΠΈΠ°Π»ΡŒΠ½ΠΎΠΉ Π½Π°ΡƒΠΊΠ΅, Π°Ρ‚ΠΎΠΌΡ‹ ΠΌΠΎΠ³ΡƒΡ‚ Π³Π΅Π½Π΅Ρ€ΠΈΡ€ΠΎΠ²Π°Ρ‚ΡŒ ΠΌΠ°Π³Π½ΠΈΡ‚Π½Ρ‹Π΅ поля ΠΈΠ·-Π·Π° элСктронов, ΠΊΠΎΡ‚ΠΎΡ€Ρ‹Π΅ двиТутся Π²ΠΎΠΊΡ€ΡƒΠ³ ядра, создавая ΠΌΠ°Π³Π½ΠΈΡ‚Π½ΠΎΠ΅ ΠΏΠΎΠ»Π΅ Π·Π° счСт ΠΈΠ½Π΄ΡƒΠΊΡ†ΠΈΠΈ. ΠŸΡ€ΠΎΠ±Π»Π΅ΠΌΠ° Π² Ρ‚ΠΎΠΌ, Ρ‡Ρ‚ΠΎ концСпция элСктрона, двиТущСгося ΠΏΠΎ ΠΎΡ€Π±ΠΈΡ‚Π΅, ΠΊΠ°ΠΊ ΠΏΠ»Π°Π½Π΅Ρ‚Π° Π²ΠΎΠΊΡ€ΡƒΠ³ Π·Π²Π΅Π·Π΄Ρ‹, ΡΠΎΠ²Π΅Ρ€ΡˆΠ΅Π½Π½ΠΎ Π½Π΅Π²Π΅Ρ€Π½Π°. Π­Π»Π΅ΠΊΡ‚Ρ€ΠΎΠ½Ρ‹ ΡΠΎΠ·Π΄Π°ΡŽΡ‚ ΠΎΠ±Π»Π°ΠΊΠ°, Π² ΠΊΠΎΡ‚ΠΎΡ€Ρ‹Ρ… ΠΈΡ… ΠΏΠΎΠ»ΠΎΠΆΠ΅Π½ΠΈΠ΅, ΡΠΊΠΎΡ€ΠΎΡΡ‚ΡŒ ΠΈ ориСнтация ΠΊΠ²Π°Π½Ρ‚ΠΎΠ²ΠΎΠ³ΠΎ спина ΠΎΡΡ‚Π°ΡŽΡ‚ΡΡ Π² постоянной супСрпозиции ΠΈ Π½Π΅ ΠΎΠΏΡ€Π΅Π΄Π΅Π»ΡΡŽΡ‚ΡΡ Π΄ΠΎ Ρ‚Π΅Ρ… ΠΏΠΎΡ€, ΠΏΠΎΠΊΠ° Π½Π΅ Π±ΡƒΠ΄ΡƒΡ‚ ΠΈΠ·ΠΌΠ΅Ρ€Π΅Π½Ρ‹, поэтому Ρƒ элСктронного ΠΎΠ±Π»Π°ΠΊΠ° Π½Π΅Ρ‚ Π°Π±ΡΠΎΠ»ΡŽΡ‚Π½ΠΎ Π½ΠΈΠΊΠ°ΠΊΠΎΠΉ возмоТности ΠΈΠ½Π΄ΡƒΡ†ΠΈΡ€ΠΎΠ²Π°Ρ‚ΡŒ ΠΎΠΏΡ€Π΅Π΄Π΅Π»Π΅Π½Π½ΠΎΠ΅ ΠΈ ΠΎΠ΄Π½ΠΎΡ€ΠΎΠ΄Π½ΠΎΠ΅ ΠΌΠ°Π³Π½ΠΈΡ‚Π½ΠΎΠ΅ ΠΏΠΎΠ»Π΅. И, Π½Π°ΠΊΠΎΠ½Π΅Ρ†… НСт. Π½Π° самом Π΄Π΅Π»Π΅ извСстно, Ρ‡Ρ‚ΠΎ ΠΌΠΎΠΆΠ΅Ρ‚ Π±Ρ‹Ρ‚ΡŒ физичСским носитСлСм для «Π²ΠΈΡ€Ρ‚ΡƒΠ°Π»ΡŒΠ½Ρ‹Ρ…» силовых Π»ΠΈΠ½ΠΈΠΉ Π² пространствС — Π° статичСскиС поля ΠΌΠΎΠ³ΡƒΡ‚ Π²Π·Π°ΠΈΠΌΠΎΠ΄Π΅ΠΉΡΡ‚Π²ΠΎΠ²Π°Ρ‚ΡŒ Π΄Ρ€ΡƒΠ³ с Π΄Ρ€ΡƒΠ³ΠΎΠΌ Π½Π° ΠžΠ“Π ΠžΠœΠΠžΠœ расстоянии (тысячи свСтовых Π»Π΅Ρ‚) — Ρ‚Π°ΠΊ Ρ‡Ρ‚ΠΎ Π²ΠΈΡ€Ρ‚ΡƒΠ°Π»ΡŒΠ½Ρ‹Π΅ Ρ„ΠΎΡ‚ΠΎΠ½Ρ‹ Ρ€Π°Π±ΠΎΡ‚Π°Ρ‚ΡŒ Π½Π΅ Π±ΡƒΠ΄ΡƒΡ‚. Однако Π½Π° «ΠΎΠ±Ρ‹Ρ‡Π½Ρ‹Π΅» Ρ„ΠΎΡ‚ΠΎΠ½Ρ‹ Π½ΠΈΠΊΠ°ΠΊΠΈΠΌ Π²ΠΈΠ΄ΠΈΠΌΡ‹ΠΌ ΠΎΠ±Ρ€Π°Π·ΠΎΠΌ Π½Π΅ Π²Π»ΠΈΡΡŽΡ‚ элСктростатичСскиС ΠΈ ΠΌΠ°Π³Π½ΠΈΡ‚Π½Ρ‹Π΅ поля — поэтому ΠΏΠΎ Π»ΠΎΠ³ΠΈΠΊΠ΅ ΠΎΠ½ΠΈ Π½Π΅ ΠΌΠΎΠ³ΡƒΡ‚ нСсти ΠΈΡ… ΠΊΠ°ΠΊ ΠΎΡ‚Π΄Π΅Π»ΡŒΠ½Ρ‹Π΅ силы. НО всС элСктромагнитныС Π²ΠΎΠ»Π½Ρ‹ Π²ΠΊΠ»ΡŽΡ‡Π°ΡŽΡ‚ Π² сСбя ΠΌΠ°Π³Π½ΠΈΡ‚Π½Ρ‹Π΅ ΠΈ элСктричСскиС ΠΊΠΎΠΌΠΏΠΎΠ½Π΅Π½Ρ‚Ρ‹. Π­Ρ‚ΠΎ Π½Π΅ Π±ΠΎΠ»Π΅Π΅, Ρ‡Π΅ΠΌ ΠΏΡ€Π΅Π΄ΠΏΠΎΠ»ΠΎΠΆΠ΅Π½ΠΈΠ΅, Π½ΠΎ ΠœΠžΠ–Π•Π’ Π±Ρ‹Ρ‚ΡŒ, статичСскиС поля Π΄Π΅ΠΉΡΡ‚Π²ΠΈΡ‚Π΅Π»ΡŒΠ½ΠΎ ΠΌΠΎΠ³ΡƒΡ‚ ΠΈΠ·ΠΌΠ΅Π½ΠΈΡ‚ΡŒ эти значСния ΠΈ ΠΏΠΎΠ»ΡΡ€ΠΈΠ·ΠΎΠ²Π°Ρ‚ΡŒ свСт, Ρ‡Ρ‚ΠΎ Π±Ρ‹ ΠΏΡ€Π΅Π²Ρ€Π°Ρ‚ΠΈΡ‚ΡŒ Π΅Π³ΠΎ Π² Π½ΠΎΡΠΈΡ‚Π΅Π»ΡŒ силовых Π»ΠΈΠ½ΠΈΠΉ???
— Астрал (возраст 33 Π³ΠΎΠ΄Π°)
Польша

A:

Π—Π΄Π΅ΡΡŒ Π΅ΡΡ‚ΡŒ изъян Π² Π²Π°ΡˆΠΈΡ… рассуТдСниях: «»Π½ΠΎΡ€ΠΌΠ°Π»ΡŒΠ½Ρ‹Π΅» Ρ„ΠΎΡ‚ΠΎΠ½Ρ‹ Π½ΠΈΠΊΠ°ΠΊΠΈΠΌ Π²ΠΈΠ΄ΠΈΠΌΡ‹ΠΌ ΠΎΠ±Ρ€Π°Π·ΠΎΠΌ Π½Π΅ ΠΏΠΎΠ΄Π²Π΅Ρ€ΠΆΠ΅Π½Ρ‹ влиянию элСктростатичСских ΠΈ ΠΌΠ°Π³Π½ΠΈΡ‚Π½Ρ‹Ρ… ΠΏΠΎΠ»Π΅ΠΉ — поэтому ΠΏΠΎ Π»ΠΎΠ³ΠΈΠΊΠ΅ ΠΎΠ½ΠΈ Π½Π΅ ΠΌΠΎΠ³ΡƒΡ‚ ΠΈΡ… нСсти , ΠΊΠ°ΠΊ ΠΎΡ‚Π΄Π΅Π»ΡŒΠ½Ρ‹Π΅ силы». Пока уравнСния для поля Π»ΠΈΠ½Π΅ΠΉΠ½Ρ‹, ΠΊΠ°ΠΊ ΠΈ классичСскиС элСктромагнитныС уравнСния, Π½ΠΈ ΠΎΠ΄ΠΈΠ½ ΠΊΠΎΠΌΠΏΠΎΠ½Π΅Π½Ρ‚ поля Π½Π΅ влияСт Π½Π° ΠΏΠΎΠ²Π΅Π΄Π΅Π½ΠΈΠ΅ любого Π΄Ρ€ΡƒΠ³ΠΎΠ³ΠΎ ΠΊΠΎΠΌΠΏΠΎΠ½Π΅Π½Ρ‚Π°. Π’Π°ΠΊΠΈΠΌ ΠΎΠ±Ρ€Π°Π·ΠΎΠΌ, отсутствиС взаимодСйствия ΠΌΠ΅ΠΆΠ΄Ρƒ статичСскими полями ΠΈ Ρ€Π°ΡΠΏΡ€ΠΎΡΡ‚Ρ€Π°Π½ΡΡŽΡ‰ΠΈΠΌΠΈΡΡ просто Π³ΠΎΠ²ΠΎΡ€ΠΈΡ‚ ΠΎ Ρ‚ΠΎΠΌ, Ρ‡Ρ‚ΠΎ уравнСния Π»ΠΈΠ½Π΅ΠΉΠ½Ρ‹, Π° Π½Π΅ ΠΎ Ρ‚ΠΎΠΌ, ΠΌΠΎΠ³ΡƒΡ‚ Π»ΠΈ ΠΎΠ΄Π½ΠΈ ΠΈ Ρ‚Π΅ ΠΆΠ΅ Ρ„ΡƒΠ½Π΄Π°ΠΌΠ΅Π½Ρ‚Π°Π»ΡŒΠ½Ρ‹Π΅ Ρ‚ΠΈΠΏΡ‹ ΠΏΠΎΠ»Π΅ΠΉ Π΄Π΅ΠΌΠΎΠ½ΡΡ‚Ρ€ΠΈΡ€ΠΎΠ²Π°Ρ‚ΡŒ Ρ‚Π°ΠΊΠΎΠ΅ Ρ€Π°Π·Π»ΠΈΡ‡Π½ΠΎΠ΅ ΠΏΠΎΠ²Π΅Π΄Π΅Π½ΠΈΠ΅.

Π’Π°Ρˆ Π°Ρ€Π³ΡƒΠΌΠ΅Π½Ρ‚ ΠΎ Ρ‚ΠΎΠΌ, Ρ‡Ρ‚ΠΎ Π²ΠΈΡ€Ρ‚ΡƒΠ°Π»ΡŒΠ½Ρ‹Π΅ Ρ„ΠΎΡ‚ΠΎΠ½Ρ‹ Π½Π΅ ΠΌΠΎΠ³ΡƒΡ‚ Π²Ρ‹Π·Ρ‹Π²Π°Ρ‚ΡŒ Π΄Π°Π»ΡŒΠ½ΠΎΠ΄Π΅ΠΉΡΡ‚Π²ΡƒΡŽΡ‰ΠΈΡ… (1/ΠΊΠ²Π°Π΄Ρ€Π°Ρ‚ расстояния) статичСских взаимодСйствий, Π½Π΅Π²Π΅Ρ€Π΅Π½. ИмСнно Ρ‚Π°ΠΊΠΎΠ΅ ΠΏΠΎΠ²Π΅Π΄Π΅Π½ΠΈΠ΅ оТидаСтся для Π²ΠΈΡ€Ρ‚ΡƒΠ°Π»ΡŒΠ½ΠΎΠΉ Ρ„ΠΎΡ‚ΠΎΠ½Π½ΠΎΠΉ ΠΊΠ°Ρ€Ρ‚ΠΈΠ½Ρ‹.

Mike W.

(ΠΎΠΏΡƒΠ±Π»ΠΈΠΊΠΎΠ²Π°Π½ΠΎ 01.11.2017)

Π”ΠΎΠΏΠΎΠ»Π½Π΅Π½ΠΈΠ΅ β„–10: поляризованный свСт ΠΈ компасы для ΠΏΡ‚ΠΈΡ†

Q:

ΠΌΠ°Π³Π½ΠΈΡ‚Π½Ρ‹Π΅ ΠΈΠ»ΠΈ элСктростатичСскиС поля. На самом Π΄Π΅Π»Π΅, Π΅ΡΡ‚ΡŒ Π½Π΅Π΄Π°Π²Π½Π΅Π΅ исслСдованиС, ΠΊΠΎΡ‚ΠΎΡ€ΠΎΠ΅ ΠΏΠΎΠΊΠ°Π·Ρ‹Π²Π°Π΅Ρ‚, Ρ‡Ρ‚ΠΎ Β«Π²Π½ΡƒΡ‚Ρ€Π΅Π½Π½ΠΈΠΉ компас» ΠΏΡ‚ΠΈΡ† сходит с ΡƒΠΌΠ°, ΠΊΠΎΠ³Π΄Π° ΠΎΠ½ΠΈ ΠΏΠΎΠ΄Π²Π΅Ρ€Π³Π°ΡŽΡ‚ΡΡ Π²ΠΎΠ·Π΄Π΅ΠΉΡΡ‚Π²ΠΈΡŽ поляризованного свСта: http://physicsworld.com/cws/article/news/2016/feb/03/polarized -свСт-сбрасываСт-ΠΏΡ‚ΠΈΡ†-ΠΌΠ°Π³Π½ΠΈΡ‚Π½Ρ‹ΠΉ-компас-с курсаГоворят, Ρ‡Ρ‚ΠΎ поляризованный свСт позволяСт ΠΏΡ‚ΠΈΡ†Π°ΠΌ Β«Π²ΠΎΡΠΏΡ€ΠΈΠ½ΠΈΠΌΠ°Ρ‚ΡŒΒ» мСстноС ΠΌΠ°Π³Π½ΠΈΡ‚Π½ΠΎΠ΅ ΠΏΠΎΠ»Π΅. Π­Ρ‚ΠΎ, вСроятно, Π΄ΠΎΠΊΠ°Π·Ρ‹Π²Π°Π΅Ρ‚, Ρ‡Ρ‚ΠΎ информация ΠΎ ΠΏΠΎΠ»Π΅ «записываСтся» Π² поляризации свСтовой Π²ΠΎΠ»Π½Ρ‹…
— Астрал (возраст 33 Π³ΠΎΠ΄Π°)
Польша

A:

Π­Ρ‚ΠΎ ΠΎΡ‡Π΅Π½ΡŒ классный Ρ€Π΅Π·ΡƒΠ»ΡŒΡ‚Π°Ρ‚, ΡƒΠ±Π΅Π΄ΠΈΡ‚Π΅Π»ΡŒΠ½ΠΎ ΠΏΠΎΠ΄Ρ‚Π²Π΅Ρ€ΠΆΠ΄Π°ΡŽΡ‰ΠΈΠΉ Ρ‚Π΅ΠΎΡ€ΠΈΡŽ ΠΌΠΎΠ΅Π³ΠΎ ΠΏΠΎΠΊΠΎΠΉΠ½ΠΎΠ³ΠΎ ΠΊΠΎΠ»Π»Π΅Π³ΠΈ ΠšΠ»Π°ΡƒΡΠ° Π¨ΡƒΠ»ΡŒΡ‚Π΅Π½Π° ΠΎΠ± ΠΎΠ΄Π½ΠΎΠΌ способС восприятия ΠΏΡ‚ΠΈΡ†Π°ΠΌΠΈ ΠΌΠ°Π³Π½ΠΈΡ‚Π½Ρ‹Ρ… ΠΏΠΎΠ»Π΅ΠΉ. ΠœΠ΅Ρ…Π°Π½ΠΈΠ·ΠΌ Π²ΠΊΠ»ΡŽΡ‡Π°Π΅Ρ‚ Π½Π΅ΠΊΠΎΡ‚ΠΎΡ€Ρ‹Π΅ нСравновСсныС скорости химичСских Ρ€Π΅Π°ΠΊΡ†ΠΈΠΉ, ΠΊΠΎΡ‚ΠΎΡ€Ρ‹Π΅ Π² Π½Π΅ΠΎΠ±Ρ‹Ρ‡Π°ΠΉΠ½ΠΎΠΉ стСпСни зависят ΠΎΡ‚ ΠΌΠ°Π³Π½ΠΈΡ‚Π½Ρ‹Ρ… ΠΏΠΎΠ»Π΅ΠΉ. ΠŸΠΎΠ»ΡΡ€ΠΈΠ·Π°Ρ†ΠΈΡ свСта слуТит Ρ‚ΠΎΠ»ΡŒΠΊΠΎ для Ρ‚ΠΎΠ³ΠΎ, Ρ‡Ρ‚ΠΎΠ±Ρ‹ Π²Ρ‹Π·Π²Π°Ρ‚ΡŒ ΠΏΡ€Π°Π²ΠΈΠ»ΡŒΠ½ΡƒΡŽ Ρ…ΠΈΠΌΠΈΡŽ Π² Π³Π»Π°Π·Π°Ρ… ΠΏΡ‚ΠΈΡ†. Π˜Π½Ρ„ΠΎΡ€ΠΌΠ°Ρ†ΠΈΡ ΠΎ ΠΏΠΎΠ»Π΅ Π²ΠΎΠΎΠ±Ρ‰Π΅ отсутствуСт Π² свСтовой Π²ΠΎΠ»Π½Π΅.

Майк Π’.

(ΠΎΠΏΡƒΠ±Π»ΠΈΠΊΠΎΠ²Π°Π½ΠΎ 10.11.2017)

Π”ΠΎΠΏΠΎΠ»Π½Π΅Π½ΠΈΠ΅ β„–11: ΠΌΠ°Π³Π½ΠΈΡ‚ΠΎΠΎΠΏΡ‚ΠΈΠΊΠ°

Вопрос:

А ΠΊΠ°ΠΊ насчСт магнитооптичСских Π΄Π°Ρ‚Ρ‡ΠΈΠΊΠΎΠ², способных Ρ€Π΅Π°Π»ΡŒΠ½ΠΎ Π²ΠΈΠ·ΡƒΠ°Π»ΠΈΠ·ΠΈΡ€ΠΎΠ²Π°Ρ‚ΡŒ ΠΌΠ°Π³Π½ΠΈΡ‚Π½Ρ‹Π΅ поля с ΠΏΠΎΠΌΠΎΡ‰ΡŒΡŽ поляризованных Π»Π΅Π³ΠΊΠΈΠΉ: https://www.rdmag.com/content/new-sensors-optically-visualize-magnet-fields На Π»ΠΈΠ½Π΅ΠΉΠ½ΠΎ поляризованный свСт Π²ΠΎΠ·Π΄Π΅ΠΉΡΡ‚Π²ΡƒΡŽΡ‚ ΠΌΠ°Π³Π½ΠΈΡ‚Π½Ρ‹Π΅ поля, ΠΈ этот эффСкт ΠΌΠΎΠΆΠ½ΠΎ Π½Π°Π±Π»ΡŽΠ΄Π°Ρ‚ΡŒ Π² Ρ€Π΅ΠΆΠΈΠΌΠ΅ Ρ€Π΅Π°Π»ΡŒΠ½ΠΎΠ³ΠΎ Π²Ρ€Π΅ΠΌΠ΅Π½ΠΈ. Π’Π°ΠΊ Ρ‡Ρ‚ΠΎ ΠΆΠ΅ являСтся Π² Π΄Π°Π½Π½ΠΎΠΌ случаС Π½Π°ΠΈΠ±ΠΎΠ»Π΅Π΅ ΠΎΡ‡Π΅Π²ΠΈΠ΄Π½Ρ‹ΠΌ носитСлСм ΠΈΠ½Ρ„ΠΎΡ€ΠΌΠ°Ρ†ΠΈΠΈ ΠΎ ΠΌΠ°Π³Π½ΠΈΡ‚Π½Ρ‹Ρ… полях? Π€ΠΎΡ‚ΠΎΠ½Ρ‹ — Π° Ρ‚ΠΎΡ‡Π½Π΅Π΅ магнитная ΡΠΎΡΡ‚Π°Π²Π»ΡΡŽΡ‰Π°Ρ ЭМ Π²ΠΎΠ»Π½. Бвойства Ρ„ΠΎΡ‚ΠΎΠ½ΠΎΠ² ΠΎΠΏΡ€Π΅Π΄Π΅Π»ΡΡŽΡ‚ΡΡ источником излучСния, ΠΊΠ°ΠΊ супСрпозициСй 3-Ρ… ΡΠΎΡΡ‚Π°Π²Π»ΡΡŽΡ‰ΠΈΡ… (ЭМ ΠΈ распространСния). ЛинСйная поляризация свСта опрСдСляСт Π΅Π³ΠΎ распространСниС β€” ΠΏΡ€ΠΈ этом воздСйствиС внСшнСго поля влияСт Π½Π° элСктромагнитныС ΡΠΎΡΡ‚Π°Π²Π»ΡΡŽΡ‰ΠΈΠ΅ β€” ΠΈ этот эффСкт ΠΌΠΎΠΆΠ½ΠΎ ΠΈΠ·ΠΌΠ΅Ρ€ΠΈΡ‚ΡŒ.
— Астрал (возраст 33 Π³ΠΎΠ΄Π°)
Польша

A:

МногиС ΠΌΠ°Ρ‚Π΅Ρ€ΠΈΠ°Π»Ρ‹ Π΄Π΅ΠΌΠΎΠ½ΡΡ‚Ρ€ΠΈΡ€ΡƒΡŽΡ‚ эффСкты ЀарадСя ΠΈ эффСкты ΠšΠ΅Ρ€Ρ€Π° — способы, Π² ΠΊΠΎΡ‚ΠΎΡ€Ρ‹Ρ… распространСниС ΠΈΠ»ΠΈ ΠΎΡ‚Ρ€Π°ΠΆΠ΅Π½ΠΈΠ΅ поляризованного свСта зависит ΠΎΡ‚ ΠΌΠ°Π³Π½ΠΈΡ‚Π½ΠΎΠ³ΠΎ поля Π½Π° ΠΌΠ°Ρ‚Π΅Ρ€ΠΈΠ°Π»Π΅. Π£Ρ‚Π²Π΅Ρ€ΠΆΠ΄Π΅Π½ΠΈΠ΅, Ρ‡Ρ‚ΠΎ ΠΌΠ°Π³Π½ΠΈΡ‚Π½ΠΎΠ΅ ΠΏΠΎΠ»Π΅ измСняСт оптичСскиС свойства ΠΌΠ°Ρ‚Π΅Ρ€ΠΈΠ°Π»ΠΎΠ², сильно отличаСтся ΠΎΡ‚ утвСрТдСния, Ρ‡Ρ‚ΠΎ сами свСтовыС Π²ΠΎΠ»Π½Ρ‹ ΡƒΠΆΠ΅ нСсут ΠΈΠ½Ρ„ΠΎΡ€ΠΌΠ°Ρ†ΠΈΡŽ ΠΎΠ± этих статичСских полях Π΄Π°ΠΆΠ΅ Π² Π²Π°ΠΊΡƒΡƒΠΌΠ΅.

Майк Π’.

(ΠΎΠΏΡƒΠ±Π»ΠΈΠΊΠΎΠ²Π°Π½ΠΎ 16.11.2017)

Follow-up on this answer

Related Questions

  • re-magnetizing magnets

  • magnets with signatures

  • heated ferromagnet re-magnetized

  • magnet questions

  • magnetic poles attract

  • Π’Π»ΠΈΡΡŽΡ‚ Π»ΠΈ Ρ‚Π΅ΠΏΠ»ΠΎ ΠΈ Ρ…ΠΎΠ»ΠΎΠ΄ Π½Π° ΠΌΠ°Π³Π½ΠΈΡ‚?0047

    ΠΌΠ°Π³Π½ΠΈΡ‚Ρ‹ ΠΈ гравитация

  • Π—Π΅ΠΌΠ½ΠΎΠΉ ΠΌΠ°Π³Π½Π΅Ρ‚ΠΈΠ·ΠΌ ΠΈ гравитация

ВсС Π΅Ρ‰Π΅ Π»ΡŽΠ±ΠΎΠΏΡ‹Ρ‚Π½ΠΎ?

Вопросы ΠΈ ΠΎΡ‚Π²Π΅Ρ‚Ρ‹ ΠΏΠΎ Expore Π² связанных катСгориях

  • ΠœΠ°Π³Π½ΠΈΡ‚Ρ‹

Частицы Π² ΠΌΠ°Π³Π½ΠΈΡ‚Π½Ρ‹Ρ… полях: Ρ‚ΠΈΠΏΡ‹

Π—Π½Π°Π΅Ρ‚Π΅ Π»ΠΈ Π²Ρ‹, Ρ‡Ρ‚ΠΎ ΠΌΠ°Π³Π½Π΅Ρ‚ΠΈΠ·ΠΌ обСспСчиваСт Ρ€Π°Π±ΠΎΡ‚Ρƒ ΠΌΠ°Π³Π½Π΅Ρ‚Ρ€ΠΎΠ½Π°, Π° Π·Π½Π°Ρ‡ΠΈΡ‚, ΠΈ ΠΌΠΈΠΊΡ€ΠΎΠ²ΠΎΠ»Π½ΠΎΠ²ΠΎΠΉ ΠΏΠ΅Ρ‡ΠΈ? Однако Π΄ΠΎ изобрСтСния ΠΌΠΈΠΊΡ€ΠΎΠ²ΠΎΠ»Π½ΠΎΠ²ΠΎΠΉ ΠΏΠ΅Ρ‡ΠΈ ΠΌΠ°Π³Π½Π΅Ρ‚Ρ€ΠΎΠ½Ρ‹ фактичСски использовались Π² Ρ€Π°Π΄ΠΈΠΎΠ»ΠΎΠΊΠ°Ρ†ΠΈΠΎΠ½Π½Ρ‹Ρ… систСмах Π²ΠΎ врСмя Π’Ρ‚ΠΎΡ€ΠΎΠΉ ΠΌΠΈΡ€ΠΎΠ²ΠΎΠΉ Π²ΠΎΠΉΠ½Ρ‹! ΠœΠΈΠΊΡ€ΠΎΠ²ΠΎΠ»Π½Ρ‹ β€” это элСктромагнитныС Π²ΠΎΠ»Π½Ρ‹ с Π΄Π»ΠΈΠ½ΠΎΠΉ Π²ΠΎΠ»Π½Ρ‹ ΠΎΡ‚ ΠΌΠΈΠ»Π»ΠΈΠΌΠ΅Ρ‚Ρ€Π° Π΄ΠΎ ΠΌΠ΅Ρ‚Ρ€Π°. Π˜Ρ… ΠΌΠΎΠΆΠ½ΠΎ искусствСнно ΡΠ³Π΅Π½Π΅Ρ€ΠΈΡ€ΠΎΠ²Π°Ρ‚ΡŒ с ΠΏΠΎΠΌΠΎΡ‰ΡŒΡŽ устройства ΠΏΠΎΠ΄ Π½Π°Π·Π²Π°Π½ΠΈΠ΅ΠΌ 9.0059 ΠΌΠ°Π³Π½Π΅Ρ‚Ρ€ΠΎΠ½ , ΠΊΠΎΡ‚ΠΎΡ€Ρ‹ΠΉ заставляСт быстро двиТущиСся элСктроны Π΄Π²ΠΈΠ³Π°Ρ‚ΡŒΡΡ ΠΏΠΎ ΠΊΡ€ΡƒΠ³ΠΎΠ²Ρ‹ΠΌ траСкториям. НСлСгко Π·Π°ΡΡ‚Π°Π²ΠΈΡ‚ΡŒ Ρ‡Ρ‚ΠΎ-Ρ‚ΠΎ ΡΡ‚ΠΎΠ»ΡŒ малСнькоС, ΠΊΠ°ΠΊ элСктроны, Π΄Π²ΠΈΠ³Π°Ρ‚ΡŒΡΡ ΠΏΠΎ ΠΊΡ€ΡƒΠ³Ρƒ, Π½ΠΎ это Π²ΠΎΠ·ΠΌΠΎΠΆΠ½ΠΎ, Ссли ΠΌΡ‹ ΠΏΡ€ΠΈΠ»ΠΎΠΆΠΈΠΌ ΠΊ Π½ΠΈΠΌ ΠΌΠ°Π³Π½ΠΈΡ‚Π½ΠΎΠ΅ ΠΏΠΎΠ»Π΅. Когда заряТСнныС частицы двиТутся Ρ‡Π΅Ρ€Π΅Π· ΠΌΠ°Π³Π½ΠΈΡ‚Π½ΠΎΠ΅ ΠΏΠΎΠ»Π΅, взаимодСйствиС создаСт ΠΌΠ°Π³Π½ΠΈΡ‚Π½ΡƒΡŽ силу Π½Π° частицС, которая заставляСт Π΅Π΅ ΠΎΡ‚ΠΊΠ»ΠΎΠ½ΡΡ‚ΡŒΡΡ. Π’ этой ΡΡ‚Π°Ρ‚ΡŒΠ΅ ΠΌΡ‹ исслСдуСм Ρ„ΠΈΠ·ΠΈΠΊΡƒ Π² Ρ€Π°Π±ΠΎΡ‚Π΅, Ρ‡Ρ‚ΠΎΠ±Ρ‹ ΡΠΎΠ·Π΄Π°Ρ‚ΡŒ силу Π½Π° Π·Π°Ρ€ΡΠΆΠ΅Π½Π½ΡƒΡŽ частицу, ΠΏΡ€ΠΎΠ°Π½Π°Π»ΠΈΠ·ΠΈΡ€ΡƒΠ΅ΠΌ, ΠΊΠ°ΠΊ это влияСт Π½Π° Π΄Π²ΠΈΠΆΠ΅Π½ΠΈΠ΅ частиц Π² ΠΌΠ°Π³Π½ΠΈΡ‚Π½ΠΎΠΌ ΠΏΠΎΠ»Π΅, ΠΈ, Π½Π°ΠΊΠΎΠ½Π΅Ρ†, рассмотрим ΠΏΠΎΠ²Π΅Π΄Π΅Π½ΠΈΠ΅ Π½Π΅ΠΊΠΎΡ‚ΠΎΡ€Ρ‹Ρ… Ρ€Π°Π·Π»ΠΈΡ‡Π½Ρ‹Ρ… Ρ‚ΠΈΠΏΠΎΠ² частиц.

Как ΠΌΡ‹ Π·Π½Π°Π΅ΠΌ, элСктричСство ΠΈ ΠΌΠ°Π³Π½Π΅Ρ‚ΠΈΠ·ΠΌ β€” это Ρ€Π°Π·Π΄Π΅Π»Ρ‹ Ρ„ΠΈΠ·ΠΈΠΊΠΈ, ΠΊΠΎΡ‚ΠΎΡ€Ρ‹Π΅ каТутся Ρ€Π°Π·Π½Ρ‹ΠΌΠΈ, Π½ΠΎ связаны Ρ‡Π΅Ρ€Π΅Π· ΠΏΠΎΠ»Π΅ элСктромагнСтизма . Π­Ρ‚ΠΎ Ρ€Π°Π·Π΄Π΅Π» Ρ„ΠΈΠ·ΠΈΠΊΠΈ, ΠΈΠ·ΡƒΡ‡Π°ΡŽΡ‰ΠΈΠΉ ΡΠ»Π΅ΠΊΡ‚Ρ€ΠΎΠΌΠ°Π³Π½ΠΈΡ‚Π½ΡƒΡŽ силу , которая являСтся ΠΎΠ΄Π½ΠΎΠΉ ΠΈΠ· Ρ‡Π΅Ρ‚Ρ‹Ρ€Π΅Ρ… Ρ„ΡƒΠ½Π΄Π°ΠΌΠ΅Π½Ρ‚Π°Π»ΡŒΠ½Ρ‹Ρ… сил ΠΏΡ€ΠΈΡ€ΠΎΠ΄Ρ‹ ΠΈ влияСт Π½Π° взаимодСйствиС ΠΌΠ΅ΠΆΠ΄Ρƒ элСктричСски заряТСнными частицами.

УравнСния МаксвСлла ΠΎΠΏΠΈΡΡ‹Π²Π°ΡŽΡ‚, ΠΊΠ°ΠΊ элСктричСскиС ΠΈ ΠΌΠ°Π³Π½ΠΈΡ‚Π½Ρ‹Π΅ поля Π³Π΅Π½Π΅Ρ€ΠΈΡ€ΡƒΡŽΡ‚ΡΡ вмСстС ΠΈ постоянно Π²Π·Π°ΠΈΠΌΠΎΠ΄Π΅ΠΉΡΡ‚Π²ΡƒΡŽΡ‚ ΠΈ Π²Π»ΠΈΡΡŽΡ‚ Π΄Ρ€ΡƒΠ³ Π½Π° Π΄Ρ€ΡƒΠ³Π°. Однако это Π²Ρ‹Ρ…ΠΎΠ΄ΠΈΡ‚ Π·Π° Ρ€Π°ΠΌΠΊΠΈ Ρ‚ΠΎΠ³ΠΎ, Ρ‡Ρ‚ΠΎ Π²Π°ΠΌ Π½ΡƒΠΆΠ½ΠΎ ΠΏΠΎΠ½ΠΈΠΌΠ°Ρ‚ΡŒ для экзамСнов GCSE!

ΠžΠΏΡ€Π΅Π΄Π΅Π»Π΅Π½ΠΈΡ ΠΌΠ°Π³Π½ΠΈΡ‚Π½ΠΎΠΉ силы ΠΈ ΠΌΠ°Π³Π½ΠΈΡ‚Π½ΠΎΠ³ΠΎ поля

Π­ΠΊΡΠΏΠ΅Ρ€ΠΈΠΌΠ΅Π½Ρ‚Π°Π»ΡŒΠ½ΠΎ Π±Ρ‹Π»ΠΎ ΠΎΠ±Π½Π°Ρ€ΡƒΠΆΠ΅Π½ΠΎ, Ρ‡Ρ‚ΠΎ ΠΊΠΎΠ³Π΄Π° элСктричСскиС заряды двиТутся (Ρ‚. Π΅. ΠΈΠΌΠ΅ΡŽΡ‚ Π½Π΅Π½ΡƒΠ»Π΅Π²ΡƒΡŽ ΡΠΊΠΎΡ€ΠΎΡΡ‚ΡŒ), Π² Π΄ΠΎΠΏΠΎΠ»Π½Π΅Π½ΠΈΠ΅ ΠΊ элСктричСской силС измСряСтся Π΅Ρ‰Π΅ ΠΎΠ΄ΠΈΠ½ Π²ΠΊΠ»Π°Π΄ Π² силу, с ΠΊΠΎΡ‚ΠΎΡ€ΠΎΠΉ ΡΡ‚Π°Π»ΠΊΠΈΠ²Π°ΡŽΡ‚ΡΡ заряТСнныС частицы. , Ρ‡Ρ‚ΠΎ Π±Ρ‹Π»ΠΎ ΡƒΠΆΠ΅ извСстно. Π­Ρ‚Π° загадочная сила Π±Ρ‹Π»Π° элСктромагнитной силой (Ρ‚Π°ΠΊΠΆΠ΅ Π½Π°Π·Ρ‹Π²Π°Π΅ΠΌΠΎΠΉ ΠΌΠ°Π³Π½ΠΈΡ‚Π½ΠΎΠΉ силой), Π²Ρ‹Π·Π²Π°Π½Π½ΠΎΠΉ Π΄Π²ΠΈΠΆΠ΅Π½ΠΈΠ΅ΠΌ носитСлСй заряда Ρ‡Π΅Ρ€Π΅Π· ΠΊΠ°ΠΊΠΎΠ΅-Ρ‚ΠΎ ΠΏΡ€ΠΈΠ»ΠΎΠΆΠ΅Π½Π½ΠΎΠ΅ ΠΈΠ·Π²Π½Π΅ ΠΌΠ°Π³Π½ΠΈΡ‚Π½ΠΎΠ΅ ΠΏΠΎΠ»Π΅.

ΠŸΡ€ΠΎΡΡ‚ΠΎΠΉ экспСримСнт, ΠΊΠΎΡ‚ΠΎΡ€Ρ‹ΠΉ Π²Ρ‹ ΠΌΠΎΠΆΠ΅Ρ‚Π΅ провСсти Π΄ΠΎΠΌΠ°, Ρ‡Ρ‚ΠΎΠ±Ρ‹ Π½Π°Π±Π»ΡŽΠ΄Π°Ρ‚ΡŒ Π·Π° дСйствиСм ΠΌΠ°Π³Π½ΠΈΡ‚Π½ΠΎΠΉ силы, ΠΈΡΠΏΠΎΠ»ΡŒΠ·ΡƒΠ΅Ρ‚ полоску алюминиСвой Ρ„ΠΎΠ»ΡŒΠ³ΠΈ, Π±Π°Ρ‚Π°Ρ€Π΅ΡŽ ΠΈ ΠΏΠΎΠ΄ΠΊΠΎΠ²ΠΎΠΎΠ±Ρ€Π°Π·Π½Ρ‹ΠΉ ΠΌΠ°Π³Π½ΠΈΡ‚. Π˜Π·Π½Π°Ρ‡Π°Π»ΡŒΠ½ΠΎ ΠΌΠ°Π³Π½ΠΈΡ‚ Π½Π΅ Π΄ΠΎΠ»ΠΆΠ΅Π½ Π²ΠΎΠ·Π΄Π΅ΠΉΡΡ‚Π²ΠΎΠ²Π°Ρ‚ΡŒ Π½Π° полоску Ρ„ΠΎΠ»ΡŒΠ³ΠΈ, Ρ‚Π°ΠΊ ΠΊΠ°ΠΊ ΠΎΠ½Π° сдСлана ΠΈΠ· алюминия. Однако, Ссли ΠΌΡ‹ создадим Ρ†Π΅ΠΏΡŒ, ΠΏΡ€ΠΈΠΊΡ€Π΅ΠΏΠΈΠ² ΠΊΠ°ΠΆΠ΄Ρ‹ΠΉ ΠΊΠΎΠ½Π΅Ρ† полоски Ρ„ΠΎΠ»ΡŒΠ³ΠΈ ΠΊ ΠΊΠ»Π΅ΠΌΠΌΠ°ΠΌ Π±Π°Ρ‚Π°Ρ€Π΅ΠΈ, Ρ‚Π΅ΠΏΠ΅Ρ€ΡŒ Ρ‡Π΅Ρ€Π΅Π· Ρ„ΠΎΠ»ΡŒΠ³Ρƒ Π±ΡƒΠ΄ΡƒΡ‚ Ρ‚Π΅Ρ‡ΡŒ элСктроны (заряТСнныС частицы). Если ΠΌΡ‹ Ρ‚Π΅ΠΏΠ΅Ρ€ΡŒ ΠΏΡ€ΠΈΠ±Π»ΠΈΠ·ΠΈΠΌ полоску ΠΊ ΠΌΠ°Π³Π½ΠΈΡ‚Π½ΠΎΠΌΡƒ полю, ΠΎΠ½Π° отклонится! Π­Ρ‚ΠΎ дСмонстрируСт, Ρ‡Ρ‚ΠΎ магнитная сила создаСтся Π΄Π²ΠΈΠΆΠ΅Π½ΠΈΠ΅ΠΌ заряТСнных частиц Ρ‡Π΅Ρ€Π΅Π· ΠΌΠ°Π³Π½ΠΈΡ‚Π½ΠΎΠ΅ ΠΏΠΎΠ»Π΅, ΠΏΠΎΡΠΊΠΎΠ»ΡŒΠΊΡƒ сила исчСзаСт, ΠΊΠΎΠ³Π΄Π° ΠΌΡ‹ Ρ€Π°Π·ΡŠΠ΅Π΄ΠΈΠ½ΡΠ΅ΠΌ Ρ†Π΅ΠΏΡŒ ΠΈΠ· Ρ„ΠΎΠ»ΡŒΠ³ΠΈ ΠΈ частицы ΠΏΠ΅Ρ€Π΅ΡΡ‚Π°ΡŽΡ‚ Π΄Π²ΠΈΠ³Π°Ρ‚ΡŒΡΡ.

A магнитная сила β€” это сила, ΠΊΠΎΡ‚ΠΎΡ€ΡƒΡŽ испытываСт заряТСнная частица (элСктрон, ΠΏΡ€ΠΎΡ‚ΠΎΠ½, ΠΈΠΎΠ½ ΠΈ Ρ‚. Π΄.), ΠΊΠΎΠ³Π΄Π° ΠΎΠ½Π° двиТСтся Ρ‡Π΅Ρ€Π΅Π· ΠΌΠ°Π³Π½ΠΈΡ‚Π½ΠΎΠ΅ ΠΏΠΎΠ»Π΅.

ΠœΠ°Π³Π½ΠΈΡ‚Π½Π°Ρ сила измСряСтся Π² Π½ΡŒΡŽΡ‚ΠΎΠ½Π°Ρ…, ΠΊΠ°ΠΊ ΠΈ любая другая сила. Π’Π°ΠΊΠΆΠ΅ Π²Π°ΠΆΠ½ΠΎ ΠΎΡ‚ΠΌΠ΅Ρ‚ΠΈΡ‚ΡŒ, Ρ‡Ρ‚ΠΎ заряТСнная частица Π΄ΠΎΠ»ΠΆΠ½Π° Π΄Π²ΠΈΠ³Π°Ρ‚ΡŒΡΡ Π½Π° ΠΎΡ‚Π½ΠΎΡΠΈΡ‚Π΅Π»ΡŒΠ½ΠΎ ΠΌΠ°Π³Π½ΠΈΡ‚Π½ΠΎΠ³ΠΎ поля Π½Π° , Ρ‡Ρ‚ΠΎΠ±Ρ‹ ΠΈΡΠΏΡ‹Ρ‚Π°Ρ‚ΡŒ Π½Π° сСбС ΠΌΠ°Π³Π½ΠΈΡ‚Π½ΡƒΡŽ силу.

Π’Π΅ΠΏΠ΅Ρ€ΡŒ ΠΌΡ‹ Π΄ΠΎΠ»ΠΆΠ½Ρ‹ Π²Ρ‹ΡΡΠ½ΠΈΡ‚ΡŒ, ΠΊΠ°ΠΊ ΠΌΠ°Π³Π½ΠΈΡ‚Ρ‹ ΡΠΎΠ·Π΄Π°ΡŽΡ‚ эту силу, ΠΈ для этого Π½Π°ΠΌ Π½ΡƒΠΆΠ½ΠΎ ΠΎΠ±ΡΡƒΠ΄ΠΈΡ‚ΡŒ ΠΌΠ°Π³Π½ΠΈΡ‚Π½ΠΎΠ΅ ΠΏΠΎΠ»Π΅. ΠžΠΏΡ€Π΅Π΄Π΅Π»Π΅Π½ΠΈΠ΅ ΠΌΠ°Π³Π½ΠΈΡ‚Π½ΠΎΠ΅ ΠΏΠΎΠ»Π΅ выглядит ΡΠ»Π΅Π΄ΡƒΡŽΡ‰ΠΈΠΌ ΠΎΠ±Ρ€Π°Π·ΠΎΠΌ.

ΠœΠ°Π³Π½ΠΈΡ‚Π½ΠΎΠ΅ ΠΏΠΎΠ»Π΅ прСдставляСт собой ΠΎΠ±Π»Π°ΡΡ‚ΡŒ Π² пространствС, Π³Π΄Π΅ двиТущийся заряд ΠΈΠ»ΠΈ постоянный ΠΌΠ°Π³Π½ΠΈΡ‚ ΠΎΡ‰ΡƒΡ‰Π°ΡŽΡ‚ силу .

ΠœΠ°Π³Π½ΠΈΡ‚Π½ΠΎΠ΅ ΠΏΠΎΠ»Π΅ присутствуСт Π² любой Ρ‚ΠΎΡ‡ΠΊΠ΅ пространства, Π³Π΄Π΅ Π½Π° Π΄Π²ΠΈΠΆΡƒΡ‰ΡƒΡŽΡΡ Π·Π°Ρ€ΡΠΆΠ΅Π½Π½ΡƒΡŽ частицу дСйствуСт сила. Π‘ΠΈΠ»Π° ΠΌΠ°Π³Π½ΠΈΡ‚Π½ΠΎΠ³ΠΎ поля ΠΎΠ±Ρ‹Ρ‡Π½ΠΎ называСтся ΠΏΠ»ΠΎΡ‚Π½ΠΎΡΡ‚ΡŒΡŽ ΠΌΠ°Π³Π½ΠΈΡ‚Π½ΠΎΠ³ΠΎ ΠΏΠΎΡ‚ΠΎΠΊΠ° ΠΈΠ»ΠΈ Π½Π°ΠΏΡ€ΡΠΆΠ΅Π½Π½ΠΎΡΡ‚ΡŒΡŽ ΠΌΠ°Π³Π½ΠΈΡ‚Π½ΠΎΠ³ΠΎ поля ΠΈ обозначаСтся символом Π•Π΄ΠΈΠ½ΠΈΡ†Π΅ΠΉ измСрСния ΠΌΠ°Π³Π½ΠΈΡ‚Π½ΠΎΠ³ΠΎ поля являСтся ВСсла , Ρ‡Ρ‚ΠΎ эквивалСнтно Π½ΡŒΡŽΡ‚ΠΎΠ½Π°ΠΌ Π½Π° Π°ΠΌΠΏΠ΅Ρ€-ΠΌΠ΅Ρ‚Ρ€ ,.

ΠœΠ°Π³Π½ΠΈΡ‚Π½ΠΎΠ΅ ΠΏΠΎΠ»Π΅ Π½Π΅ всСгда ΠΈΠΌΠ΅Π΅Ρ‚ ΠΏΠΎΡΡ‚ΠΎΡΠ½Π½ΡƒΡŽ силу ΠΈΠ»ΠΈ ΠΏΠ»ΠΎΡ‚Π½ΠΎΡΡ‚ΡŒ ΠΏΠΎΡ‚ΠΎΠΊΠ° . Поля ΠΎΠ±Ρ‹Ρ‡Π½ΠΎ ΠΏΡ€Π΅Π΄ΡΡ‚Π°Π²Π»ΡΡŽΡ‚ΡΡ линиями ΠΌΠ°Π³Π½ΠΈΡ‚Π½ΠΎΠ³ΠΎ поля, ΠΊΠΎΡ‚ΠΎΡ€Ρ‹Π΅ проходят ΠΎΡ‚ сСвСрного ΠΊ ΡŽΠΆΠ½ΠΎΠΌΡƒ ΠΏΠΎΠ»ΡŽΡΡƒ ΠΌΠ°Π³Π½ΠΈΡ‚Π°, ΠΏΡ€ΠΈΡ‡Π΅ΠΌ Π½Π°ΠΏΡ€ΡΠΆΠ΅Π½Π½ΠΎΡΡ‚ΡŒ поля максимальна Ρ‚Π°ΠΌ, Π³Π΄Π΅ Π»ΠΈΠ½ΠΈΠΈ Π±Π»ΠΈΠΆΠ΅ всСго Π΄Ρ€ΡƒΠ³ ΠΊ Π΄Ρ€ΡƒΠ³Ρƒ. Как ΠΏΡ€Π°Π²ΠΈΠ»ΠΎ, это ΠΏΡ€ΠΈΠ²ΠΎΠ΄ΠΈΡ‚ ΠΊ Ρ‚ΠΎΠΌΡƒ, Ρ‡Ρ‚ΠΎ поля становятся сильнСС Π±Π»ΠΈΠΆΠ΅ ΠΊ полюсам ΠΌΠ°Π³Π½ΠΈΡ‚Π° ΠΈ ΠΎΡΠ»Π°Π±Π΅Π²Π°ΡŽΡ‚ с ΡƒΠ²Π΅Π»ΠΈΡ‡Π΅Π½ΠΈΠ΅ΠΌ расстояния.

ЗаряТСнныС частицы Π² ΠΌΠ°Π³Π½ΠΈΡ‚Π½ΠΎΠΌ ΠΏΠΎΠ»Π΅

Как ΠΌΡ‹ ΡƒΠ·Π½Π°Π»ΠΈ ΠΈΠ· Ρ‚Π΅ΠΌΡ‹ ΠΎΠ± элСктричСствС, ΠΏΠΎΡ‚ΠΎΠΊ ΠΏΠΎΠ»ΠΎΠΆΠΈΡ‚Π΅Π»ΡŒΠ½Ρ‹Ρ… элСктричСских зарядов прСдставляСт собой ΠΎΠ±Ρ‹Ρ‡Π½Ρ‹ΠΉ элСктричСский Ρ‚ΠΎΠΊ. Π‘Π»Π΅Π΄ΠΎΠ²Π°Ρ‚Π΅Π»ΡŒΠ½ΠΎ, элСктричСскиС Ρ‚ΠΎΠΊΠΈ Π±ΡƒΠ΄ΡƒΡ‚ ΠΈΡΠΏΡ‹Ρ‚Ρ‹Π²Π°Ρ‚ΡŒ силу Π² ΠΌΠ°Π³Π½ΠΈΡ‚Π½ΠΎΠΌ ΠΏΠΎΠ»Π΅. Если ΠΌΡ‹ ΠΏΠΎΠ΄ΡƒΠΌΠ°Π΅ΠΌ ΠΎΠ± элСктронах, двиТущихся ΠΏΠΎ ΠΊΡ€ΡƒΠ³Ρƒ Π² ΠΌΠ°Π³Π½Π΅Ρ‚Ρ€ΠΎΠ½Π΅, ΠΊΠ°ΠΊ ΠΌΡ‹ ΡƒΠΏΠΎΠΌΠΈΠ½Π°Π»ΠΈ Π² Π½Π°Ρ‡Π°Π»Π΅ ΡΡ‚Π°Ρ‚ΡŒΠΈ, ΠΊΠ°ΠΊ ΠΌΡ‹ ΠΌΠΎΠΆΠ΅ΠΌ Π±Ρ‹Ρ‚ΡŒ ΡƒΠ²Π΅Ρ€Π΅Π½Ρ‹, Ρ‡Ρ‚ΠΎ элСктроны Π±ΡƒΠ΄ΡƒΡ‚ Π΄Π²ΠΈΠ³Π°Ρ‚ΡŒΡΡ ΠΈΠΌΠ΅Π½Π½ΠΎ Ρ‚Π°ΠΊΠΈΠΌ ΠΎΠ±Ρ€Π°Π·ΠΎΠΌ, ΠΏΠΎΡΠΊΠΎΠ»ΡŒΠΊΡƒ ΠΎΠ½ΠΈ Π½Π΅ содСрТатся Π²Π½ΡƒΡ‚Ρ€ΠΈ ΠΏΡ€ΠΎΠ²ΠΎΠ΄Π°? ΠžΠΊΠ°Π·Ρ‹Π²Π°Π΅Ρ‚ΡΡ, сущСствуСт связь ΠΌΠ΅ΠΆΠ΄Ρƒ направлСниями ΠΌΠ°Π³Π½ΠΈΡ‚Π½ΠΎΠ΅ ΠΏΠΎΠ»Π΅ , магнитная сила ΠΈ Ρ‚ΠΎΠΊ . Если ΠΌΡ‹ Π·Π½Π°Π΅ΠΌ Π½Π°ΠΏΡ€Π°Π²Π»Π΅Π½ΠΈΠ΅ Π»ΡŽΠ±Ρ‹Ρ… Π΄Π²ΡƒΡ… ΠΈΠ· этих Π²Π΅Π»ΠΈΡ‡ΠΈΠ½, ΠΌΡ‹ ΠΌΠΎΠΆΠ΅ΠΌ Π½Π°ΠΉΡ‚ΠΈ Π½Π°ΠΏΡ€Π°Π²Π»Π΅Π½ΠΈΠ΅ Ρ‚Ρ€Π΅Ρ‚ΡŒΠ΅ΠΉ.

ΠŸΡ€Π°Π²ΠΈΠ»ΠΎ Π»Π΅Π²ΠΎΠΉ Ρ€ΡƒΠΊΠΈ

Π§Ρ‚ΠΎΠ±Ρ‹ ΠΎΠΏΡ€Π΅Π΄Π΅Π»ΠΈΡ‚ΡŒ Π½Π°ΠΏΡ€Π°Π²Π»Π΅Π½ΠΈΠ΅ силы, ΠΊΠΎΡ‚ΠΎΡ€ΡƒΡŽ Π±ΡƒΠ΄Π΅Ρ‚ ΠΎΡ‰ΡƒΡ‰Π°Ρ‚ΡŒ двиТущийся заряд, ΠΊΠΎΠ³Π΄Π° ΠΎΠ½ Π²Ρ…ΠΎΠ΄ΠΈΡ‚ Π² ΠΌΠ°Π³Π½ΠΈΡ‚Π½ΠΎΠ΅ ΠΏΠΎΠ»Π΅, ΠΈΡΠΏΠΎΠ»ΡŒΠ·ΡƒΠ΅Ρ‚ΡΡ ΠΏΡ€Π°Π²ΠΈΠ»ΠΎ Π»Π΅Π²ΠΎΠΉ Ρ€ΡƒΠΊΠΈ Π€Π»Π΅ΠΌΠΈΠ½Π³Π° . ΠŸΡ€Π°Π²ΠΈΠ»ΠΎ Π»Π΅Π²ΠΎΠΉ Ρ€ΡƒΠΊΠΈ гласит, Ρ‡Ρ‚ΠΎ Π²Ρ‹ Π΄ΠΎΠ»ΠΆΠ½Ρ‹ Ρ€Π°ΡΡΡ‚Π°Π²ΠΈΡ‚ΡŒ большой, ΡƒΠΊΠ°Π·Π°Ρ‚Π΅Π»ΡŒΠ½Ρ‹ΠΉ ΠΈ срСдний ΠΏΠ°Π»ΡŒΡ†Ρ‹ Ρ‚Π°ΠΊ, Ρ‡Ρ‚ΠΎΠ±Ρ‹ ΠΎΠ½ΠΈ Π½Π°Ρ…ΠΎΠ΄ΠΈΠ»ΠΈΡΡŒ ΠΏΠΎΠ΄ прямым ΡƒΠ³Π»ΠΎΠΌ Π΄Ρ€ΡƒΠ³ ΠΊ Π΄Ρ€ΡƒΠ³Ρƒ (ΠΊΠ°ΠΊ ΠΏΠΎΠΊΠ°Π·Π°Π½ΠΎ Π½Π° Π΄ΠΈΠ°Π³Ρ€Π°ΠΌΠΌΠ΅ Π½ΠΈΠΆΠ΅).

  1. ΠΠ°ΠΏΡ€Π°Π²ΡŒΡ‚Π΅ ΡƒΠΊΠ°Π·Π°Ρ‚Π΅Π»ΡŒΠ½Ρ‹ΠΉ ΠΏΠ°Π»Π΅Ρ† Π² Π½Π°ΠΏΡ€Π°Π²Π»Π΅Π½ΠΈΠΈ ΠΌΠ°Π³Π½ΠΈΡ‚Π½ΠΎΠ³ΠΎ поля, Ρ‚ΠΎ Π΅ΡΡ‚ΡŒ с сСвСрного полюса Π½Π° ΡŽΠΆΠ½Ρ‹ΠΉ полюс.
  2. Π’Π°Ρˆ срСдний ΠΏΠ°Π»Π΅Ρ† Π΄Π°Π΅Ρ‚ Π½Π°ΠΏΡ€Π°Π²Π»Π΅Π½ΠΈΠ΅ условного Ρ‚ΠΎΠΊΠ°(Π΄Π²ΠΈΠΆΠ΅Π½ΠΈΠ΅ ΠΏΠΎΠ»ΠΎΠΆΠΈΡ‚Π΅Π»ΡŒΠ½ΠΎΠ³ΠΎ заряда).
  3. Π’Π°Ρˆ большой ΠΏΠ°Π»Π΅Ρ† ΡƒΠΊΠ°ΠΆΠ΅Ρ‚ Π½Π°ΠΏΡ€Π°Π²Π»Π΅Π½ΠΈΠ΅ силы, Π΄Π΅ΠΉΡΡ‚Π²ΡƒΡŽΡ‰Π΅ΠΉ Π½Π° частицу.

ΠŸΡ€Π°Π²ΠΈΠ»ΠΎ Π»Π΅Π²ΠΎΠΉ Ρ€ΡƒΠΊΠΈ ΠΈΡΠΏΠΎΠ»ΡŒΠ·ΡƒΠ΅Ρ‚ΡΡ для опрСдСлСния направлСния силы, Π΄Π΅ΠΉΡΡ‚Π²ΡƒΡŽΡ‰Π΅ΠΉ Π½Π° Π·Π°Ρ€ΡΠΆΠ΅Π½Π½ΡƒΡŽ частицу, ΠΊΠΎΠ³Π΄Π° ΠΎΠ½Π° двиТСтся Ρ‡Π΅Ρ€Π΅Π· ΠΌΠ°Π³Π½ΠΈΡ‚Π½ΠΎΠ΅ ΠΏΠΎΠ»Π΅ ΠΏΠΎΠ΄ прямым ΡƒΠ³Π»ΠΎΠΌ ΠΊ ​​нСму. Викисклад CC BY-SA 4.0

Π‘ΠΈΠ»Π°, Π΄Π΅ΠΉΡΡ‚Π²ΡƒΡŽΡ‰Π°Ρ Π½Π° заряТСнныС частицы Π² ΠΌΠ°Π³Π½ΠΈΡ‚Π½ΠΎΠΌ ΠΏΠΎΠ»Π΅

ΠžΠ±Ρ‰ΠΈΠΉ Π·Π°ΠΊΠΎΠ½, ΠΎΠΏΡ€Π΅Π΄Π΅Π»ΡΡŽΡ‰ΠΈΠΉ ΠΏΠΎΠ²Π΅Π΄Π΅Π½ΠΈΠ΅ элСктричСского заряда Π² присутствии элСктромагнитного поля, извСстСн ΠΊΠ°ΠΊ сила Π›ΠΎΡ€Π΅Π½Ρ†Π° . ΠžΠ±Ρ‰Π΅Π΅ Π²Ρ‹Ρ€Π°ΠΆΠ΅Π½ΠΈΠ΅ Ρ‚Π°ΠΊΠΆΠ΅ Π²ΠΊΠ»ΡŽΡ‡Π°Π΅Ρ‚ влияниС внСшнСго элСктричСского поля, Π½ΠΎ здСсь ΠΌΡ‹ ограничимся ситуациями, ΠΊΠΎΠ³Π΄Π° присутствуСт Ρ‚ΠΎΠ»ΡŒΠΊΠΎ ΠΌΠ°Π³Π½ΠΈΡ‚Π½ΠΎΠ΅ ΠΏΠΎΠ»Π΅.

Π’Ρ‹Ρ€Π°ΠΆΠ΅Π½ΠΈΠ΅ для силы, Π΄Π΅ΠΉΡΡ‚Π²ΡƒΡŽΡ‰Π΅ΠΉ Π½Π° Π·Π°Ρ€ΡΠΆΠ΅Π½Π½ΡƒΡŽ частицу, Π΄Π²ΠΈΠΆΡƒΡ‰ΡƒΡŽΡΡ пСрпСндикулярно ΠΌΠ°Π³Π½ΠΈΡ‚Π½ΠΎΠΌΡƒ полю, ΠΈΠΌΠ΅Π΅Ρ‚ Π²ΠΈΠ΄:

Π³Π΄Π΅ β€” заряд частицы, β€” Π²Π΅Π»ΠΈΡ‡ΠΈΠ½Π° Π΅Π΅ скорости (скорости), Π° β€” Π½Π°ΠΏΡ€ΡΠΆΠ΅Π½Π½ΠΎΡΡ‚ΡŒ ΠΌΠ°Π³Π½ΠΈΡ‚Π½ΠΎΠ³ΠΎ поля.

Π­Ρ‚ΠΎ ΡƒΡ€Π°Π²Π½Π΅Π½ΠΈΠ΅ ясно ΡƒΠΊΠ°Π·Ρ‹Π²Π°Π΅Ρ‚ Π½Π° Ρ‚ΠΎ, Ρ‡Ρ‚ΠΎ частица Π΄ΠΎΠ»ΠΆΠ½Π° Π΄Π²ΠΈΠ³Π°Ρ‚ΡŒΡΡ, Ρ‡Ρ‚ΠΎΠ±Ρ‹ магнитная сила Π²ΠΎΠΎΠ±Ρ‰Π΅ ΠΎΡ‰ΡƒΡ‰Π°Π»Π°ΡΡŒ. Π£ нас Π΅ΡΡ‚ΡŒ ΠΎΡ‚Π½ΠΎΡˆΠ΅Π½ΠΈΠ΅, Π² ΠΊΠΎΡ‚ΠΎΡ€ΠΎΠΌ Π½Π°ΠΏΡ€Π°Π²Π»Π΅Π½ΠΈΠ΅ двиТСния, силы ΠΈ поля находятся ΠΏΠΎΠ΄ прямым ΡƒΠ³Π»ΠΎΠΌ Π΄Ρ€ΡƒΠ³ ΠΊ Π΄Ρ€ΡƒΠ³Ρƒ ΠΈ ΠΌΠΎΠ³ΡƒΡ‚ Π±Ρ‹Ρ‚ΡŒ ΠΎΠΏΡ€Π΅Π΄Π΅Π»Π΅Π½Ρ‹ с ΠΏΠΎΠΌΠΎΡ‰ΡŒΡŽ ΠΏΡ€Π°Π²ΠΈΠ»Π° Π»Π΅Π²ΠΎΠΉ Ρ€ΡƒΠΊΠΈ.

Если заряТСнная частица двиТСтся Ρ‡Π΅Ρ€Π΅Π· ΠΌΠ°Π³Π½ΠΈΡ‚Π½ΠΎΠ΅ ΠΏΠΎΠ»Π΅ нСпСрпСндикулярно, Π½Π° Π½Π΅Π΅ всС Ρ€Π°Π²Π½ΠΎ дСйствуСт сила, ΠΎΠ΄Π½Π°ΠΊΠΎ ΠΎΠ½Π° Π±ΡƒΠ΄Π΅Ρ‚ мСньшС максимальной силы, Π΄Π΅ΠΉΡΡ‚Π²ΡƒΡŽΡ‰Π΅ΠΉ, ΠΊΠΎΠ³Π΄Π° Π΄Π²Π° Π²Π΅ΠΊΡ‚ΠΎΡ€Π° находятся ΠΏΠΎΠ΄ прямым ΡƒΠ³Π»ΠΎΠΌ. ΠŸΡ€ΠΈ Π΄ΠΎΠ±Π°Π²Π»Π΅Π½ΠΈΠΈ Ρ‡Π»Π΅Π½Π° ΠΊ ΡƒΡ€Π°Π²Π½Π΅Π½ΠΈΡŽ, Π³Π΄Π΅ ΡƒΠ³ΠΎΠ» ΠΌΠ΅ΠΆΠ΄Ρƒ ΠΌΠ°Π³Π½ΠΈΡ‚Π½Ρ‹ΠΌ ΠΏΠΎΠ»Π΅ΠΌ ΠΈ Π²Π΅ΠΊΡ‚ΠΎΡ€ΠΎΠΌ скорости частицы, ΡƒΡ€Π°Π²Π½Π΅Π½ΠΈΠ΅ ΠΌΠ°Π³Π½ΠΈΡ‚Π½ΠΎΠΉ силы ΠΏΡ€ΠΈΠ½ΠΈΠΌΠ°Π΅Ρ‚ Π²ΠΈΠ΄:

ΠšΡ€ΡƒΠ³ΠΎΠ²ΠΎΠ΅ Π΄Π²ΠΈΠΆΠ΅Π½ΠΈΠ΅ заряТСнных частиц Π² ΠΌΠ°Π³Π½ΠΈΡ‚Π½ΠΎΠΌ ΠΏΠΎΠ»Π΅

ΠœΡ‹ Π²ΠΈΠ΄Π΅Π»ΠΈ, Ρ‡Ρ‚ΠΎ Π½Π° Π·Π°Ρ€ΡΠΆΠ΅Π½Π½ΡƒΡŽ частицу дСйствуСт сила, пСрпСндикулярная Π½Π°ΠΏΡ€Π°Π²Π»Π΅Π½ΠΈΡŽ Π΅Π΅ двиТСния, ΠΊΠΎΠ³Π΄Π° ΠΎΠ½Π° Π²Ρ…ΠΎΠ΄ΠΈΡ‚ Π² ΠΌΠ°Π³Π½ΠΈΡ‚Π½ΠΎΠ΅ ΠΏΠΎΠ»Π΅ ΠΏΠΎΠ΄ прямым ΡƒΠ³Π»ΠΎΠΌ ΠΊ β€‹β€‹ΠΏΠΎΠ»ΡŽ. Π­Ρ‚ΠΎ ΠΎΠ·Π½Π°Ρ‡Π°Π΅Ρ‚, Ρ‡Ρ‚ΠΎ Π½Π°ΠΏΡ€Π°Π²Π»Π΅Π½ΠΈΠ΅ двиТСния частицы Π±ΡƒΠ΄Π΅Ρ‚ ΠΌΠ΅Π½ΡΡ‚ΡŒΡΡ ΠΏΠΎ ΠΌΠ΅Ρ€Π΅ Ρ‚ΠΎΠ³ΠΎ, ΠΊΠ°ΠΊ магнитная сила отклоняСт частицу. ΠŸΠΎΡΠΊΠΎΠ»ΡŒΠΊΡƒ заряТСнная частица Ρ‚Π΅ΠΏΠ΅Ρ€ΡŒ ΠΈΠ·ΠΌΠ΅Π½ΠΈΠ»Π° Π½Π°ΠΏΡ€Π°Π²Π»Π΅Π½ΠΈΠ΅, это прСдставляСт собой ΠΈΠ·ΠΌΠ΅Π½Π΅Π½ΠΈΠ΅ направлСния Ρ‚ΠΎΠΊΠ°. Если ΠΏΠΎΠ»Π΅ остаСтся постоянным, Π° Ρ‚ΠΎΠΊ мСняСт Π½Π°ΠΏΡ€Π°Π²Π»Π΅Π½ΠΈΠ΅, Ρ‚ΠΎ создаваСмая магнитная сила Ρ‚Π°ΠΊΠΆΠ΅ Π΄ΠΎΠ»ΠΆΠ½Π° постоянно ΠΌΠ΅Π½ΡΡ‚ΡŒ Π½Π°ΠΏΡ€Π°Π²Π»Π΅Π½ΠΈΠ΅! Π­Ρ‚ΠΎ ΠΏΡ€ΠΈΠ²ΠΎΠ΄ΠΈΡ‚ ΠΊ Π·Π°ΠΏΡƒΡ‚Π°Π½Π½ΠΎΠΌΡƒ ΡΡ†Π΅Π½Π°Ρ€ΠΈΡŽ, Π½ΠΎ ΠΎΠ±ΡŠΡΡΠ½ΡΠ΅Ρ‚, ΠΊΠ°ΠΊ заряТСнная частица Π² ΠΌΠ°Π³Π½ΠΈΡ‚Π½ΠΎΠΌ ΠΏΠΎΠ»Π΅ ΠΌΠΎΠΆΠ΅Ρ‚ Π΄Π²ΠΈΠ³Π°Ρ‚ΡŒΡΡ ΠΏΠΎ ΠΊΡ€ΡƒΠ³ΠΎΠ²ΠΎΠΉ ΠΏΡƒΡ‚ΡŒ .

НС Π±ΡƒΠ΄Π΅ΠΌ слишком Π·Π°ΠΏΡƒΡ‚Ρ‹Π²Π°Ρ‚ΡŒΡΡ ΠΈ попытаСмся ΠΏΠΎΠ½ΡΡ‚ΡŒ, Ρ‡Ρ‚ΠΎ происходит, Π½Π° ΠΏΡ€ΠΈΠΌΠ΅Ρ€Π΅ рисунка Π½ΠΈΠΆΠ΅.

  1. ΠŸΡ€Π΅Π΄ΡΡ‚Π°Π²ΡŒΡ‚Π΅ сСбС элСктрон, двиТущийся с постоянной ΡΠΊΠΎΡ€ΠΎΡΡ‚ΡŒΡŽ Π² ΠΎΠ΄Π½ΠΎΡ€ΠΎΠ΄Π½ΠΎΠΌ ΠΌΠ°Π³Π½ΠΈΡ‚Π½ΠΎΠΌ ΠΏΠΎΠ»Π΅ ΠΈ ΡƒΠΊΠ°Π·Ρ‹Π²Π°ΡŽΡ‰ΠΈΠΉ Π½Π° страницу (обозначаСтся этим символом βŠ—) .
  2. Когда элСктрон двиТСтся Π² ΠΏΠΎΠ»Π΅, Π½Π° Π½Π΅Π³ΠΎ дСйствуСт сила, Π΄Π΅ΠΉΡΡ‚Π²ΡƒΡŽΡ‰Π°Ρ ΠΏΠΎΠ΄ прямым ΡƒΠ³Π»ΠΎΠΌ ΠΊ ​​скорости. ВраСктория элСктрона слСгка искривляСтся, ΠΈ Ρ‚Π΅ΠΏΠ΅Ρ€ΡŒ Π΅Π³ΠΎ ΡΠΊΠΎΡ€ΠΎΡΡ‚ΡŒ ΠΈΠΌΠ΅Π΅Ρ‚ Π΄Ρ€ΡƒΠ³ΠΎΠ΅ Π½Π°ΠΏΡ€Π°Π²Π»Π΅Π½ΠΈΠ΅. Однако ΡΠΊΠΎΡ€ΠΎΡΡ‚ΡŒ ΠΏΠΎ-ΠΏΡ€Π΅ΠΆΠ½Π΅ΠΌΡƒ находится ΠΏΠΎΠ΄ прямым ΡƒΠ³Π»ΠΎΠΌ ΠΊ β€‹β€‹Π½Π°ΠΏΡ€Π°Π²Π»Π΅Π½ΠΈΡŽ ΠΌΠ°Π³Π½ΠΈΡ‚Π½ΠΎΠ³ΠΎ поля.
  3. Π‘ΠΈΠ»Π° ΠΈ ΡΠΊΠΎΡ€ΠΎΡΡ‚ΡŒ ΠΏΠΎ-ΠΏΡ€Π΅ΠΆΠ½Π΅ΠΌΡƒ находятся ΠΏΠΎΠ΄ прямым ΡƒΠ³Π»ΠΎΠΌ Π΄Ρ€ΡƒΠ³ ΠΊ Π΄Ρ€ΡƒΠ³Ρƒ ΠΈ ΠΎΡΡ‚Π°ΡŽΡ‚ΡΡ Π² ΠΎΠ΄Π½ΠΎΠΉ плоскости. На самом Π΄Π΅Π»Π΅ магнитная сила Π½Π°ΠΏΡ€Π°Π²Π»Π΅Π½Π° ​​к Ρ†Π΅Π½Ρ‚Ρ€Ρƒ ΠΊΡ€ΡƒΠ³ΠΎΠ²ΠΎΠ³ΠΎ ΠΏΡƒΡ‚ΠΈ, ΠΏΠΎ ΠΊΠΎΡ‚ΠΎΡ€ΠΎΠΌΡƒ двиТСтся элСктрон.
  4. Π­Ρ‚ΠΎ продолТаСтся, сила ΠΈ ΡΠΊΠΎΡ€ΠΎΡΡ‚ΡŒ всСгда пСрпСндикулярны. Если ΠΌΡ‹ ΠΏΠΎΠΌΠ½ΠΈΠΌ ΠΈΠ· двиТСния ΠΏΠΎ ΠΊΡ€ΡƒΠ³Ρƒ, Ρƒ нас Π΅ΡΡ‚ΡŒ ΠΊΡ€ΡƒΠ³ΠΎΠ²Ρ‹Ρ… Π΄Π²ΠΈΠΆΠ΅Π½ΠΈΠΉ , Ссли эти Π΄Π²Π΅ Π²Π΅Π»ΠΈΡ‡ΠΈΠ½Ρ‹ пСрпСндикулярны с постоянными Π²Π΅Π»ΠΈΡ‡ΠΈΠ½Π°ΠΌΠΈ.

На этом ΠΈΠ·ΠΎΠ±Ρ€Π°ΠΆΠ΅Π½ΠΈΠΈ ΠΏΠΎΠΊΠ°Π·Π°Π½Π° ΠΎΡ‚Ρ€ΠΈΡ†Π°Ρ‚Π΅Π»ΡŒΠ½ΠΎ заряТСнная частица, такая ΠΊΠ°ΠΊ элСктрон, двиТущаяся Π² ΠΌΠ°Π³Π½ΠΈΡ‚Π½ΠΎΠΌ ΠΏΠΎΠ»Π΅. Π‘ΠΈΠ»Π°, Π΄Π΅ΠΉΡΡ‚Π²ΡƒΡŽΡ‰Π°Ρ Π½Π° частицу, постоянно мСняСт Π½Π°ΠΏΡ€Π°Π²Π»Π΅Π½ΠΈΠ΅, Π½ΠΎ постоянна ΠΏΠΎ Π²Π΅Π»ΠΈΡ‡ΠΈΠ½Π΅, ΠΊΠ°ΠΊ ΠΈ ΡΠΊΠΎΡ€ΠΎΡΡ‚ΡŒ. ΠŸΠΎΡΠΊΠΎΠ»ΡŒΠΊΡƒ Π΄Π²Π΅ Π²Π΅Π»ΠΈΡ‡ΠΈΠ½Ρ‹ всСгда пСрпСндикулярны, элСктрон двиТСтся ΠΏΠΎ окруТности, Wikimedia Commons CC 4.0

ΠžΠΏΡ€Π΅Π΄Π΅Π»ΡΠ΅ΠΌ Π½Π°ΠΏΡ€Π°Π²Π»Π΅Π½ΠΈΠ΅ силы ΠΏΠΎ ΠΏΡ€Π°Π²ΠΈΠ»Ρƒ Π»Π΅Π²ΠΎΠΉ Ρ€ΡƒΠΊΠΈ. Π­Π»Π΅ΠΊΡ‚Ρ€ΠΎΠ½Ρ‹ заряТСны ΠΎΡ‚Ρ€ΠΈΡ†Π°Ρ‚Π΅Π»ΡŒΠ½ΠΎ, Π° это ΠΎΠ·Π½Π°Ρ‡Π°Π΅Ρ‚, Ρ‡Ρ‚ΠΎ Ρ‚ΠΎΠΊ ΠΏΡ€ΠΎΡ‚ΠΈΠ²ΠΎΠΏΠΎΠ»ΠΎΠΆΠ΅Π½ Π½Π°ΠΏΡ€Π°Π²Π»Π΅Π½ΠΈΡŽ двиТСния элСктрона.

РассмотрСниС полоТСния элСктрона Π² ΠΌΠ°Π³Π½ΠΈΡ‚Π½ΠΎΠΌ ΠΏΠΎΠ»Π΅, ΠΏΠΎΠ΄ΠΎΠ±Π½ΠΎΠ³ΠΎ ΠΈΠ·ΠΎΠ±Ρ€Π°ΠΆΠ΅Π½Π½ΠΎΠΌΡƒ Π½Π° Π΄ΠΈΠ°Π³Ρ€Π°ΠΌΠΌΠ΅ Π½ΠΈΠΆΠ΅, Π½Π΅ позволяСт частицам Π΄Π²ΠΈΠ³Π°Ρ‚ΡŒΡΡ ΠΏΠΎ ΠΊΡ€ΡƒΠ³ΠΎΠ²ΠΎΠΉ Ρ‚Ρ€Π°Π΅ΠΊΡ‚ΠΎΡ€ΠΈΠΈ. Π­Ρ‚ΠΎ происходит ΠΏΠΎ Π΄Π²ΡƒΠΌ ΠΏΡ€ΠΈΡ‡ΠΈΠ½Π°ΠΌ:

  1. Частица сначала Π½Π°Ρ‡ΠΈΠ½Π°Π΅Ρ‚ Π΄Π²ΠΈΠ³Π°Ρ‚ΡŒΡΡ ΠΏΠΎ ΠΊΡ€ΡƒΠ³ΠΎΠ²ΠΎΠΉ Ρ‚Ρ€Π°Π΅ΠΊΡ‚ΠΎΡ€ΠΈΠΈ, Π½ΠΎ ΠΏΠΎΠΊΠΈΠ΄Π°Π΅Ρ‚ ниТнюю Ρ‡Π°ΡΡ‚ΡŒ плоскости, ΠΏΠΎΡ‚ΠΎΠΌΡƒ Ρ‡Ρ‚ΠΎ ΠΌΠ°Π³Π½ΠΈΡ‚Π½ΠΎΠ΅ ΠΏΠΎΠ»Π΅ заканчиваСтся. Π­Ρ‚ΠΎ ΠΌΠΎΠΆΠ½ΠΎ Ρ€Π΅ΡˆΠΈΡ‚ΡŒ, Ρ€Π°ΡΡˆΠΈΡ€ΠΈΠ² ΠΌΠ°Π³Π½ΠΈΡ‚Π½ΠΎΠ΅ ΠΏΠΎΠ»Π΅ Π² этом Π½Π°ΠΏΡ€Π°Π²Π»Π΅Π½ΠΈΠΈ.
  2. Если ΠΌΠ°Π³Π½ΠΈΡ‚Π½ΠΎΠ΅ ΠΏΠΎΠ»Π΅ растянуто Π² Π½Π°ΠΏΡ€Π°Π²Π»Π΅Π½ΠΈΠΈ, ΡƒΠΊΠ°Π·Π°Π½Π½ΠΎΠΌ Π² ΠΏΡƒΠ½ΠΊΡ‚Π΅ 1, частица Π²Ρ‹ΠΉΠ΄Π΅Ρ‚ ΠΈΠ· области Π²Π»Π΅Π²ΠΎ, Π·Π°Π²Π΅Ρ€ΡˆΠΈΠ² ΠΎΠ΄ΠΈΠ½ ΠΏΠΎΠ»ΡƒΠΊΡ€ΡƒΠ³. Π­Ρ‚ΠΎ ΠΏΡ€ΠΎΠΈΠ·ΠΎΡˆΠ»ΠΎ Π±Ρ‹ Π² любом случаС, ΠΊΠΎΠ³Π΄Π° частица ΠΏΠΎΠΏΠ°Π΄Π°Π΅Ρ‚ Π² ΠΎΠ±Π»Π°ΡΡ‚ΡŒ ΠΌΠ°Π³Π½ΠΈΡ‚Π½ΠΎΠ³ΠΎ поля: частица вырвСтся, пройдя ΠΏΠΎΠ»ΠΎΠ²ΠΈΠ½Ρƒ окруТности.
  3. Π§Ρ‚ΠΎΠ±Ρ‹ Π·Π°ΡΡ‚Π°Π²ΠΈΡ‚ΡŒ частицу Π΄Π²ΠΈΠ³Π°Ρ‚ΡŒΡΡ ΠΏΠΎ ΠΊΡ€ΡƒΠ³Ρƒ, ΠΌΠ°Π³Π½ΠΈΡ‚Π½ΠΎΠ΅ ΠΏΠΎΠ»Π΅ Π΄ΠΎΠ»ΠΆΠ½ΠΎ Π±Ρ‹Ρ‚ΡŒ ΠΏΡ€ΠΈΠ»ΠΎΠΆΠ΅Π½ΠΎ ΠΈΠ·Π²Π½Π΅, ΠΊΠ°ΠΊ Ρ‚ΠΎΠ»ΡŒΠΊΠΎ частица ΡƒΠΆΠ΅ находится Π² Π΅Π³ΠΎ области. ΠŸΠ»ΠΎΡ‰Π°Π΄ΡŒ ΠΌΠ°Π³Π½ΠΈΡ‚Π½ΠΎΠ³ΠΎ поля Ρ‚Π°ΠΊΠΆΠ΅ Π΄ΠΎΠ»ΠΆΠ½Π° Π±Ρ‹Ρ‚ΡŒ достаточно большой, Ρ‡Ρ‚ΠΎΠ±Ρ‹ ΠΎΡ…Π²Π°Ρ‚Ρ‹Π²Π°Ρ‚ΡŒ вСсь ΠΊΡ€ΡƒΠ³ΠΎΠ²ΠΎΠΉ ΠΏΡƒΡ‚ΡŒ, ΠΏΠΎ ΠΊΠΎΡ‚ΠΎΡ€ΠΎΠΌΡƒ Π±ΡƒΠ΄Π΅Ρ‚ Π΄Π²ΠΈΠ³Π°Ρ‚ΡŒΡΡ заряТСнная частица.

Π­Π»Π΅ΠΊΡ‚Ρ€ΠΎΠ½ Π±ΡƒΠ΄Π΅Ρ‚ Π΄Π²ΠΈΠ³Π°Ρ‚ΡŒΡΡ ΠΏΠΎ окруТности, Ссли ΠΎΠ½ Π²Ρ…ΠΎΠ΄ΠΈΡ‚ Π² ΠΎΠ΄Π½ΠΎΡ€ΠΎΠ΄Π½ΠΎΠ΅ ΠΌΠ°Π³Π½ΠΈΡ‚Π½ΠΎΠ΅ ΠΏΠΎΠ»Π΅ с постоянной ΡΠΊΠΎΡ€ΠΎΡΡ‚ΡŒΡŽ. Он ΠΏΡ€Π΅ΠΊΡ€Π°Ρ‚ΠΈΡ‚ своС ΠΊΡ€ΡƒΠ³ΠΎΠ²ΠΎΠ΅ Π΄Π²ΠΈΠΆΠ΅Π½ΠΈΠ΅, Ссли Π²Ρ‹ΠΉΠ΄Π΅Ρ‚ ΠΈΠ· области, содСрТащСй ΠΌΠ°Π³Π½ΠΈΡ‚Π½ΠΎΠ΅ ΠΏΠΎΠ»Π΅, Ρ‡Ρ‚ΠΎ происходит Π² Π½ΠΈΠΆΠ½Π΅ΠΉ части этой Π΄ΠΈΠ°Π³Ρ€Π°ΠΌΠΌΡ‹. Π˜Π·ΡƒΡ‡Π°ΠΉΡ‚Π΅ ΡƒΠΌΠ½Ρ‹Π΅ ΠΎΡ€ΠΈΠ³ΠΈΠ½Π°Π»Ρ‹.

Π’ΠΈΠΏΡ‹ частиц Π² ΠΌΠ°Π³Π½ΠΈΡ‚Π½Ρ‹Ρ… полях

ΠœΡ‹ Π²ΠΈΠ΄Π΅Π»ΠΈ, ΠΊΠ°ΠΊ элСктроны ΠΎΡ‚ΠΊΠ»ΠΎΠ½ΡΡŽΡ‚ΡΡ ΠΌΠ°Π³Π½ΠΈΡ‚Π½Ρ‹ΠΌΠΈ полями, Π½ΠΎ ΠΌΡ‹ ΠΌΠΎΠΆΠ΅ΠΌ Π½Π°Π±Π»ΡŽΠ΄Π°Ρ‚ΡŒ ΠΏΠΎΠ΄ΠΎΠ±Π½Ρ‹Π΅ отклонСния ΠΈ для Π΄Ρ€ΡƒΠ³ΠΈΡ… частиц. Π—Π΄Π΅ΡΡŒ ΠΌΡ‹ рассмотрим Ρ‚Ρ€ΠΈ ΠΈΠ· этих частиц; Π°Π»ΡŒΡ„Π°-частица, Π±Π΅Ρ‚Π°-частица ΠΈ Π³Π°ΠΌΠΌΠ°-частица .

ΠΠ»ΡŒΡ„Π°-, Π±Π΅Ρ‚Π°- ΠΈ Π³Π°ΠΌΠΌΠ°-частицы Π² ΠΌΠ°Π³Π½ΠΈΡ‚Π½ΠΎΠΌ ΠΏΠΎΠ»Π΅

Из Π°Ρ‚ΠΎΠΌΠΎΠ² ΠΈ излучСния ΠΌΡ‹ Π·Π½Π°Π΅ΠΌ, Ρ‡Ρ‚ΠΎ Π°Π»ΡŒΡ„Π°-частицы ΠΏΡ€Π΅Π΄ΡΡ‚Π°Π²Π»ΡΡŽΡ‚ собой ядра гСлия (ΠΎΠ½ΠΈ содСрТат Π΄Π²Π° Π½Π΅ΠΉΡ‚Ρ€ΠΎΠ½Π° ΠΈ Π΄Π²Π° ΠΏΡ€ΠΎΡ‚ΠΎΠ½Π°, Ρ‡Ρ‚ΠΎ Π΄Π΅Π»Π°Π΅Ρ‚ ΠΈΡ… ΠΏΠΎΠ»ΠΎΠΆΠΈΡ‚Π΅Π»ΡŒΠ½ΠΎ заряТСнными). Они Ρ‚Π°ΠΊΠΆΠ΅ довольно тяТСлыС, ΠΏΠΎ ΠΊΡ€Π°ΠΉΠ½Π΅ΠΉ ΠΌΠ΅Ρ€Π΅, ΠΏΠΎ ΡΡ€Π°Π²Π½Π΅Π½ΠΈΡŽ с элСктроном, Π° это ΠΎΠ·Π½Π°Ρ‡Π°Π΅Ρ‚, Ρ‡Ρ‚ΠΎ трСбуСтся большая магнитная сила, Ρ‡Ρ‚ΠΎΠ±Ρ‹ ΠΎΡ‚ΠΊΠ»ΠΎΠ½ΠΈΡ‚ΡŒ ΠΈΡ… Π½Π° Ρ‚Ρƒ ΠΆΠ΅ Π²Π΅Π»ΠΈΡ‡ΠΈΠ½Ρƒ.

Π‘Π΅Ρ‚Π°-частицы β€” это быстро двиТущиСся элСктроны, поэтому ΠΎΠ½ΠΈ Π΄Π΅ΠΌΠΎΠ½ΡΡ‚Ρ€ΠΈΡ€ΡƒΡŽΡ‚ Ρ‚ΠΎ ΠΆΠ΅ ΠΏΠΎΠ²Π΅Π΄Π΅Π½ΠΈΠ΅ отклонСния, ΠΊΠΎΡ‚ΠΎΡ€ΠΎΠ΅ ΠΌΡ‹ наблюдали Ρ€Π°Π½Π΅Π΅. На рисункС Π½ΠΈΠΆΠ΅ ΠΏΠΎΠΊΠ°Π·Π°Π½Π° Ρ€Π°Π·Π½ΠΈΡ†Π° ΠΌΠ΅ΠΆΠ΄Ρƒ Ρ‚Π΅ΠΌ, ΠΊΠ°ΠΊ Π°Π»ΡŒΡ„Π°- ΠΈ Π±Π΅Ρ‚Π°-частицы ΠΎΡ‚ΠΊΠ»ΠΎΠ½ΡΡŽΡ‚ΡΡ ΠΎΠ΄Π½ΠΈΠΌ ΠΈ Ρ‚Π΅ΠΌ ΠΆΠ΅ ΠΎΠ΄Π½ΠΎΡ€ΠΎΠ΄Π½Ρ‹ΠΌ ΠΌΠ°Π³Π½ΠΈΡ‚Π½Ρ‹ΠΌ ΠΏΠΎΠ»Π΅ΠΌ ΠΏΡ€ΠΈ Π΄Π²ΠΈΠΆΠ΅Π½ΠΈΠΈ Ρ‡Π΅Ρ€Π΅Π· Π½Π΅Π³ΠΎ.

Π“Π°ΠΌΠΌΠ°-частицы ΠΏΡ€Π΅Π΄ΡΡ‚Π°Π²Π»ΡΡŽΡ‚ собой Ρ„ΠΎΡ‚ΠΎΠ½Ρ‹ высокой энСргии, испускаСмыС ΠΏΡ€ΠΈ Ρ€Π°Π΄ΠΈΠΎΠ°ΠΊΡ‚ΠΈΠ²Π½ΠΎΠΌ распадС. Они часто ΠΈΡΠΏΡƒΡΠΊΠ°ΡŽΡ‚ΡΡ вмСстС с Π°Π»ΡŒΡ„Π°- ΠΈ Π±Π΅Ρ‚Π°-частицами. Однако большая Ρ€Π°Π·Π½ΠΈΡ†Π° Π² Ρ‚ΠΎΠΌ, Ρ‡Ρ‚ΠΎ это Ρ„ΠΎΡ‚ΠΎΠ½Ρ‹ ΠΈ, ΡΠ»Π΅Π΄ΠΎΠ²Π°Ρ‚Π΅Π»ΡŒΠ½ΠΎ, нСзаряТСнных . Они Π½Π΅ Π±ΡƒΠ΄ΡƒΡ‚ ΠΎΡ‚ΠΊΠ»ΠΎΠ½ΡΡ‚ΡŒΡΡ ΠΌΠ°Π³Π½ΠΈΡ‚Π½Ρ‹ΠΌ ΠΏΠΎΠ»Π΅ΠΌ. На ΠΏΡ€ΠΈΠ²Π΅Π΄Π΅Π½Π½ΠΎΠΉ Π½ΠΈΠΆΠ΅ Π΄ΠΈΠ°Π³Ρ€Π°ΠΌΠΌΠ΅ ΠΏΠΎΠΊΠ°Π·Π°Π½Ρ‹ ΠΏΡƒΡ‚ΠΈ Π°Π»ΡŒΡ„Π°-, Π±Π΅Ρ‚Π°- ΠΈ Π³Π°ΠΌΠΌΠ°-частиц, ΠΊΠΎΠ³Π΄Π° ΠΎΠ½ΠΈ двиТутся ΠΏΠ΅Ρ€Π²ΠΎΠ½Π°Ρ‡Π°Π»ΡŒΠ½ΠΎ ΠΏΠΎ прямой Π»ΠΈΠ½ΠΈΠΈ Π² ΠΎΠ±Π»Π°ΡΡ‚ΡŒ, ΡΠΎΠ΄Π΅Ρ€ΠΆΠ°Ρ‰ΡƒΡŽ ΠΌΠ°Π³Π½ΠΈΡ‚Π½ΠΎΠ΅ ΠΏΠΎΠ»Π΅.

ΠΠ»ΡŒΡ„Π°- ΠΈ Π±Π΅Ρ‚Π°-частицы Π±ΡƒΠ΄ΡƒΡ‚ ΠΎΡ‚ΠΊΠ»ΠΎΠ½ΡΡ‚ΡŒΡΡ, Ссли ΠΏΠΎΠΏΠ°Π΄ΡƒΡ‚ Π² ΠΎΠ±Π»Π°ΡΡ‚ΡŒ с ΠΌΠ°Π³Π½ΠΈΡ‚Π½Ρ‹ΠΌ ΠΏΠΎΠ»Π΅ΠΌ. Частицы ΠΎΡ‚ΠΊΠ»ΠΎΠ½ΡΡŽΡ‚ΡΡ Π² ΠΏΡ€ΠΎΡ‚ΠΈΠ²ΠΎΠΏΠΎΠ»ΠΎΠΆΠ½Ρ‹Ρ… направлСниях, Ρ‚Π°ΠΊ ΠΊΠ°ΠΊ ΠΎΠ½ΠΈ заряТСны ΠΏΡ€ΠΎΡ‚ΠΈΠ²ΠΎΠΏΠΎΠ»ΠΎΠΆΠ½ΠΎ. ΠΠ»ΡŒΡ„Π°-частица отклоняСтся мСньшС, ΠΏΠΎΡ‚ΠΎΠΌΡƒ Ρ‡Ρ‚ΠΎ ΠΎΠ½Π° Π½Π°ΠΌΠ½ΠΎΠ³ΠΎ тяТСлСС Π±Π΅Ρ‚Π°-частицы. Π“Π°ΠΌΠΌΠ°-частица Π½Π΅ отклоняСтся Π² ΠΌΠ°Π³Π½ΠΈΡ‚Π½ΠΎΠΌ ΠΏΠΎΠ»Π΅, Ρ‚Π°ΠΊ ΠΊΠ°ΠΊ ΠΎΠ½Π° Π½Π΅ заряТСна. StudySmarter Originals

ΠΠ»ΡŒΡ„Π°-частица отклоняСтся Π² мСньшСй стСпСни, Π΄Π°ΠΆΠ΅ Ссли ΠΎΠ½Π° ΠΈΠΌΠ΅Π΅Ρ‚ больший заряд, Ρ‡Π΅ΠΌ Π±Π΅Ρ‚Π°-частица. Π­Ρ‚ΠΎ ΠΏΠΎΡ‚ΠΎΠΌΡƒ, Ρ‡Ρ‚ΠΎ ΠΎΠ½ тяТСлСС (ΠΈΠΌΠ΅Π΅Ρ‚ Π±ΠΎΠ»ΡŒΡˆΡƒΡŽ массу) ΠΈ силС Ρ‚Ρ€ΡƒΠ΄Π½Π΅Π΅ Π΅Π³ΠΎ ΠΎΡ‚ΠΊΠ»ΠΎΠ½ΠΈΡ‚ΡŒ. Π”Π²Π΅ частицы ΠΎΡ‚ΠΊΠ»ΠΎΠ½ΡΡŽΡ‚ΡΡ Π² ΠΏΡ€ΠΎΡ‚ΠΈΠ²ΠΎΠΏΠΎΠ»ΠΎΠΆΠ½Ρ‹Ρ… направлСниях ΠΈΠ·-Π·Π° Ρ‚ΠΎΠ³ΠΎ, Ρ‡Ρ‚ΠΎ ΠΈΡ… заряды ΠΈΠΌΠ΅ΡŽΡ‚ ΠΏΡ€ΠΎΡ‚ΠΈΠ²ΠΎΠΏΠΎΠ»ΠΎΠΆΠ½Ρ‹Π΅ Π·Π½Π°ΠΊΠΈ (Π°Π»ΡŒΡ„Π°-частицы ΠΏΠΎΠ»ΠΎΠΆΠΈΡ‚Π΅Π»ΡŒΠ½Ρ‹Π΅, Π° Π±Π΅Ρ‚Π°-частицы ΠΎΡ‚Ρ€ΠΈΡ†Π°Ρ‚Π΅Π»ΡŒΠ½Ρ‹Π΅), создавая ΠΌΠ°Π³Π½ΠΈΡ‚Π½Ρ‹Π΅ силы Π² ΠΏΡ€ΠΎΡ‚ΠΈΠ²ΠΎΠΏΠΎΠ»ΠΎΠΆΠ½Ρ‹Ρ… направлСниях. Π“Π°ΠΌΠΌΠ°-частица Π½Π΅ взаимодСйствуСт с ΠΌΠ°Π³Π½ΠΈΡ‚Π½Ρ‹ΠΌ ΠΏΠΎΠ»Π΅ΠΌ ΠΈ ΠΏΡ€ΠΎΡ…ΠΎΠ΄ΠΈΡ‚ Ρ‡Π΅Ρ€Π΅Π· ΠΎΠ±Π»Π°ΡΡ‚ΡŒ Π±Π΅Π· отклонСния.

Частицы Π² ΠΌΠ°Π³Π½ΠΈΡ‚Π½Ρ‹Ρ… полях β€” основныС Π²Ρ‹Π²ΠΎΠ΄Ρ‹

  • ΠœΠ°Π³Π½ΠΈΡ‚Π½Π°Ρ сила β€” это сила, ΠΊΠΎΡ‚ΠΎΡ€ΡƒΡŽ испытываСт заряТСнная частица (элСктрон, ΠΏΡ€ΠΎΡ‚ΠΎΠ½, ΠΈΠΎΠ½ ΠΈ Ρ‚. Π΄.), ΠΊΠΎΠ³Π΄Π° ΠΎΠ½Π° двиТСтся Ρ‡Π΅Ρ€Π΅Π· ΠΌΠ°Π³Π½ΠΈΡ‚Π½ΠΎΠ΅ ΠΏΠΎΠ»Π΅.

  • ΠœΠ°Π³Π½ΠΈΡ‚Π½ΠΎΠ΅ ΠΏΠΎΠ»Π΅ β€” это ΠΎΠ±Π»Π°ΡΡ‚ΡŒ Π² пространствС, Π² ΠΊΠΎΡ‚ΠΎΡ€ΠΎΠΉ двиТущийся заряд ΠΈΠ»ΠΈ постоянный ΠΌΠ°Π³Π½ΠΈΡ‚ испытываСт силу.

  • И заряд, ΠΈ Π΄Π²ΠΈΠΆΠ΅Π½ΠΈΠ΅ частицы Π½Π΅ΠΎΠ±Ρ…ΠΎΠ΄ΠΈΠΌΡ‹ для проявлСния силы ΠΏΠΎΠ»Π΅ΠΌ.

  • Π‘ΠΈΠ»Π°, Π΄Π΅ΠΉΡΡ‚Π²ΡƒΡŽΡ‰Π°Ρ ΠΌΠ°Π³Π½ΠΈΡ‚Π½Ρ‹ΠΌ ΠΏΠΎΠ»Π΅ΠΌ силы Π½Π° Π·Π°Ρ€ΡΠΆΠ΅Π½Π½ΡƒΡŽ частицу, Π΄Π²ΠΈΠΆΡƒΡ‰ΡƒΡŽΡΡ со ΡΠΊΠΎΡ€ΠΎΡΡ‚ΡŒΡŽ, опрСдСляСтся ΠΏΠΎ ΠΏΡ€ΠΈΠ²Π΅Π΄Π΅Π½Π½ΠΎΠΉ Π½ΠΈΠΆΠ΅ Ρ„ΠΎΡ€ΠΌΡƒΠ»Π΅. НаправлСниС силы пСрпСндикулярно Π½Π°ΠΏΡ€Π°Π²Π»Π΅Π½ΠΈΡŽ двиТСния частицы ΠΈ ΠΌΠ°Π³Π½ΠΈΡ‚Π½ΠΎΠ³ΠΎ поля ΠΈ опрСдСляСтся Π²Ρ‹Ρ€Π°ΠΆΠ΅Π½ΠΈΠ΅ΠΌ:

  • ДвиТущаяся заряТСнная частица Π² области с ΠΎΠ΄Π½ΠΎΡ€ΠΎΠ΄Π½Ρ‹ΠΌ ΠΌΠ°Π³Π½ΠΈΡ‚Π½Ρ‹ΠΌ ΠΏΠΎΠ»Π΅ΠΌ двиТСтся ΠΏΠΎ ΠΊΡ€ΡƒΠ³ΠΎΠ²ΠΎΠΉ Ρ‚Ρ€Π°Π΅ΠΊΡ‚ΠΎΡ€ΠΈΠΈ.

alexxlab

Π”ΠΎΠ±Π°Π²ΠΈΡ‚ΡŒ ΠΊΠΎΠΌΠΌΠ΅Π½Ρ‚Π°Ρ€ΠΈΠΉ

Π’Π°Ρˆ адрСс email Π½Π΅ Π±ΡƒΠ΄Π΅Ρ‚ ΠΎΠΏΡƒΠ±Π»ΠΈΠΊΠΎΠ²Π°Π½. ΠžΠ±ΡΠ·Π°Ρ‚Π΅Π»ΡŒΠ½Ρ‹Π΅ поля ΠΏΠΎΠΌΠ΅Ρ‡Π΅Π½Ρ‹ *